OB Hesi Final

Réussis tes devoirs et examens dès maintenant avec Quizwiz!

How much glucose is ingested and what is a good result that will to requiem additional testing?

-->Pt will ingestion 50g of oral glucose solution --> Blood sampling is taken 1-hour later and if less than 130-140 then they will to need additional testing (Oral Glucose Challenge Test).

A pregnant patient would like to know which foods, other than dairy products, contain the most calcium. Which food group would the nurse recommend? A. Legumes B. Lean meat C. Whole grains D. Yellow vegetables

ANS: A Rationale: Although dairy products contain the greatest amount of calcium, it can also be found in legumes, nuts, dried fruits, and some dark green leafy vegetables. Lean meats are rich in protein and phosphorus. Whole grains are rich in zinc and magnesium. Yellow vegetables are rich in vitamin A.

A nurse is discussing diet with a pregnant client who is 5 feet 4 inches tall (163 cm) and whose pre-pregnancy weight was 120 lb (54 kg). What should the nurse include about the changes in calories and nutrients, compared with the pre-pregnancy diet, during the second trimester? 1. Decreasing daily fat consumption by 220 calories 2. Increasing total daily caloric intake by 340 calories 3. Increasing total daily caloric intake by 460 calories 4. Decreasing daily carbohydrate consumption by 130 calories

Correct 2. Increasing total daily caloric intake by 340 calories

A client at 10 weeks' gestation phones the prenatal clinic to report that she is experiencing some vaginal bleeding and abdominal cramping. The nurse arranges for her to go to the local hospital. The vaginal examination reveals that her cervix is dilated 2 cm. What diagnosis should the nurse expect? 1. Septic abortion 2. Inevitable abortion 3. Threatened abortion 4. Incomplete abortion

Correct 2. Inevitable abortion

When teaching a client about using a diaphragm as a form of contraception, what instructions should the nurse provide about the diaphragm? 1. It may or may not be used with a spermicidal lubricant. 2. It should remain in place for at least 6 hours after intercourse. 3. It must be inserted with the dome facing down to be maximally effective. 4. It often appears puckered but this will not interfere with its effectiveness.

Correct 2. It should remain in place for at least 6 hours after intercourse.

Select the priority intervention for a pregnant client whose monitor strip shows fetal heart rate decelerations characterized by a rapid descent and ascent to and from the lowest point of the deceleration. 1. Elevating the legs 2. Repositioning the client from side to side 3. Increasing the rate of intravenous infusion 4. Administering oxygen by way of face mask

Correct 2. Repositioning the client from side to side

Which nursing intervention is specific to clients in active labor who present with a history of cardiac disease? 1. Encouraging frequent voiding 2. Checking the blood pressure hourly 3. Auscultating the lungs for crackles every 30 minutes 4. Helping turn the client from side to side at 15-minute intervals

Correct 3. Auscultating the lungs for crackles every 30 minutes

A pregnant woman with a history of heart disease visits the prenatal clinic toward the end of her second trimester. Which intervention does the nurse anticipate will be part of this client's care plan? 1. Preparation for a cesarean birth 2. Bed rest during the last trimester 3. Prophylactic antibiotics at the time of birth 4. Increasing dosages of cardiac medications as pregnancy progresses

Correct 3. Prophylactic antibiotics at the time of birth

What's the term for: Identifying presence of fetal erythrocytes in the maternal circulation

K-B test

47. A prenatal client in her second trimester is admitted to the maternity unit with painless, bright-red vaginal bleeding. What test might the physician order? a. Alpha-fetoprotein (AFP) b. Contraction stress test (CST) c. Amniocentesis d. Ultrasound

Answer is D. Rationale: US is noninvasive & would show placenta previa which is what the s/s indicate. Too early in pregnancy for CST. Amniocentesis is not indicated for the s/s.

What's the term for: Blocking effect of a drug

Antagonist

The nurse discusses fetal weight gain with a pregnant client. When does the fetus generally show a marked increase in size? 1. During the third trimester 2. During the second trimester 3. At the end of the first trimester 4. No difference is observed.

Correct 1. During the third trimester

While changing her baby girl's diaper, a client expresses concern about a small spot of red vaginal discharge on the diaper. How should the nurse respond to this concern? 1. Explain that this is an expected finding 2. Obtain a prescription for vaginal cultures 3. Assess the infant for other signs of bleeding 4. Apply a urine specimen bag to the perineum

Correct 1. Explain that this is an expected finding

The nurse is reviewing a client's history. Which two predisposing causes of puerperal (postpartum) infection should prompt the nurse to monitor this client closely? 1. Malnutrition and anemia 2. Hemorrhage and trauma during labor 3. Preeclampsia and retention of placental fragments 4. Organisms in the birth canal and trauma during labor

Correct 2. Hemorrhage and trauma during labor

What's the term for: Alpha-fetoprotein, human chorionic gonadotropin, unconjugated estriol

Alpha-fetoprotein (AFP)

What's the term for: Infusion of a sterile solution into the amniotic cavity to reduce cord compression

Amnioinfusion

What's the term for: Loss of sensation, with or without loss of consciousness

Anesthesia

43. A multigravida at 38 weeks' gestation is admitted with painless, bright redbleeding and mild contractions every 7 to 10 minutes. Which of the followingassessments should be avoided? a. Maternal vital signs b. Fetal heart rate c. Contraction monitoring d. Cervical dilation

Answer is D. Rationale: SVE is not performed to assess cervical dilatation or effacement as S/S indicate a possible placenta previa which would be aggravated by the exam. VS, FHTs, & monitoring of contractions is noninvasive & should be performed to establish baseline maternal/fetal data.

What's the term for: Pinkish color on a wet diaper

Brick dust stain

What's the term for: Tissue designed for newborn heat production

Brown Fat

What's the term for: Maternal substances secreted in responses to stress

Catecholamines

Define: Discoloration of the cervix, labia, and vagina in early pregnancy

Chadwick's sign

What's the term for: Technique to oxygenate the blood while bypassing the lungs

Extracorporeal membrane oxygenation

Define: Decent of the fetus into the pelvis, reducing pressure on the diaphragm

Lightening

Define: Brownish discoloration of the face

Melasma

What's the term for: Protrusion of meninges through a spinal defect

Meningocele

What's the term for: Long-acting drug to substitute for heroin or morphine

Methadone

What's the term for: Change in the shape of the fetal head during birth

Molding

What's the term for: Withdrawing amniotic fluid for laboratory examination

Multiple-marker screen

What's the term for: Protusion of meninges and spinal cord through a vertebral defect

Myelomeningocele

Lab Values: Platelets

Non-pregnant: 165,000 - 415,000 Pregnant Pt.: 146,000 - 429,000

Define: Ingestion of non-food substances

Pica

What's the term for: Rupture of the membranes before the onset of labor

Premature rupture of the membranes (PROM)

Define: First fetal movements felt by the mother

Quickening

What's the term for: Thrombus formation with inflammation

Thrombophlebitis

What's the term for: A blood clot within a vessel

Thrombus

What's the term for: Drug that reduces uterine muscle contractions

Tocolytic

What's the term for: Medication to stop preterm of hypertonic uterine contractions

Tocolytic

Define: One of the three 13-week periods of pregnancy

Trimester

What's the term for: Postbirth uterine contractions

Afterpains

What's the term for: Causing a physiologic effect

Agonist

What's the term for: Results from withdrawal from certain drugs

Abstinence Syndrome

What's the term for: Bluish color of the hands and feet

Acrocyanosis

What's the term for: Relief of pain without loss of consciousness

Analgesia

While performing bag-and-mask ventilation on a newborn, a nurse does not see the newborn's chest rise. Place the following interventions in order of their priority. *Reposition the head *Hold the mask using the "E-C clamp" for a better seal *Suction the mouth if there are secretions *Assess the neonate's response to these measures *Cover the newborns nose and mouth with the mask

Correct1. Suction the mouth if there are secretions Correct2. Reposition the head Correct3. Cover the newborns nose and mouth with the mask Correct4. Hold the mask using the "E-C clamp" for a better seal Correct5. Assess the neonate's response to these measures

Define: Rituals or symptoms experienced by some fathers-to-be

Couvade syndrome

What's the term for: Opening of the cervix

Dilation

What's the term for: Difficult or prolonged labor

Dystocia

What's the term for: Thinning of the cervix

Effacement

Define: Unstable mood

Emotional Lability

Define: a reproductive cell

Gamete cell that was 23 chromosomes, half number

Define: Excess blood in a body part

Hyperemia

What's the term for: Adverse condition resulting from treatment

Iatrogenic

What's the term for: Breast inflammation

Mastitis

What's the term for: Cord around the neck

Nuchal cord

Define: Abnormally sell quantity of amniotic fluid

Oligohydramnios

What's the term for: Intestines protruding into the cord

Omphalocele

What's the term for: Drugs as morphine, heroin, methadone, and meperdine

Opiates

What's the term for: Milk in the stomach before savage feeding

Residual

What's the term for: Scrotal skin creases

Rugae

What's the term for: Delayed or difficult birth of the shoulders after the head has emerged

Shoulder dystocia

What's the term for: "crossed" eyes

Strabismus

The nurse is performing a gestational age assessment using the New Ballard Scale. The infant's total neuromuscular score is 16, and total physical maturity score is 20. According to the graph, at how many weeks' gestation is the newborn? Record your answer using a whole number. ____ weeks

The fetus is at 38 weeks' gestation.

What's the term for: Fluctuations in the baseline fetal heart rate

Variability

The upper uterus is the best place for the fertilized ovum to implant due to which anatomical adaptation? a. Maternal blood flow is lower. b. Placenta attaches most firmly. c. Uterine endometrium is softer. d. Developing baby is best nourished.

ANS: D Rationale: The uterine fundus is richly supplied with blood and has the thickest endometrium, both of which promote optimum nourishment of the fetus. The blood supply is rich in the fundus, which allows for optimal nourishment of the fetus. If the placenta attaches too deeply, it does not easily detach. Softness is not a concern with implantation; attachment and nourishment are the major concerns.

What's the term for: Precedes true milk

Colostrum

What's the term for: Muscle tension when the uterus is not contracting

Uterine resting tone

A client with preeclampsia has a prescription for a magnesium sulfate infusion to be initiated. The nurse assesses the client's status to obtain baseline information. Which assessments are necessary? Select all that apply. 1. Patellar reflex 2. Output of urine 3. Respiratory rate 4. Body temperature 5. Urine specific gravity

Correct 1. Patellar reflex 2. Output of urine 3. Respiratory rate

What's the term for: Newborn scalp edema caused by a vacuum extractor

Chignon

Define: Cells that remain after ovum formation and secrete estrogen and progesterone

Corpus Luteum

What's the term for: Bilirubin staining of the skin and sclerae

Jaundice

What's the term for: Hormone that causes cartilage to soften

Relaxin

The nurse discusses the recommended weight gain during pregnancy with a newly pregnant client who is 5 ft 3 in (160 centimeters) tall and weighs 130 lb (57 kilograms). The nurse explains that with the recommended weight gain, at term the client should weigh how much? 1. 155 lb (70 kg) 2. 140 lb (63.5 kg) 3. 135 lb (61 kg) 4. 130 lb (57 kg)

Correct 1. 155 lb (70 kg)

A nurse is assessing a postpartum client 6 hours after delivery. Which sign should immediately alert the nurse that the client is hemorrhaging? 1. Decrease in pulse rate 2. Increase in blood pressure 3. Continuous trickling of blood 4. Persistent muscle twitching

Correct 3. Continuous trickling of blood

A newborn's Apgar score at 5 minutes is 5. Which condition correlates with this low Apgar score? 1. Cerebral palsy 2. Genetic defects 3. Mental retardation 4. Neonatal morbidity

Correct 4. Neonatal morbidity

What's the term for: Holding an infant skin to skin

Kangaroo care

Define: Irregular reddish streaks caused by tears in connective tissue (stretch marks)

Striae Gravidarum

When should iron supplementation during a normal pregnancy begin? A. Before pregnancy B. In the first trimester C. In the third trimester D. In the second trimester

ANS: D Rationale: Vitamin supplements should be prescribed in the second trimester, when the need for iron is increased. Healthy young women do not usually need iron supplementation for their diets. Morning sickness in the first trimester increases the routine side effects of iron supplements. The iron supplements may continue to be prescribed in the third trimester and during the postpartum period.

Lab Values: Fibrinogen (is a blood plasma protein that's made in the liver, and is a coagulation factors responsible for normal blood clotting)

Non-Pregnant: 233-496 Pregnant Pt.: 244 - 696

What's the term for: Measurements of descents of the fetal presenting part into the pelvis

Station

When a pregnant woman develops ptyalism, which guidance should the nurse provide? A. Chew gum or suck on lozenges between meals. B. Eat nutritious meals that provide adequate amounts of essential vitamins and minerals. C. Take short walks to stimulate circulation in the legs and elevate the legs periodically. D. Use pillows to support the abdomen and back during sleep.

ANS: A Rationale: Some women experience ptyalism, or excessive salivation. The cause of ptyalism may be decreased swallowing associated with nausea or stimulation of the salivary glands by the ingestion of starch. Small frequent meals and use of chewing gum and oral lozenges offer limited relief for some women. All other options include recommendations for pregnant women; however, they do not address ptyalism.

A patient with a BMI of 32 has a positive pregnancy test. What is the maximum number of pounds that the nurse will advise the patient gain during the pregnancy? a. 20lb b. 25lb c. 28lb d. 40lb

ANS: A Rationale: The weight gain for obese women is 5 to 9 kg (11 to 20 lb). A BMI of 30 or higher categorizes the patient as obese. The other options refer to minimal or maximal weight gain for patients in other BMI categories.

Which findings are presumptive signs of pregnancy? (Select all that apply.) A. Quickening B. Amenorrhea C. Ballottement D. Goodell's sign E. Chadwick's sign

ANS: A,B,E Rationale: Quickening, amenorrhea, and Chadwick's sign are presumptive signs of pregnancy.Ballottement and Goodell's sign are probable signs of pregnancy.

What's the term for: Fetal substance used to screen for specific abnormalities

Amniocentesis

14. A client with type 1 diabetes mellitus who's a multigravidavisits the clinic at 27 weeks gestation. The nurse should instruct the client that for most pregnant women with type 1 diabetes mellitus: a. Daily kick counts are done by the mother. b. Contraction stress testing is performed weekly. c. Induction of labor is begun at 34 weeks' gestation. d. Nonstresstesting is performed weekly until 32 weeks' gestation.

Answer is A. Rationale: Daily kick counts assess fetal well-being & are noninvasive. CST is not initiated when uterine activity is undesirable (i.e. 27 wks. Gestation) Induction @ 34 weeks is risky, due to potential for lung immaturity. NST would more likely be performed after 32 weeks than before.

26. FHR can be auscultated with a fetoscope as early as which of the following? a. 5 weeks gestation b. 10 weeks gestation c. 15 weeks gestation d. 20 weeks gestation

Answer is D. Rationale: FHTs are audible by fetoscope @ 18-20 weeks. FHTs can be auscultated as early as 10 weeks by doppler.

61. The client asks for information about ectopic pregnancy. The nurse correctly responds by saying ectopic pregnancy is the implantation of a fertilized ovum: a. In the fundusof the uterus. b. In the vaginal vault. c. At the lower part of the uterus. d. In a site other than the endometrial lining of the uterus.

Answer is D. Rationale: Ectorefers to "outside"- ectopic refers to a pregnancy which is implanted outside the uterus Fundal implantation would be "normal" Implantation in lower uterine segment might be a placenta previa depending on location Implantation in vagina is not possible

32. When developing a plan of care for a patient newly diagnosed with gestational diabetes, which of the following instructions would be the priority? a. Dietary intake b. Medication c. Exercise d. Glucose monitoring

Answer is D. Rationale: Priority is glucose monitoring in an attempt tomaintain normoglycemia. Teaching would also include dietary intake, exercise & medications.

A client is scheduled to have a contraction stress test (CST) to determine fetal well-being. Which type of fetal heart rate (FHR) decelerations constitutes a nonreassuring outcome? 1. Late 2. Early 3. Baseline 4. Variable

Correct 1. Late

Which pregnant client does the nurse suspect is most likely to have placenta previa? 1. 19 years old, gravida 1, para 0 2. 30 years old, gravida 6, para 5 3. 25 years old, gravida 2, para 1 4. 40 years old, gravida 3, para 2

Correct 2. 30 years old, gravida 6, para 5

Which complication is a primipara with a second-degree laceration and repair most likely to experience during the postpartum period? 1. Posterior vaginal varicosities 2. Difficulty voiding spontaneously 3. Delayed onset of milk production 4. Maladaptive bonding with the newborn

Correct 2. Difficulty voiding spontaneously

The primary healthcare provider prescribes a contraction stress test (CST) for a client whose nonstress test (NST) was nonreactive. Which maternal complications should prompt the nurse to question the prescription? Select all that apply. 1. Hypertension 2. Preterm labor 3. Drug addiction 4. Incompetent cervix 5. Premature rupture of membranes

Correct 2. Preterm labor 3. Drug addiction 4. Incompetent cervix 5. Premature rupture of membranes

A client is receiving magnesium sulfate therapy for severe preeclampsia. What initial sign of toxicity should prompt the nurse to intervene? 1. Hyperactive sensorium 2. Increase in respiratory rate 3. Lack of the knee-jerk reflex 4. Development of a cardiac dysrhythmia

Correct 3. Lack of the knee-jerk reflex

A woman arrives at the women's health clinic complaining of frequency and burning pain when voiding. The diagnosis is a urinary tract infection. What is important for the nurse to encourage the client to do? 1. Void every 2 hours. 2. Record fluid intake and urinary output. 3. Pour warm water over the vulva after voiding. 4. Wash the hands thoroughly after urinating and defecating.

Correct 4. Wash the hands thoroughly after urinating and defecating.

The nurse is caring for a postpartum client who has experienced an abruptio placentae. Which assessment indicates that disseminated intravascular coagulation (DIC) is occurring? 1. Boggy uterus 2. Hypovolemic shock 3. Multiple vaginal clots 4.Bleeding at the venipuncture site

Correct 4.Bleeding at the venipuncture site

What's the term for: Interest centered on self, rather than on others

Egocetrism

What's the term for: A clot or amniotic fluid material that may obstruct smaller vessels

Embolism

What's the term for: Removal of small amounts of blood and replacement with donor blood

Exchange transfusion

What's the term for: Learned breathing techniques used during labor

Paced breathing

What's the term for: Temporary recurrent changes in the fetal heart rate

Periodic fetal heart rate changes

What's the term for: Forceps used to assist vaginal breech birth

Piper

What's the term for: Placenta that adheres abnormally to the uterine wall

Placenta accreta

What's the term for: Air leakage into the chest cavity

Pneumothorax

Define: Abnormally large quantity of amniotic fluid

Polyhydramnios

What's the term for: Rupture of the membranes before the end of week 37

Pre-term premature rupture of the membranes (PPROM)

What's the term for: Slippery substance that reduces surface tension in lung aveoli

Surfactant

What's the term for: Substance in the alveoli that reduces surface tension

Surfactant

VEAL CHOP

V: variable decels C: cord compression (Nursing Interventions: Reposition, O2, notify Dr) E: early decels H: head compression Normal A: accels O: okay L: late decels P: placental deficiency (Nursing Interventions: Turn pt, give O2, d/c Pitocin, notify Dr.)

In childbirth classes the nurse is teaching paced breathing techniques for use during labor. In which order should the breathing techniques be used as labor progresses? *Slow, deep breaths *Cleansing breaths *Pant-blow breathing *Slow, exhalation pushing *Modified-paced breathing

Correct 1. Cleansing breaths Correct 2. Slow, deep breaths Correct 3. Modified-paced breathing Correct 4. Pant-blow breathing Correct 5. Slow, exhalation pushing

Lab Values: Hemoglobin

Non-pregnant: 12-16 Pregnant pt: 10.5-11 ( may be anemic if less than 11 in the 1st or 3rd trimester, or less than 10.5 in the 2nd trimester

Lab Values: RBC's

Non-pregnant: 4-5.2 Pregnant Pt.: 2.71-4.55 (Decrease slightly because of hemodilution)

lab Values: WBC's

Non-pregnant: 5,000 - 10,000 Pregnant Pt.: 5,000 - 15,000

Lab Values: Blood Glucose, fasting

Non-pregnant: 70 - 100 Pregnant Pt.: 95 or lower

Lab Values: 24Hr creatinine clearance

Non-pregnant: 91 - 130 Pregnant Pt.: 50 - 166

What's the term for: Production of heat by the use of specialized fat

Non-shivering themogenesis

56. The nurse is counseling a prenatal client regarding the need to take folic acid supplements during pregnancy. The nurse also encourages the client to eat foods high in folic acid, such as: a. fruits and fruit juice. b. rice and pasta. c. eggs and yogurt. d. fresh green leafy vegetables and legumes.

Answer is D. Rationale: Fruits & fruit juice- high in Vitamin C; juices may be fortified with Calcium & Folic acid Dried fruits- high in iron Rice & pasta- provide complex CHO, fiber, Thiamine, Folic acid, Vitamin B6, riboflavin, Niacin Eggs- high in Vitamins B6, B12, D, A, riboflavin, iron, & zinc Yogurt- high in calcium Fresh green leafy vegetables - high in Folic acid, Vitamins K, E, C, Iron, & Magnesium Legumes- high in Folic acid, Thiamine, Calcium, Magnesium, & Zinc

17. A primigravidaclient at about 35 weeks gestation is in active labor. She has had no prenatal care and admits to cocaine use during the pregnancy. Which of the following persons must the nurse notify? (Select all that apply.) a. Nursing unit manager so appropriate agencies can be notified b. Head of the hospital's security department c. Chaplain, in case the fetus dies in utero d. Physician who will attend the delivery of the infant

Answers A & D. CPS will be involved due to child endangerment. Physician needs to be alerted that potential complications may occur. Security is not an issue according to data provided. Chaplain would not be notified unless patient requested.

How does the nurse explain physiologic anemia to a pregnant client? 1.Erythropoiesis decreases. 2.Plasma volume increases. 3.Utilization of iron decreases. 4.Detoxification by the liver increases.

Correct 2.Plasma volume increases.

A client tells a nurse in the prenatal clinic that she has vaginal staining but no pain. Her history reveals amenorrhea for the last 2 months and pregnancy confirmation after her first missed period. What type of abortion is suspected? 1. Missed 2. Inevitable 3. Threatened 4. Incomplete

Correct 3. Threatened

A Nitrazine test strip that turns deep blue indicates that the fluid being tested has a pH of what? 1. 4.5 2. 5.5 3. 6.5 4. 7.5

Correct 4. 7.5

A client in the high-risk postpartum unit has had a precipitous labor and birth. Which maternal complication should the nurse anticipate? 1. Hypertension 2. Hypoglycemia 3. Chilling and shivering 4. Bleeding and infection

Correct 4. Bleeding and infection

Which of the following factors would affect pain perception or tolerance for the laboring patient? A. Right occiput posterior fetal position during labor B. Bishop score of 10 prior to the induction of labor C. Gynecoid pelvis D. Absence of Ferguson's reflex

ANS: A RATIONALE: A fetus in the posterior position during labor can cause increased back pain to the mother because it is spine against spine. A Bishop score of 10 indicates that conditions are favorable for induction; the cervix is soft, anterior, effaced, and dilated and the presenting part is engaged. A gynecoid pelvic structure is considered to be an adequate passage for vaginalbirth. Ferguson's reflex occurs when a contraction is stimulated as a result of vaginal stimulation.

Which patient is a candidate for internal monitoring with an intrauterine pressure catheter? A. Obese patient whose contractions are 3 to 6 minutes apart, lasting 20 to 50 seconds B. Gravida 1, para 0, whose contractions are 2 to 3 minutes apart, lasting 60 seconds C. Multigravida whose contractions are 2 minutes apart, lasting 60 to 70 seconds D. Gravida 2, para 1, in latent phase whose contractions are irregular and mild

ANS: A RATIONALE: A thick layer of abdominal fat absorbs energy from uterine contractions, reducing their apparent intensity on the monitor strip. Contraction patterns of 2 to 3 minutes lasting 60 seconds and every 2 minutes lasting 60 to 70 seconds indicate accurate measurement of uterine activity. Irregular and mild contractions are common in the latent phase.

Excessive anxiety during labor heightens the patient's sensitivity to pain by increasing A. muscle tension. B. the pain threshold. C. blood flow to the uterus. D. rest time between contractions.

ANS: A RATIONALE: Anxiety and fear increase muscle tension, diverting oxygenated blood to the woman's brain and skeletal muscles. Prolonged tension results in general fatigue, increased pain perception, and reduced ability to use coping skills. Anxiety will decrease the pain threshold. Anxiety can decrease blood flow to the uterus. Anxiety will decrease the amount of rest the mother gets between contractions.

An increase in urinary frequency and leg cramps after the 36th week of pregnancy are an indication of A. lightening. B. breech presentation. C. urinary tract infection. D. onset of Braxton-Hicks contractions.

ANS: A RATIONALE: As the fetus descends toward the pelvic inlet near the end of pregnancy, increased pelvic pressure occurs, resulting in greater urinary frequency and more leg cramps. Breech presentation does not cause urinary frequency and leg cramps. A urinary tract infection may cause urinary frequency but with burning and would not cause leg cramps. Braxton-Hicks contractions are irregular and mild and occur throughout the pregnancy.

In a prenatal education class, the nurse is reviewing the importance of using relaxation techniques during labor. Which patient statement will the nurse need to correct? A. "We will practice relaxation techniques only in a quiet setting so I can focus." B. "Relaxation is important during labor because it will help me conserve my energy." C. "If I relax in between contractions, my baby will get more oxygen during labor." D. "My partner and I will practice relaxation throughout the remainder of my pregnancy."

ANS: A Rationale: Relaxation exercises must be practiced frequently to be useful during labor. Couples begin practice sessions in a quiet, comfortable setting. Later, they practice in other places that simulate the noise and unfamiliar setting of the hospital. The ability to relax during labor is an important component of coping effectively with childbirth. Relaxation conserves energy, decreases oxygen use, and enhances other pain relief techniques. Women learn exercises to help them recognize and release tension. The labor partner assists the woman by providing feedback during exercise sessions and labor.

The traditional diet of Asian women includes little meat and few dairy products and may be low in calcium and iron. The nurse can assist a patient increase her intake of these foods by which action? A. Suggest that she eat more tofu, bok choy, and broccoli. B. Suggest that she eat more hot foods during pregnancy. C. Emphasize the need for increased milk intake during pregnancy. D. Tell her husband that she must increase her intake of fruits and vegetables for the baby's sake.

ANS: A Rationale: The diet should be improved by increasing foods acceptable to the woman. These foods are common in the Asian diet and are good sources of calcium and iron. Pregnancy is considered hot; therefore the woman would eat cold foods. Because milk products are not part of thiswoman's diet, it should be respected and other alternatives offered. Also, lactose intoleranceis common. Fruits and vegetables are cold foods and should be included in the diet. In regard to the family dynamics, however, the husband does not dictate to the wife in this culture.

The nurse is teaching a pregnant patient about food safety during pregnancy and lactation. Which statements by the patient indicate she understood the teaching? (Select all that apply.) A. "I will limit my intake of shrimp to 12 oz a week." B. "I will avoid the soft cheeses made with unpasteurized milk." C. "I plan to continue to pack my bologna sandwich for lunch." D. "I am glad I can still go to the sushi bar during my pregnancy." E. "I will not eat any swordfish or shark while I am pregnant or nursing."

ANS: A,B,E Rationale: Statements that indicate the patient understood the teaching are limiting shrimp to 12 oz a week, avoiding soft cheeses, and not eating any swordfish. A bologna sandwich should be avoided unless it is reheated until steaming hot. Raw or undercooked fish should be avoided.

The nurse is preparing supplies for an amnioinfusion on a patient with intact membranes. Which supplies should the nurse gather? (Select all that apply.) A. Extra underpads B. Solution of 3% normal saline C. Amniotic hook to perform an amniotomy D. Solid intrauterine pressure catheter with a pressure transducer on its tip

ANS: A,C RATIONALE: Amnioinfusion is performed with lactated Ringer's solution or normal saline, not 3%. Normalsaline is infused into the uterus through an intrauterine pressure catheter (IUPC). The underpads must be changed regularly because fluid leaks out constantly. The membranes need to be ruptured before an amnioinfusion can be initiated so an amniotic hook will be needed. The IUPC must have a double lumen to run the infusion through.

. The nurse is advising a lactose-intolerant pregnant patient about calcium intake. Which calcium sources are approximately equivalent to 1 cup of milk? (Select all that apply.) a. 3/4 cup yogurt b. 1 cup of sherbet c. 1 1/4 oz of hard cheese d. 1 1/4 cups of ice cream e. 3/4 cup of low-fat cottage cheese

ANS: A,C,D Rationale: Calcium sources approximately equivalent to 1 cup of milk include cup yogurt, ounce of hard cheese, and cups of ice cream. It takes 3 cups of sherbet and low-fat cottage cheese to equal the calcium equivalent of 1 cup of milk.

The nurse is instructing a patient on how to perform kick counts. Which information should the nurse include in the teaching session? (Select all that apply.) A. Use a clock or timer when performing kick counts. B. Your bladder should be full before performing kick counts. C. Notify your health care provider if you have not felt movement in 24 hours. D. Protocols can provide a structured timetable for concentrating on fetal movements. E. You should lie on your side, place your hands on the largest part of the abdomen, and concentrate on the number of movements felt.

ANS: A,D,E Rationale: The nurse should instruct the patient to lie on her side, place her hands on the largest part of her abdomen, and concentrate on fetal movements. She should use a clock or timer and record the number of movements felt during that time. Protocols are not essential; however, they may give the patient a more structured timetable for when to concentrate on fetal movements. The bladder does not need to be full for kick counts; it is better to have the patient empty her bladder before beginning the assessment of fetal movements. Further evaluation is recommended if the patient feels no movements in 12 hours; 24 hours is too long before notifying the health care provider.

The nurse is assessing a patient in the active phase of labor. What should the nurse expect during this phase? A. The patient is sociable and excited. B. The patient is requesting pain medication. C. The patient begins to experience the urge to push. D. The patient experiences loss of control and irritability.

ANS: B RATIONAL: During the active phase of labor, contraction intensity and discomfort increase to the point where women often request pain medication. Sociability and excitability occur during the latent phase. The urge to push occurs at the end of the transition phase or the second stage of labor. Loss of control and irritability occur during the transition phase of labor.

A multipara's labor plan includes the use of jet hydrotherapy during the active phase of labor. What is the priority patient assessment prior to assisting the patient with this request? A. Maternal pulse B. Maternal temperature C. Maternal blood pressure D. Maternal blood glucose level

ANS: B RATIONALE: A shower, tub bath, or whirlpool bath is relaxing and provides thermal stimulation. Several studies have shown benefits of water therapy during labor, including immersion in a tub or whirlpool (jet hydrotherapy, or Jacuzzi). The major concern regarding immersion therapy has been newborn and postpartum maternal infections caused by microorganisms in the water.Infections can be caused by the woman's own ascending vaginal bacteria or by preexistingorganisms in an improperly cleaned tub. Several studies have not found a significant association between newborn or postpartum maternal infections and the use of immersion hydrotherapy with proper cleaning

The physician has ordered an amnioinfusion for the laboring patient. Which data supports the use of this therapeutic procedure? A. Presenting part not engaged B. +4 meconium-stained amniotic fluid on artificial rupture of membranes (AROM) C. Breech position of fetus D. Twin gestation

ANS: B RATIONALE: Amnioinfusion is a procedure utilized during labor when cord compression or the detection of gross meconium staining is found in the amniotic fluid. An isotonic (Lactated Ringers or normal saline) solution is used as an irrigation method through the IUPC (intrauterine pressure catheter).

Which method of intrapartum fetal monitoring is the most appropriate when a woman has a history of hypertension during pregnancy? A. Continuous auscultation with a fetoscope B. Continuous electronic fetal monitoring C. Intermittent assessment with a Doppler transducer D. Intermittent electronic fetal monitoring for 15 minutes each hour

ANS: B RATIONALE: Maternal hypertension may reduce placental blood flow through vasospasm of the spiral arteries. Reduced placental perfusion is best assessed with continuous electronic fetal monitoring to identify patterns associated with this condition. It is not practical to provide continuous auscultation with a fetoscope. This fetus needs continuous monitoring because it is at high risk for complications.

Which statement correctly describes the nurse's responsibility related to electronicmonitoring? A. Report abnormal findings to the physician before initiating corrective actions. B. Teach the woman and her support person about the monitoring equipment and discuss any of their questions. C. Document the frequency, duration, and intensity of contractions measured by the external device. D. Inform the support person that the nurse will be responsible for all comfort measures when the electronic equipment is in place.

ANS: B RATIONALE: Teaching is an essential part of the nurse's role. Corrective actions should be initiated first to correct abnormal findings as quickly as possible. Electronic monitoring will record the contractions and FHR response. The support person should still be encouraged to assist with the comfort measures.

A patient whose cervix is dilated to 6 cm is considered to be in which phase of labor? A. Latent phase B. Active phase C. Second stage D. Third stage

ANS: B RATIONALE: The active phase of labor is characterized by cervical dilation of 5 to 6 cm. Historically, the latent phase is from the beginning of true labor until 3 cm of cervical dilation. Recent research has suggested that the latent phase be considered to last up until 5 to 6 cm. dilated. The second stage of labor begins when the cervix is completely dilated until the birth of the baby. The third stage of labor is from the birth of the baby until the expulsion of the placenta.

Which factor ensures that the smallest anterior-posterior diameter of the fetal head enters the pelvis? a. Station b. Flexion c. Descent d. Engagement

ANS: B RATIONALE: The anterior-posterior diameter of the head varies with how much it is flexed. In the most favorable situation, the head is fully flexed and the anterior-posterior diameter is the suboccipitobregmatic, averaging 9.5 cm. The station is the relationship of the fetal presenting part to the level of the ischial spine. Descent is the moving of the fetus through the birth canal. Engagement occurs when the largest diameter of the fetal presenting part has passed the pelvic outlet.

Which advice to the patient is one of the most effective methods for preventing venous stasis? A. Sit with the legs crossed. B. Rest often with the feet elevated. C. Sleep with the foot of the bed elevated. D. Wear elastic stockings in the afternoon.

ANS: B Rationale: Elevating the feet and legs improves venous return and prevents venous stasis. Sitting with the legs crossed will decrease circulation in the legs and increase venous stasis. Elevating the legs at night may cause pressure on the diaphragm and increase breathing problems. Elastic stockings should be applied before lowering the legs in the morning.

The nurse is scheduling the next appointment for a healthy primigravida currently at 28 weeks gestation. When will the nurse schedule the next prenatal visit? A. 1 week B. 2 weeks C. 3 weeks D. 4 weeks

ANS: B Rationale: From 29 to 36 weeks, routine prenatal assessment is every 2 weeks. If the pregnancy is high risk, the patient will see the health care provider more frequently.

A patient in her fifth month of pregnancy asks the nurse, "How many more calories should I be eating daily?" What is the correct response by the nurse? A. 180 more calories a day B. 340 more calories a day C. 452 more calories a day D. 500 more calories a day

ANS: B Rationale: The increased nutritional needs of pregnancy can be met with an additional 340 calories per day. 180 calories are not enough to meet the increased nutritional needs of pregnancy. 452 calories are more than the recommended calories for pregnancy at this gestation. A patient in her third trimester would increase her energy intake by 452 calories per day. 500 calories are more than the recommended calories for pregnancy.

When a Category II pattern of the fetal heart rate is noted and the patient is lying on her left side, which nursing action is indicated? A. Lower the head of the bed. B. Place a wedge under the left hip. C. Change her position to the right side. D. Place the mother in Trendelenburg position.

ANS: C RATIONALE: A Category II pattern indicates an indeterminate fetal heart rate. Repositioning on the opposite side may relieve compression on the umbilical cord and improve blood flow to the placenta. Lowering the head of the bed would not be the first position change choice. The woman is already on her left side, so a wedge on that side would not be an appropriate choice. Repositioning to the opposite side is the first intervention. If unsuccessful with improving the FHR pattern, further changes in position can be attempted; the Trendelenburg position might be the choice.

An infant is diagnosed with fetal anemia. Which information would support this clinical diagnosis? a. Presence of excess maternal hormones b. Maternal blood type O-negative, Rh-negative, and infant blood type O-negative, Rh-negative c. Passive immunity d. Rh-negative mother and Rh-positive baby

ANS: C Rationale: Passive immunity provides temporary protection to the baby based on the transfer of maternal antibodies. Maternal hormones would not lead to a clinical diagnosis of fetal anemia. These blood types and Rh factors are the same; therefore, no antibodies will be created. In this situation, an Rh-negative mother and Rh-positive baby will result in stimulation of antibodies that will stimulate a reaction leading to hemolysis.

One of the assessments performed in the birth room is checking the umbilical cord for blood vessels. Which finding is considered to be within normal limits? a. One artery and one vein b. Two veins and one artery c. Two arteries and one vein d. Two arteries and two veins

ANS: C Rationale: The umbilical cord contains two arteries and one vein to transport blood between the fetus and the placenta. Any option other than two arteries and one vein is considered abnormal and requires further assessment. Two veins and one artery is abnormal and may indicate an anomaly. Two arteries instead is a normal finding; this infant would require further assessment for anomalies due to the finding of two veins.

Physiologic anemia often occurs during pregnancy due to A. inadequate intake of iron. B. the fetus establishing iron stores. C. dilution of hemoglobin concentration. D. decreased production of erythrocytes.

ANS: C Rationale: When blood volume expansion is more pronounced and occurs earlier than the increase in red blood cells, the woman will have physiologic anemia, which is the result of dilution of hemoglobin concentration rather than inadequate hemoglobin. Inadequate intake of iron may lead to true anemia. If the woman does not take an adequate amount of iron, true anemia may occur when the fetus pulls stored iron from the maternal system. There is increased production of erythrocytes during pregnancy.

The nurse detects hypotension in a laboring patient after an epidural. Which actions should the nurse plan to implement? (Select all that apply.) A. Encourage the patient to drink fluids. B. Place the patient in a Trendelenburg position. C. Administer a normal saline bolus as prescribed. D. Administer oxygen at 8 to 10 L/minute per face mask. E. Administer IV ephedrine in 5- to 10-mg increments as prescribed.

ANS: C,D,E RATIONALE: If hypotension occurs after an epidural has been placed, techniques such as a rapid nondextrose IV fluid bolus, maternal repositioning, and oxygen administration are implemented. If those interventions are ineffective, IV ephedrine in 5- to 10-mg increments can be prescribed to promote vasoconstriction to raise the blood pressure. The patient in active labor should not be encouraged to drink fluids. In a Trendelenburg position, the body is flat, with the feet elevated. This would not be a position to use for a pregnant patient.

Which clinical conditions are associated with increased levels of alpha fetoprotein (AFP)? (Select all that apply.) A. Down syndrome B. Molar pregnancy C. Twin gestation D. Incorrect gestational age assessment of a normal fetus—estimation is earlier in the pregnancy E. Threatened abortion

ANS: C,D,E Rationale: Elevated APF levels are seen in multiple gestations, underestimation of fetal age, and threatened abortion. Decreased levels are seen in Down syndrome and a molar pregnancy.

The nurse is preparing a patient for a nonstress test (NST). Which interventions should the nurse plan to implement? (Select all that apply.) A. Ensure that the patient has a full bladder. B. Plan approximately 15 minutes for the test. C. Have the patient sit in a recliner with the head elevated 45 degrees. D. Apply electronic monitoring equipment to the patient's abdomen. E. Instruct the patient to press an event marker every time she feels fetal movement.

ANS: C,D,E Rationale: The patient may be seated in a reclining chair or have her head elevated at least 45 degrees. The nurse applies external electronic monitoring equipment to the patient's abdomen to detectthe fetal heart rate and any contractions. The woman may be given an event marker to press each time she senses movement. Before the NST, the patient should void and her baseline vital signs should be taken. The NST takes about 40 minutes, allowing for most fetal sleep-wake cycles, although the fetus may show a reassuring pattern more quickly or need more time to awaken and become active. Fifteen minutes would not allow enough time to complete the test.

An expectant couple asks the nurse about intercourse during pregnancy and whether it is safe for the baby. What information should the nurse provide? A. Intercourse is safe until the third trimester. B. Safer sex practices should be used once the membranes rupture. C. Intercourse should be avoided if any spotting from the vagina occurs afterward. D. Intercourse and orgasm are often contraindicated if a history of or signs of preterm labor are present.

ANS: D Rationale: Uterine contractions that accompany orgasm can stimulate labor and would be problematic if the woman is at risk for or has a history of preterm labor. Intercourse can continue as long as the pregnancy is progressing normally. Rupture of the membranes may require abstaining from intercourse. Safer sex practices are always recommended. Some spotting can normally occur as a result of the increased fragility and vascularity of the cervix and vagina during pregnancy.

In order to increase the absorption of iron by a pregnant patient, which beverage should an iron preparation be given with? A. Tea B. Milk C. Coffee D. Orange juice

ANS: D Rationale: Vitamin C source may increase the absorption of iron and would be the optimal choice. Tannin in the tea reduces the absorption of iron. The calcium and phosphorus in milk decrease iron absorption. Decreased intake of caffeine is recommended during pregnancy.

The nurse is administering fentanyl (Sublimaze) to a patient in labor. The health careprovider's prescription reads fentanyl (Sublimaze), 100 mcg IV stat. The medication vialreads fentanyl (Sublimaze), 50 mcg/mL. The nurse should prepare how many milliliters to administer the correct dose? Record your answer as a whole number. _____ mL

ANS:2 Desired/available X volume = milliliters per dose 100 mcg/50 mcg X 1 mL = 2 mL/dose

The nurse is teaching a client who is formula feeding her infant how to care for her engorged breasts. Which statement by the client demonstrates that teaching was understood? 1. "I am wearing a well-fitting, tight brassiere." 2. "I am drinking 10 glasses of liquid every day." 3. "I am expressing milk from my breasts every 4 hours." 4. "I am letting warm water run over my breasts while I'm showering."

Correct 1. "I am wearing a well-fitting, tight brassiere."

A pregnant woman tells a nurse, "I think I can feel the baby move now. It feels like butterflies in my stomach. My friend calls it feeling life." What term should the nurse include when discussing fetal movement with the woman? 1. Lightening 2. Quickening 3. Engagement 4. Ballottement

Correct 2. Quickening

What is the optimal method for the nurse to use to assess blood loss in a client with placenta previa? 1. Count or weigh perineal pads 2. Monitor pulse and blood pressure 3. Check hemoglobin and hematocrit values 4. Measure or estimate the height of the fundus

Correct 1. Count or weigh perineal pads

A client's membranes ruptured 20 hours before admission. The client was in labor for 24 hours before giving birth. For which postpartum complication is this client at risk? 1. Infection 2. Hemorrhage 3. Uterine atony 4. Amniotic fluid embolism

Correct 1. Infection

Which of the following is the priority intervention for the patient in a left side-lying position whose monitor strip shows a deceleration that extends beyond the end of the contraction? A. Administer O2 at 8 to 10 L/minute. B. Decrease the IV rate to 100 mL/hour. C. Reposition the ultrasound transducer. D. Perform a vaginal exam to assess for cord prolapse.

ANS: A RATIONALE: A deceleration that returns to baseline after the end of the contraction is a late deceleration caused by placental perfusion problems. Administering oxygen will increase the patient'sblood oxygen saturation, making more oxygen available to the fetus. Decreasing the IV rate, repositioning the ultrasound transducer, and performing a vaginal exam to assess for cord prolapse are not effective interventions to improve fetal oxygenation.

Which maternal factor may inhibit fetal descent during labor? A. A full bladder B. Decreased peristalsis C. Rupture of membranes D. Reduction in internal uterine size

ANS: A RATIONALE: A full bladder may inhibit fetal descent because it occupies space in the pelvis needed by the fetal presenting part. Peristalsis does not influence fetal descent. Rupture of membranes will assist in the fetal descent. Contractions will reduce the internal uterine size to assist fetal descent.

The health care provider's prescription reads diphenhydramine (Benadryl), 25 mg IV stat. The medication vial reads diphenhydramine (Benadryl), 50 mg/mL. The nurse should prepare how many milliliters to administer the correct dose? Record your answer to one decimal point. _____ mL

ANS:0.5 Desired/available X volume = milliliters per dose 25 mg/50 mg X 1 mL = 0.5 mL/dose

20. Which foods should the nurse recommend to increase a pregnant patient's calcium intake? a. Legumes and enriched cereals. b. Tuna and cottage cheese. c. Green, leafy vegetables. d. Cheese and yogurt

Answer D. Milk and milk products, such as cheese and yogurt, are the best sources of calcium. Legumes are high in protein. Tuna is high in niacin & B12. Green, leafy vegetables are high in folic acid. Enriched cereals are high in folic acid, B6, B12, & niacin.

58. The nurse is providing prenatal care to an asymptomatic HIV- infected client. Which nursing intervention should take priority? a. Taking her temperature b. Performing a hearing test c. Performing a vision test d. Assessing reflexes

Answer is A. Rationale: Concern with HIV patient is r/f infection due to immunocompromised status. Hearing, vision & reflexes are not directly r/t HIV infection.

What's the term for: Intense back pain associated with fetal occiput posterior position

Back labor

Define: Fetal rebound in te amniotoic fluid when the cervix is tapped

Ballottement

What's the term for: Fetal heart rate when the uterus is at rest

Baseline fetal heart rate

When is a Glucose Challenge Test completed?

Between 24 and 28 weeks of gestation

On her first visit to the prenatal clinic, a client with rheumatic heart disease asks the nurse whether she has any special nutritional needs. What supplements in addition to the regular pregnancy diet and prenatal vitamin and minerals will she need? Select all that apply. 1. Iron 2. Calcium 3. Folic acid 4. Vitamin C 5. Vitamin B12

Correct 1. Iron 3. Folic acid

Define: sampling of placental tissue for microscopic and chemical examination to detect fetal abnormalities

Chorionic villous sampling

What's the term for: Deep breath taken at the beginning and end of each contraction

Cleansing breath

What's the term for: hemolysis, elevated liver enzymes, low platelet count may occur in preeclampsia

HELLP syndrome

What is an Oral Glucose Challenge Test?

It is the 3-hour GOLD STANDARD for diagnosing diabetes, which requires fasting from midnight the night before. The plasma glucose level will be withdrawn when fasting, and every hour after that for three hours and if two or more of the values meet or exceed the threshold limits then a physician can diagnosis the patient with Gestational Diabetes Mellitus.

What's the term for: Permanent neurologic damage from bilirubin

Kernicterus

What's the term for: Failure of the uterus to return to its pre-pregnant state in the time expected

Subinvolution

A pregnant client is admitted with abdominal pain and heavy vaginal bleeding. What is the priority nursing action? 1. Administering oxygen 2. Elevating the head of the bed 3. Drawing blood for a hematocrit level 4. Giving an intramuscular analgesic

Correct 1. Administering oxygen

Which clinical finding can be determined only by electronic fetal monitoring? A. Variability B. Tachycardia C. Bradycardia D. Fetal response to contractions

ANS: A RATIONALE: Beat-to-beat variability cannot be determined by auscultation because auscultation provides only an average fetal heart rate (FHR) as it fluctuates. Tachycardia can be determined by any of the FHR monitoring techniques. Bradycardia can be determined by any of the FHR monitoring techniques. The fetal response to the contractions is usually noted by an increase or decrease in fetal heart rate. These can be determined by any of the FHR monitoring techniques.

Which clinical finding would be considered normal for a preterm fetus during the labor period? A. Baseline tachycardia B. Baseline bradycardia C. Fetal anemia D. Acidosis

ANS: A RATIONALE: Because the nervous system is immature, it is expected that the preterm fetus will have a baseline tachycardia because of stimulation of the sympathetic nervous system. Baseline bradycardia, fetal anemia, and acidosis would indicate abnormal findings and fetal compromise.

To determine if the patient is in true labor, the nurse would assess for changes in A. cervical dilation. B. amount of bloody show. C. fetal position and station. D. pattern of uterine contractions.

ANS: A RATIONALE: Cervical changes are the only indication of true labor and are used to determine true and false labor. Changes in the amount of bloody show, fetal position and station, and pattern of uterine contractions are unreliable indicators of true labor.

A client's membranes rupture during labor, and the amniotic fluid is meconium stained. Which heart rate pattern indicates that the fetus's status is nonreassuring? 1. Early decelerations with average variability 2. Changes in baseline variability from 5 to 10 beats/min 3. Increases in fetal heart rate from 135 to 150 beats/min with fetal activity 4. Variable decelerations that last 60 seconds, then return to baseline tachycardia

Correct 4. Variable decelerations that last 60 seconds, then return to baseline tachycardia

Five minutes after birth, a newborn is pale; has irregular, slow respirations; has a heart rate of 120 beats/min; displays minimal flexion of the extremities; and has minimal reflex responses. What is this newborn's Apgar score? Record your answer using a whole number. _____

Correct 5

Lab Values: Creatinine

Non-pregnant: 0.5 - 0.9 Pregnant Pt.: 0.4 - 0.9

A laboring patient who imagines her body opening to let the baby out is using a mental technique called a. imagery. b. effleurage. c. distraction. d. dissociation.

ANS: A RATIONALE: Imagery is a technique of visualizing images that will assist the woman in coping with labor. Effleurage is self-massage. Distraction can be used in the early latent phase by having the woman involved in another activity. Dissociation helps the woman learn to relax all muscles except those that are working.

Decelerations that mirror the contractions are present with each contraction on the monitor strip of a multipara who received epidural anesthesia 20 minutes ago. The nurse should A. maintain the normal assessment routine. B. administer O2 at 8 to 10 L/minute by face mask. C. increase the IV flow rate from 125 to 150 mL/hour. D. assess the maternal blood pressure for a systolic pressure below 100 mm Hg.

ANS: A RATIONALE: Decelerations that mirror the contraction are early decelerations caused by fetal head compression. Early decelerations are not associated with fetal compromise and require no intervention. Administering O2, increasing the IV flow rate, and assessing for hypotension are not necessary within early decelerations.

The nurse is instructing a nursing student on the application of fetal monitoring devices. Which method of assessing the fetal heart rate requires the use of a gel? A. Doppler B. Fetoscope C. Scalp electrode D. Tocodynamometer

ANS: A RATIONALE: Doppler is the only listed method involving ultrasonic transmission of fetal heart rates; it requires the use of a gel. The fetoscope does not require gel because ultrasonic transmission is not used. The scalp electrode is attached to the fetal scalp; gel is not necessary. The tocodynamometer does not require gel. This device monitors uterine contractions.

The nurse is monitoring a patient in labor and notes this fetal heart rate pattern on the electronic fetal monitoring strip (see figure). Which is the most appropriate nursing action? A.Administer oxygen with a face mask at 8 to 10 L/minute. B. Reposition the fetal monitor ultrasound transducer. C. Assist the patient to the bathroom to empty her bladder. D. Continue to monitor the patient and fetal heart rate patterns.

ANS: A RATIONALE: Late decelerations are similar to early decelerations in that the FHR slows (30 to 40 bpm); however, the decelerations are shifted to the right in relation to the contraction. They often begin after the peak of the contraction. They reflect possible impaired placental exchange (uteroplacental insufficiency). Administration of 100% oxygen through a snug face mask makes more oxygen available for transfer to the fetus. A commonly suggested rate is 8 to 10 L/minute. The pattern is abnormal, so repositioning the fetal ultrasound transducer, assisting the patient to the bathroom, or continuing to monitor the pattern will not correct the problem.

The nurse is reviewing an electronic fetal monitor tracing from a patient in active labor and notes the fetal heart rate gradually drops to 20 beats per minute (bpm) below the baseline andreturns to the baseline well after the completion of the patient's contractions. How will thenurse document these findings? A. Late decelerations B. Early decelerations C. Variable decelerations D. Proximal decelerations

ANS: A RATIONALE: Late decelerations are similar to early decelerations in the degree of FHR slowing and lowest rate (30 to 40 bpm) but are shifted to the right in relation to the contraction. They often begin after the peak of the contraction. The FHR returns to baseline after the contraction ends. The early decelerations mirror the contraction, beginning near its onset and returning to the baseline by the end of the contraction, with the low point (nadir) of the deceleration occurringnear the contraction's peak. The rate at the lowest point of the deceleration is usually no lower than 30 to 40 bpm from the baseline. Conditions that reduce flow through the umbilical cord may result in variable decelerations. These decelerations do not have the uniform appearance of early and late decelerations. Their shape, duration, and degree of fall below baseline rate vary. They fall and rise abruptly (within 30 seconds) with the onset and relief of cord compression, unlike the gradual fall and rise of early and late decelerations. Proximal deceleration is not a recognized term.

After birth of the placenta the patient states, "All of a sudden I feel very cold." What is themost appropriate nursing action at this time? A. Place a warm blanket over the patient. B. Place the baby on the patient's abdomen. C. Tell the patient that chills are expected after birth. D. "What do you mean by your words 'very cold'?"

ANS: A RATIONALE: Many women are chilled after birth. The cause of this reaction is unknown but probably relates to the sudden decrease in effort, loss of the heat produced by the fetus, decrease in intraabdominal pressure, and fetal blood cells entering the maternal circulation. The chill lasts for about 20 minutes and subsides spontaneously. A warm blanket, hot drink, or soup may help relieve the chill and make the woman more comfortable. Placing the baby on her abdomen may result in transfer of heat and make her feel even colder. Reassurance is appropriate after the blanket is provided. Validation of an expected physical response to the birthing process results in a delay of care and is unnecessary.

The primary difference between the labor of a nullipara and that of a multipara is A. total duration of labor. B. level of pain experienced. C. amount of cervical dilation. D. sequence of labor mechanisms.

ANS: A RATIONALE: Multiparas usually labor more quickly than nulliparas, making the total duration of their labor shorter. The level of pain is individual to the woman, not the number of labors she has experienced. Cervical dilation is the same for all labors. The sequence of labor mechanisms is the same with all labors.

A woman with a known heroin habit is admitted in early labor. Which drug is contraindicated with opiate-dependent patients? A. Nalbuphine (Nubain) B. Hydroxyzine (Vistaril) C. Promethazine (Phenergan) D. Diphenhydramine (Benadryl)

ANS: A RATIONALE: Nalbuphine may precipitate withdrawal if given to an opiate-dependent woman. Hydroxyzine is an antihistamine with antiemetic effects. Promethazine usually relieves nausea and vomiting. Diphenhydramine is commonly used to relieve pruritus from epidural narcotics.

A patient is admitted to the labor and birth room in active labor; contractions are 4 to 5 minutes apart and last for 30 seconds. The nurse needs to perform a detailed assessment. When is the best time to ask questions or perform procedures? A. After the contraction is over B. When it is all right with the coach C. During the increment of the next contraction D. After administration of analgesic-anesthetic

ANS: A RATIONALE: Reduce intrusions as much as possible. Longer assessments may span several contractions. The coach is the support person. The woman needs to feel confident in her ability to go through labor and birth, and she should be encouraged to express her own needs and concerns. The increment is the beginning of the next contraction. It is best to stop with questions and procedures during each contraction. An analgesic or anesthetic may cause adverse reactions in the woman, preventing her from answering questions correctly.

A laboring patient asks the nurse how she will know that the contraction is at its peak. The nurse explains that the contraction peaks during which stage of measurement? A. The acme B. The interval C. The increment D. The decrement

ANS: A RATIONALE: The acme is the peak or period of greatest strength during the middle of a contraction cycle. The interval is the period between the end of the contraction and the beginning of the next. The increment is the beginning of the contraction until it reaches the peak. The decrement occurs after the peak until the contraction ends.

The best time to teach nonpharmacologic pain control methods to an unprepared laboring patient is during which stage? A. Latent phase B. Active phase C. Second stage D. Transition phase

ANS: A RATIONALE: The latent phase of labor is the best time for intrapartum teaching because the woman is usually anxious enough to be attentive yet comfortable enough to understand the teaching. During the active phase, the woman is focused internally and unable to concentrate on teaching. During the second stage, the woman is focused on pushing. She normally handles the pain better at this point because she is active in doing something to hasten the birth. During transition, the woman is focused on keeping control; she is unable to focus on anyone else or learn at this time.

In which situation would a baseline fetal heart rate of 160 to 170 bpm be considered a normal finding? A. The fetus is at 30 weeks of gestation. B. The mother has a history of fast labors. C. The mother has been given an epidural block. D. The mother has mild preeclampsia but is not in labor.

ANS: A RATIONALE: The normal preterm fetus may have a baseline rate slightly higher than the term fetus because of an immature parasympathetic nervous system that does not yet exert a slowing effect on the fetal heart rate (FHR). Fast labors should not alter the FHR normally. Any change in the FHR with an epidural is not considered an expected outcome. Preeclampsia should not cause a normal elevation of the FHR.

A patient just delivered her baby via the vaginal route. The patient asks the nurse why thebaby's head is not round, but oval. Which explanation should the nurse provide the patient? A. This results from molding. B. This results from lightening. C. This results from the fetal lie. D. This results from the fetal presentation.

ANS: A RATIONALE: The sutures and fontanels allow the bones of the fetal head to move slightly, changing the shape of the fetal head so it can adapt to the size and shape of the pelvis. Lightening is the descent of the fetus toward the pelvic inlet before labor. Lie is the relationship of the long axis of the fetus to the long axis of the mother. Presentation is the fetal part that first enters the pelvic outlet.

A labor patient has brought in a photograph of her two children and asks the nurse to place it on the wall so that she can look at it during labor contractions. This is an example of A. focal point. B. distraction. C. effleurage. D. relaxation.

ANS: A RATIONALE: The use of a focal point (image and/or point reference in the labor room) is an example ofnonpharmacologic pain control during labor. The image of the patient's children is not servingas a method of distraction. Effleurage is the use of massage techniques to minimize painperception. The image of the patient's children is not serving as a method of relaxation.

A patient at 41 weeks' gestation is undergoing an induction of labor with an IV administration of oxytocin (Pitocin). The fetal heart rate starts to demonstrate a recurrent pattern of latedecelerations with moderate variability. What is the nurse's priority action? A. Stop the infusion of Pitocin. B. Reposition the patient from her right to her left side. C. Perform a vaginal exam to assess for a prolapsed cord. D. Prepare the patient for an emergency cesarean birth.

ANS: A RATIONALE: There are multiple reasons for late decelerations. Address the probable cause first, such as uterine hyperstimulation with Pitocin, to alleviate the outcome of late decelerations. Repositioning can increase oxygenation to the fetus but does not address the cause of the problem. Variable decelerations are more often seen with a prolapsed cord. In the presence of moderate variability, the fetus continues to have adequate oxygen reserves. If a Category II (indeterminate) or III (abnormal) tracing is interpreted, a prompt approach to assessing oxygenation should be completed.

Which maternal condition should be considered a contraindication for the application of internal monitoring devices? A. Unruptured membranes B. Cervix dilated to 4 cm C. Fetus has known heart defect D. Maternal HIV

ANS: A RATIONALE: To apply internal monitoring devices, the membranes must be ruptured. Cervical dilation of 4 cm would permit the insertion of fetal scalp electrodes and an intrauterine catheter. A compromised fetus should be monitored with the most accurate monitoring devices. An internal electrode should not be placed if the patient has hemophilia, maternal HIV, or genital herpes.

The nurse is providing care to a patient in the active phase of the first stage of labor. Thepatient is crying out loudly with each contraction. What is the nurse's most respectfulapproach for this patient? A. Ask the patient's labor coach if this is a usual expression of pain for her. B. Refer to the patient's chart to determine any orders for pain medication. C. Tell the patient that she is disturbing the other laboring patients on the unit. D. Encourage the patient to try to suppress her noisiness during contractions.

ANS: A RATIONALE: Women should be encouraged to express themselves in any way they find comforting, and the diversity of their expressions must be respected. Loud and vigorous expression may be awoman's personal pain coping mechanism, whereas a quiet woman may need medication relief but feels the need to remain stoic. Accepting a woman's individual response to laborand pain promotes a therapeutic relationship. Restraint is difficult because noisy women are challenging to work with and may disturb others.

Which laboratory result would be a cause for concern if exhibited by a patient at her first prenatal visit during the second month of her pregnancy? A. Rubella titer, 1:6 B. Platelets, 300,000/mm3 C. White blood cell count, 6000/mm3 D. Hematocrit 38%, hemoglobin 13 g/dL

ANS: A Rationale: A rubella titer of less than 1:8 indicates a lack of immunity to rubella, a viral infection that has the potential to cause teratogenic effects on fetal development. Arrangements should be made to administer the rubella vaccine after birth during the postpartum period because administration of rubella, a live vaccine, would be contraindicated during pregnancy. Women receiving the vaccine during the postpartum period should be cautioned to avoid pregnancy for 3 months. The lab values for WBCs, platelets, and hematocrit/hemoglobin are within the expected range for pregnant women.

Which complaint made by a patient at 35 weeks of gestation requires additional assessment? A. Abdominal pain B. Ankle edema in the afternoon C. Backache with prolonged standing D. Shortness of breath when climbing stairs

ANS: A Rationale: Abdominal pain at 35 weeks gestation may indicate preeclampsia, or abruptio placentae. Ankle edema in the afternoon is a normal finding at this stage of the pregnancy. Backaches while standing is a normal finding in the later stages of pregnancy. Shortness of breath is an expected finding at 35 weeks.

In teaching a pregnant adolescent about nutrition, what should the nurse include in the care plan? A. Determine the weight gain needed to meet adolescent growth and add 35 lb. B. Suggest that she does not eat at fast food restaurants to avoid foods of poor nutritional value. C.Realize that most adolescents are unwilling to make dietary changes during pregnancy. D. Emphasize the need to eliminate common teen snack foods because they are too high in fat and sodium.

ANS: A Rationale: Adolescents should gain in the upper range of the recommended weight gain. They also need to gain weight that would be expected for their own normal growth. Adolescents are willing to make changes; however, they still need to be like their peers. Eliminating fast foods will make her appear different from her peers. She should be taught to choose foods that add needed nutrients. Changes in the diet should be kept at a minimum and snacks should be included. Snack foods can be included in moderation and other foods added to make up for the lost nutrients.

A 36-year-old divorcee with a successful modeling career finds out that her 18-year-old daughter is expecting her first child. Which is a major factor in determining how this woman will respond to becoming a grandmother? A. Her age B. Her career C. Being divorced D. Age of the daughter

ANS: A Rationale: Age is a major factor in determining the emotional response of prospective grandparents. Young grandparents may not be happy with the stereotype of grandparents as being old. Career responsibilities may have demands that make the grandparents not as accessible but arenot a major factor in determining the woman's response to becoming a grandmother. Being divorced is not a major factor that determines the adaptation of grandparents. The age of the daughter is not a major factor that determines the adaptation of grandparents. The age of the grandparent is a major factor.

A patient postdelivery is concerned about getting back to her prepregnancy weight as soon as possible. She had only gained 15 lb during her pregnancy. Which assessment factor would be of concern at her 6-week postpartum checkup? A. Patient has lost 30 lb during the 6-week period prior to her scheduled checkup. B. Patient states that she is eating healthy and limiting intake of processed foods. C. Patient relates increased consumption of fruits and vegetables in her diet postbirth. D. Patient has resumed her usual exercise pattern of walking around the neighborhood for 10 minutes each night.

ANS: A Rationale: Although a certain amount of weight loss is expected in the postpartum period, the fact that the reported weight loss is double the amount of weight gained during the pregnancy places the patient at risk for malnutrition. Further inquiry is needed. Limiting the intake of processed foods is a healthy dietary alternative to decreasing sodium intake. Increases in fruits and vegetables are a healthy dietary alternative to decrease possible occurrence of hypertension. An exercise program is part of a healthy nutrition approach.

Which aspect of fetal diagnostic testing is most important to expectant parents? A. Safety of the fetus B. Duration of the test C. Cost of the procedure D. Physical discomfort caused by the procedure

ANS: A Rationale: Although all of these are considerations, parents are usually most concerned about the safety of the fetus. Parents are concerned about the duration of the test; however, it is not the greatest concern. The cost of the procedure is important to parents, especially those without third-party payers; but again, this is not the greatest concern. With adequate preparation for the procedure by the nurse physical discomfort can be allayed.

A newly pregnant patient tells the nurse that she has irregular periods and is unsure of whenshe got pregnant. Scheduling an ultrasound is a standing prescription for the patient's health care provider. When is the best time for the nurse to schedule the patient's ultrasound? A. Immediately B. In 2 weeks C. In 4 weeks D. In 6 weeks

ANS: A Rationale: An embryo can be seen about 5 to 6 weeks after the last menstrual period. At this time the crown-rump length (CRL) of the embryo is the most reliable measure of gestational age. Fetal viability is confirmed by observation of fetal heartbeat, which is visible when the CRL of the embryo is 5 mm.

An expectant mother, diagnosed with oligohydramnios, asks the nurse what this condition means for the baby. Which statement should the nurse provide for the patient? A. Oligohydramnios can cause poor fetal lung development. B. Oligohydramnios means that the fetus is excreting excessive urine. C. Oligohydramnios could mean that the fetus has a gastrointestinal blockage. D. Oligohydramnios is associated with fetal central nervous system abnormalities.

ANS: A Rationale: Because an abnormally small amount of amniotic fluid restricts normal lung development, the fetus may have poor fetal lung development. Oligohydramnios may be caused by a decrease in urine secretion. Excessive amniotic fluid production may occur when the gastrointestinal tract prevents normal ingestion of amniotic fluid. Excessive amniotic fluid production may occur when the fetus has a central nervous system abnormality.

Which effect is a common response to both smoking and cocaine use in the pregnant patient? A. Vasoconstriction B. Increased appetite C. Increased metabolism D. Changes in insulin metabolism

ANS: A Rationale: Both smoking and cocaine use cause vasoconstriction, which results in impaired placental blood flow to the fetus. Smoking and cocaine use do not increase appetite, change insulin metabolism, or increase metabolism.

A pregnant patient has lactose intolerance. Which recommendation will the nurse provide to best help the patient meet dietary needs for calcium? A. Add foods such as nuts, dried fruit, and broccoli to the diet. B. Consume dairy products but take an over-the-counter anti-gas product. C. Increase the intake of dark leafy vegetables, such as spinach and chard. D. Use powdered milk instead of liquid forms of milk.

ANS: A Rationale: Calcium is present in legumes, nuts, dried fruits, and broccoli, so these foods can be added to increase calcium intake. Although dark leafy vegetables contain calcium, they also contain oxalates that decrease the availability of calcium. Powdered milk contains lactase, similar to the nondehydrated varieties. Milk products should be avoided by patients with lactose intolerance. Adequate calcium may be obtained from food and supplements. Some patients may be able to tolerate lactose free dairy products.

The clinic nurse is obtaining a health history on a newly pregnant patient. Which is an indication for fetal diagnostic procedures if present in the health history? A. Maternal diabetes B. Weight gain of 25 lb C. Maternal age older than 30 years D. Previous infant weighing more than 3000 g at birth

ANS: A Rationale: Diabetes is a risk factor in pregnancy because of possible impairment of placental perfusion. Excessive weight gain is an indication for testing. Normal weight gain is 25 to 35 lb. A maternal age older than 35 years is an indication for testing. Having had another infant weighing more than 4000 g is an indication for testing.

Which statement best describes the changes that occur during the fetal period of development? a. Maturation of organ systems b. Development of basic organ systems c. Resistance of organs to damage from external agents d. Development of placental oxygen-carbon dioxide exchange

ANS: A Rationale: During the fetal period, the body systems grow in size and mature in function to allow independent existence after birth. Basic organ systems are developed during the embryonic period. The organs are always at risk for damage from external sources; however, the older the fetus, the more resistant will be the organs. The greatest risk is when the organs are developing. The placental system is complete by week 12, but that is not the best description of the fetal period.

What is the purpose of the ovum's zona pellucida? a. Prevents multiple sperm from fertilizing the ovum b. Stimulates the ovum to begin mitotic cell division c. Allows the 46 chromosomes from each gamete to merge d. Makes a pathway for more than one sperm to reach the ovum

ANS: A Rationale: Fertilization causes the zona pellucida to change its chemical composition so that multiple sperm cannot fertilize the ovum. Mitotic cell division begins when the nuclei of the sperm and ovum unite. Each gamete (sperm and ovum) has only 23 chromosomes; there will be 46 chromosomes when they merge. Once sperm has entered the ovum, the zona pellucida changes to prevent other sperm from entering.

Which fetal position may cause the laboring patient increased back discomfort? A. Left occiput anterior B. Left occiput posterior C. Right occiput anterior D. Right occiput transverse

ANS: B RATIONALE: In the left occiput posterior position, each contraction pushes the fetal head against themother's sacrum, which results in intense back discomfort. Back labor is seen mostly when the fetus is in the posterior position.

The labor nurse is reviewing breathing techniques with a primiparous patient admitted for induction of labor. When is the best time to encourage the laboring patient to use slow, deep chest breathing with contractions? A. During labor, when she can no longer talk through contractions B. During the first stage of labor, when the contractions are 3 to 4 minutes apart C. Between contractions, during the transitional phase of the first stage of labor D. Between her efforts to push to facilitate relaxation between contractions

ANS: A Rationale: Focused breathing techniques should not be used in labor until they are actually needed,which is usually when the woman can no longer walk and talk during a contraction. If breathing techniques are used too early, the woman tends to move through the different techniques too quickly, and she may stop using them. In addition, the use of the more complex breathing patterns in latent labor may increase fatigue. Women should be encouraged to adapt the techniques to their own comfort and needs. Breathing deeply between contractions or pushing can increase the possibility of carbon dioxide retention and make the patient dizzy

What point in the pregnancy is the most accurate time to determine gestational age through ultrasound? A. First trimester B. Second trimester C. Third trimester D. No difference in accuracy among the trimesters

ANS: A Rationale: Gestational age determination by ultrasonography is increasingly less accurate after the first trimester. Gestational age determination is best done in the first trimester. There is a difference in trimesters when doing a gestational age ultrasonography.

A pregnant patient asks the nurse how her baby gets oxygen to breathe. What is the nurse'sbest response? a. "Oxygen-rich blood is delivered through the umbilical vein to the baby." b. "Take lots of deep breaths because the baby gets all of its oxygen from you." c. "You don't need to be concerned about your baby getting enough oxygen." d. "The baby's lungs are not mature enough to actually breathe, so don't worry."

ANS: A Rationale: Oxygen-rich blood travels from the mother's circulatory system to the placenta and from the placenta to the umbilical vein (veins carry blood to the heart). From the vein, most of the oxygenated blood travels to the fetal liver or the inferior vena cava. Taking deep breaths can temporarily increase oxygenation but can also lead to increased carbon dioxide retention and dizziness. The patient is asking a normal fetal developmental question often asked by pregnant women. Fetal lungs reach maturity by 37 weeks of gestation, but fetal breathing movements are common. Oxygen transport across lung tissue occurs with the first breath.

Which guidance related to a healthy diet during pregnancy will the nurse provide to a patient in her 1st trimester? A. "Every day you need to have at least 6 ounces of protein from sources such as meat, fish, eggs, beans, nuts, soybean products, and tofu." B. "High-dose vitamin A supplements will promote optimal vision while preventing a common cause of blindness in neonates." C. "Meals such as sushi with a cold deli salad made with raw sprouts combine high-fiber foods with protein sources to meet multiple nutritional needs." D. "Vitamin and mineral supplements can meet your nutrient needs if you have inadequate intake because of nausea or a sensation of fullness."

ANS: A Rationale: Protein sources include meat, poultry, fish, eggs, legumes (e.g., beans, peas, lentils), nuts, and soybean products such as tofu. Pregnant women need 6 to 6.5 oz of protein daily. Vitamin A can cause fetal anomalies of the bones, urinary tract, and central nervous system when taken in high doses. Pregnant women should avoid raw fish and foods such as cold deli salads and raw sprouts. Supplements do not generally contain protein and calories and may lack many necessary nutrients; therefore they cannot serve as food substitutes.

A pregnant patient's biophysical profile score is 8. The patient asks the nurse to explain the results. What is the nurse's most appropriate response? A. "The test results are within normal limits." B. "Immediate birth by cesarean birth is being considered." C. "Further testing will be performed to determine the meaning of this score." D. "An obstetric specialist will evaluate the results of this profile and, within the next week, will inform you of your options regarding birth."

ANS: A Rationale: The normal biophysical score ranges from 8 to 10 points if the amniotic fluid volume is adequate. A normal score allows conservative treatment of high-risk patients. Birth can be delayed if fetal well-being is an issue. Scores less than 4 would be investigated, and birth could be initiated sooner than planned. This score is within normal range, and no further testing is required at this time. The results of the biophysical profile are usually available immediately after the procedure is performed.

The nurse is assessing a newborn immediately after birth. After assigning the first Apgar scoreof 9, the nurse notes two vessels in the umbilical cord. What is the nurse's next action? a. Assess for other abnormalities of the infant. b. Note the assessment finding in the infant's chart. c. Notify the health care provider of the assessment finding. d. Call for the neonatal resuscitation team to attend the infant immediately.

ANS: A Rationale: The normal finding in the umbilical cord is two arteries and one vein. Two vessels may indicate other fetal anomalies. Notation of the finding is the appropriate next step when the finding is expected. The health care provider will need to be notified; however, the infant isthe nurse's primary concern and must be assessed for abnormalities first. The initial Apgar score is 9, indicating no signs of distress or need of resuscitation.

Which physical characteristics decrease as the fetus nears term? (Select all that apply.) A. Vernix caseosa B. Lanugo C. Port wine stain D. Brown fat E. Eyebrows or head hair

ANS: A,B Rationale: Both vernix caseosa and lanugo decrease as the fetus reaches term. Port wine stain is a birthmark and, if present, will be exhibited at or shortly after birth. Brown fat in the fetus will be maintained in order to maintain core temperature. Eyebrows and head hair increase as the fetus nears term.

The nurse is planning care for a patient during the fourth stage of labor. Which interventions should the nurse plan to implement? (Select all that apply.) A. Offer the patient a warm blanket. B. Place an ice pack on the perineum. C. Massage the uterus if it is boggy. D. Delay breastfeeding until the patient is rested. E. Explain to the patient that the lochia will be light pink in color.

ANS: A,B,C RATIONALE: The fourth stage of labor lasts from the birth of the placenta through the first 1 to 4 hours after birth. Many women are chilled after birth. A warm blanket, hot drink, or soup may help relieve the chill and make the woman more comfortable. Localized discomfort from birth trauma such as lacerations, episiotomy, edema, or hematoma is evident as the effects of local and regional anesthetics diminish. Ice packs on the perineum limit this edema and hematoma formation. A soft (boggy) uterus and increasing uterine size are associated with postpartum hemorrhage because large blood vessels at the placenta site are not compressed. The uterus should be massaged if it is not firm. The fourth stage is the best time to initiate breastfeeding if maternal and infant problems are absent. The vaginal drainage after childbirth is calledlochia. The three stages are lochia rubra, lochia serosa, and lochia alba. Lochia rubra, consisting mostly of blood, is present in the fourth stage of labor. The color of the lochia will be bright red not pink.

Which medications could potentially cause hyperstimulation of the uterus during labor? (Select all that apply.) A. Oxytocin (Pitocin) B. Misoprostol (Cytotec) C. Dinoprostone (Cervidil) D. Methylergonovine maleate (Methergine)

ANS: A,B,C,D RATIONALE: Oxytocin, misoprostol, and dinoprostone fall under the general category of uterine stimulants. Cytotec and Cervidil are prostaglandins. Methergine is an ergot alkaloid.

While developing an intrapartum care plan for the patient in early labor, it is important thatthe nurse recognize that psychosocial factors may influence a woman's experience of pain. These include which of the following? (Select all that apply.) A. Culture B. Anxiety and fear C. Support systems D. Preparation for childbirth E. Previous experiences with pain

ANS: A,B,C,D,E RATIONALE: **Culture: A woman's sociocultural roots influence how she perceives, interprets, and responds to pain during childbirth. Some cultures encourage loud and vigorous expressions of pain, whereas others value self-control. The nurse should avoid praising some behaviors (stoicism) while belittling others (noisy expression). **Anxiety and fear: Extreme anxiety and fear magnify sensitivity to pain and impair awoman's ability to tolerate it. Anxiety and fear increase muscle tension in the pelvic area,which counters the expulsive forces of uterine contractions and pushing efforts. **Support systems: An anxious partner is less able to provide help and support to awoman during labor. A woman's family and friends can be an important source of support if they convey realistic and positive information about labor and birth. **Preparation for childbirth: This does not ensure a pain-free labor. Preparation does reduce anxiety and fear. It also allows a woman to rehearse for labor. **Previous experiences with pain: Fear and withdrawal are natural responses to pain during labor.Learning about these normal sensations ahead of time helps a woman suppress her natural reactions of fear regarding the impending birth. If a woman previously had a long and difficult labor, she is likely to be anxious. She may also have learned ways to cope and may use these skills to adapt to the present labor experience.

When evaluating the patient's progress, the nurse knows that four of the five fetal factors thatinteract to regulate the heart rate are (Select all that apply.) A. baroreceptors. B. adrenal glands. C. chemoreceptors. D. uterine activity. E. autonomic nervous system.

ANS: A,B,C,E RATIONALE: The sympathetic and parasympathetic branches of the autonomic nervous system are balanced forces that regulate FHR. Sympathetic stimulation increases the heart rate, whereas parasympathetic responses, through stimulation of the vagus nerve, reduce the FHR, and maintain variability. The baroreceptors stimulate the vagus nerve to slow the FHR and decrease the blood pressure. These are located in the carotid arch and major arteries. The chemoreceptors are cells that respond to changes in oxygen, carbon dioxide, and pH. They are found in the medulla oblongata and aortic and carotid bodies. The adrenal medulla secretes epinephrine and norepinephrine in response to stress, causing accelerations in FHR. Hypertonic uterine activity can reduce the time available for the exchange of oxygen and waste products; however, this is a maternal factor. The fifth fetal factor is the central nervous system. The fetal cerebral cortex causes the heart rate to increase during fetal movement and decrease when the fetus sleeps.

Transvaginal ultrasonography is often performed during the first trimester. A 6-week-gestation patient expresses concerns over the necessity for this test. The nurse should explain that this diagnostic test may be necessary to determine which of the following? (Select all that apply.) A. Multifetal gestation B. Bicornuate uterus C. Presence and location of pregnancy D. Amniotic fluid volume E. Presence of ovarian cysts

ANS: A,B,C,E Rationale: A bicornuate uterus, multifetal gestation, presence of ovarian cysts, and presence and location of pregnancy can be determined by transvaginal ultrasound in the first trimester of pregnancy. This procedure is also used for estimating gestational age, confirming fetal viability, identifying fetal abnormalities or chromosomal defects, and identifying the maternal abnormalities mentioned, as well as fibroids. Amniotic fluid volume is assessed during the second and third trimesters. Conventional ultrasound would be used.

For which patient would an L/S ratio of 2:1 potentially be considered abnormal? A. A 38-year-old gravida 2, para 1, who is 38 weeks' gestation B. A 24-year-old gravida 1, para 0, who has diabetes C. A 44-year-old gravida 6, para 5, who is at term D. An 18-year-old gravida 1, para 0, who is in early labor at term

ANS: B Rationale: Even though an L/S ratio of 2:1 is typically considered to be a normal finding to validate fetallung maturity prior to 38 weeks' gestation, the result may not be accurate in determining fetal lung maturity if a patient is diabetic.

Which clinical finding should the nurse expect to assess in the third stage of labor that indicates the placenta has separated from the uterine wall? (Select all that apply.) A. A gush of blood appears. B. The uterus rises upward in the abdomen. C. The fundus descends below the umbilicus. D. The cord descends further from the vagina. E. The uterus becomes boggy and soft, with an elongated shape.

ANS: A,B,D RATIONALE: Four signs suggest placenta separation. The uterus has a spherical shape. The uterus rises upward in the abdomen as the placenta descends into the vagina and pushes the fundus upward. The cord descends further from the vagina. A gush of blood appears as blood trapped behind the placenta is released. The fundus rises upward above the umbilicus. A boggy uterus with an elongated shape would not be expected.

The nurse is preparing a patient for a cesarean birth scheduled to be performed under general anesthesia. Which should the nurse plan to administer, if ordered by the health care provider, to prevent aspiration of gastric contents? (Select all that apply.) A. Citric acid (Bicitra) B. Ranitidine (Zantac) C. Hydroxyzine (Vistaril) D. Glycopyrrolate (Robinul) E. Promethazine (Phenergan)

ANS: A,B,D RATIONALE: To prevent aspiration of gastric contents during general anesthesia administration of medications to raise the gastric pH and make secretions less acidic, such as citric acid (Bicitra) and ranitidine (Zantac) may be prescribed. In addition, medications to reduce secretions, such as glycopyrrolate (Robinul) may be prescribed. Hydroxyzine (Vistaril) and promethazine (Phenergan) are used to prevent and relieve nausea often associated with opioids.

The nurse is teaching a pregnant patient about signs of possible pregnancy complications. Which should the nurse include in the teaching plan? (Select all that apply.) A. Report watery vaginal discharge. B. Report puffiness of the face or around the eyes. C. Report any bloody show when you go into labor. D. Report visual disturbances, such as spots before the eyes. E. Report any dependent edema that occurs at the end of the day.

ANS: A,B,D Rationale: Watery vaginal discharge could mean that the membranes have ruptured. Puffiness of the face or around the eyes and visual disturbances may indicate preeclampsia or eclampsia. These three signs should be reported. Bloody show as labor starts may mean the mucus plug has been expelled. One of the earliest signs of labor may be bloody show, which consists of the mucus plug and a small amount of blood. This is a normal occurrence. Up to 70% of women have dependent edema during pregnancy. This is not a sign of a pregnancy complication.

The nurse is teaching a breastfeeding patient about substances to avoid while she is breastfeeding. Which substances should the nurse include in the teaching session? (Select all that apply.) A. Caffeine B. Alcohol C. Omega-6 fatty acids D. Appetite suppressants E. Polyunsaturated omega-3 fatty acids

ANS: A,B,D Foods high in caffeine should be limited. Infants of mothers who drink more than two or three cups of caffeinated coffee or the equivalent each day may be irritable or have trouble sleeping. Although the relaxing effect of alcohol was once thought to be helpful to the nursing mother, the deleterious effects of alcohol are too important to consider this suggestion appropriate today. An occasional single glass of an alcoholic beverage may not be harmful, but larger amounts may interfere with the milk-ejection reflex and may be harmful to the infant. Nursing mothers should avoid appetite suppressants, which may pass into the milk and harm the infant. The long-chain polyunsaturated omega-3 and omega-6 fatty acids are present in human milk. Therefore they should be included in the mother's diet during lactation.

When administering a narcotic to a laboring patient, which statement explains why the nurse should inject the medication at the beginning of a contraction? A. The medication will be rapidly circulated. B. Less medication will be transferred to the fetus. C. The maternal vital signs will not be adversely affected. D. Full benefit of the medication is received during that contraction.

ANS: B RATIONALE: Injecting the medication at the beginning of a contraction, when blood flow to the placenta is normally reduced, limits transfer to the fetus. It will not increase the circulation of the medication. It will not alter the vital signs any more than giving it at another time. The full benefit of the medication will be received by the patient.

The nurse who elects to practice in the area of obstetrics often hears discussion regarding the four Ps. What are the four Ps that interact during childbirth? (Select all that apply.) a. Powers b. Passage c. Position d. Passenger e. Psyche

ANS: A,B,D,E RATIONALE: **Powers: The two powers of labor are uterine contractions and pushing efforts. During the first stage of labor, through full cervical dilation, uterine contractions are the primary force moving the fetus through the maternal pelvis. At some point after full dilation, the woman adds her voluntary pushing efforts to propel the fetus through the pelvis. **Passage: The passage for birth of the fetus consists of the maternal pelvis and its soft tissues. The bony pelvis is more important to the successful outcome of labor because bones and joints do not yield as readily to the forces of labor. **Passenger: This is the fetus plus the membranes and placenta. Fetal lie, attitude, presentation, and position are all factors that affect the fetus as passenger. **Psyche: The psyche is a crucial part of childbirth. Marked anxiety, fear, or fatigue decreases the woman's ability to cope.Position is not one of the four Ps.

The nurse is assessing a patient in her 37th week of pregnancy for the psychological responses commonly experienced as birth nears. Which psychological responses should the nurse expect to evaluate? (Select all that apply.) A. The patient is excited to see her baby. B. The patient has not started to prepare the nursery for the new baby. C. The patient expresses concern about how to know if labor has started. D. The patient and her spouse are concerned about getting to the birth center in time. E. The patient and her spouse have not discussed how they will share household tasks.

ANS: A,C,D Rationale: As birth nears, the expectant patient will express a desire to see the baby. Most pregnant patients are concerned with their ability to determine when they are in labor. Many couples are anxious about getting to the birth facility in time for the birth. As birth nears, a nesting behavior occurs, which means getting the nursery ready. Not preparing the nursery at this stage is not a response that the nurse should expect to assess. Negotiation of tasks is done during this stage. Discussion regarding the division of household chores is not a response that the nurse should expect to assess at this stage.

A pregnant patient reports that she works in a long-term care setting and is concerned about the impending flu season. She asks about receiving the flu vaccine. As the nurse, you are aware that some immunizations are safe to administer during pregnancy, whereas others are not. Which vaccines could this patient receive? (Select all that apply.) A. Tetanus B. Varicella C. Influenza D. Hepatitis A and B E. Measles, mumps, rubella (MMR)

ANS: A,C,D Rationale: Inactivated vaccines such as those for tetanus, hepatitis A, hepatitis B, and influenza are safe to administer to women who have a risk for contracting or developing the disease. Immunizations with live virus vaccines such as MMR, varicella (chickenpox), or smallpox are contraindicated during pregnancy because of the possible teratogenic effects on the fetus.

A patient asks the nurse how she can tell if labor is real. Which information should the nurse provide to this patient? (Select all that apply.) A. In true labor, the cervix begins to dilate. B. In true labor, the contractions are felt in the abdomen and groin. C. In true labor, contractions often resemble menstrual cramps during early labor. D. In true labor, contractions are inconsistent in frequency, duration, and intensity in the early stages. E. In true labor your contractions tend to increase in frequency, duration, and intensity with walking.

ANS: A,C,E RATIONALE: In true labor, the cervix begins to dilate, contractions often resemble menstrual cramps in the early stage, and labor contractions increase in frequency, duration, and intensity with walking. False labor contractions are felt in the abdomen and groin and the contractions are inconsistent in frequency, duration, and intensity.

The prenatal nurse educator is teaching couples the technique of applying sacral pressure during labor. Which should be included in the teaching session? (Select all that apply.) A. The technique can be combined with heat to the area. B. A jiggling motion should be used while applying the pressure. C. Tennis balls may be used to apply the pressure to the sacral area. D. The pressure against the sacrum should be intermittent during the contraction. E. The hand may be moved slowly or remain positioned directly over the sacrum.

ANS: A,C,E Rationale: Sacral pressure can be combined with thermal stimulation to increase effectiveness.The hand may be moved slowly over the area or remain positioned directly over the sacrum, but pressure should be continuous and firm throughout the contraction. Care should be taken not to jiggle the woman, which may be irritating.

The nurse is planning care for a patient in her first trimester of pregnancy. The patient is experiencing nausea and vomiting. Which interventions should the nurse plan to share with this patient? (Select all that apply.) A. Suck on hard candy. B. Take prenatal vitamins in the morning. C. Try some herbal tea to relieve the nausea. D. Drink fluids frequently but separate from meals. E. Eat crackers or dry cereal before arising in the morning.

ANS: A,D,E Rationale: A patient experiencing nausea and vomiting should be taught to suck on hard candy, drink fluids frequently but separately from meals, and eat crackers, dry toast, or dry cereal before arising in the morning. Prenatal vitamins should be taken at bedtime because they may increase nausea if taken in the morning. Before taking herbal tea, the patient should check with her health care provider.

Which factors contribute to the presence of edema in the pregnant patient? (Select all that apply.) A. Diet consisting of processed foods B. Hemoconcentration C. Increase in colloid osmotic pressure D. Last trimester of pregnancy E. Decreased venous return

ANS: A,D,E Rationale: Processed foods, which are high in sodium content, can contribute to edema formation. As the pregnancy progresses, because of the weight of the uterus, compression takes place, leading to decreased venous return and an increase in edema formation. A decrease in colloid osmotic pressure would contribute to edema formation and fluid shifting. Hemodilution would also lead to edema formation.

A nurse is conducting prenatal education classes for a group of expectant parents. Which information should the nurse include in her discussion of the purpose of amniotic fluid? (Select all that apply.) A. Cushions the fetus B. Protects the skin of the fetus C. Provides nourishment for the fetus D. Allows for buoyancy for fetal movement E. Maintains a stable temperature for the fetus

ANS: A,D,E Rationale: The amniotic fluid provides cushioning for the fetus against impacts to the maternal abdomen. It provides a stable temperature and allows room and buoyancy for fetal movement. Vernix caseosa, the cheeselike coating on the fetus, provides skin protection. The placenta provides nourishment for the fetus.

The examiner indicates to the labor nurse that the fetus is in the left occiput anterior (LOA) position. To facilitate the labor process, how will the nurse position the laboring patient? A. On her back B. On her left side C. On her right side D. On her hands and knees

ANS: B RATIONALE: LOA is the desired fetal position for the birthing process. Positioning the patient on her left side will accomplish two objectives: (1) by the use of gravity, the fetus will most likely stay in the LOA position; and (2) increase perfusion of the placenta and increase oxygen to the fetus. Positioning the patient on her back decreases placental perfusion. Positioning on her right may facilitate internal rotation and move the fetus out of the LOA position. The hands and knees position is reserved to decrease cord compression, facilitate the fetus out of a posterior position, or increase oxygenation in the presence of hypoxia. Because none of these conditions are present, there is no need to implement this position.

A 28-year-old gravida 1, para 0 patient who is at term calls the labor and birth unit stating that she thinks she is in labor. She states that she does have some vaginal discharge and feels wet; however, it is not bloody in nature. She relates a contraction pattern that is irregular, ranging from 5 to 7 minutes and lasting 30 seconds. Which questions should the nurse pose to the patient during this telephone triage? (Select all that apply.) A. Does she think that her membranes have ruptured? B. Is there any evidence of bloody show? C. Instruct the patient to keep monitoring her contraction pattern and call you back if they become more regular. D. When is her next scheduled visit with her health care provider? E. Tell her to come into the hospital for evaluation.

ANS: A,E RATIONALE: The cornerstone of obstetric triage is reassurance of maternal-fetal well-being. Thus in view of the assessment data that the patient provided, the nurse should ascertain membrane status and ask the patient to come in for evaluation. The patient has already indicated that the vaginal discharge was not bloody in nature. Having the patient continue to monitor at home would not provide assurance of maternal-fetal well-being. Asking the patient about the next scheduled physician visit does not address current health concerns of impending labor.

On admission to the labor and birth unit, a 38-year-old female, gravida 4, para 3, at term in early labor is found to have a transverse lie on vaginal examination. What is the priority intervention at this time? A. Perform a vaginal exam to denote progress. B. Notify the health care provider. C. Initiate parenteral therapy. D. Apply oxygen via nasal cannula at 8 L/minute.

ANS: B RATIONALE: A transverse lie is considered to be an abnormal presentation so the physician should be notified and the process of a Caesarean section as the birth method should be initiated. The information provided relative to transverse lie was found on vaginal exam. At this point, the priority is to prepare for a surgical birth because assessment data also indicate that the patient is in early labor; thus a vaginal birth is not imminent. Although initiating parenteral therapy will be required, it is not the priority at this time. Application of oxygen is not required because there is no evidence of fetal or maternal distress.

The primiparous patient at 39 weeks' gestation states to the nurse, "I can breathe easier now." What is the nurse's most appropriate response? A. "Your labor will start any day now since the baby has dropped." B. "That process is called lightening. Do you have to urinate more frequently?" C. "Contact your health care provider when your contractions are every 5 minutes for1 hour." D. "You will likely not feel you baby's movements as much now, so do not be concerned."

ANS: B RATIONALE: As the fetus descends toward the pelvic inlet (dropping), the woman notices that she breathes more easily because upward pressure on her diaphragm is reduced. However, increased pressure on her bladder causes her to urinate more frequently. Pressure of the fetal head in the pelvis also may cause leg cramps and edema. Lightening (descent of the fetus toward the pelvic inlet before labor) is most noticeable in primiparas and occurs about 2 to 3 weeks before the natural onset of labor. Instructions for labor, although correct, do not address thepatient's statement of being able to breathe easier. Fetal movement continues throughout the final weeks of gestation. A decrease in fetal movement is a concerning sign and the health care provider must be notified.

A patient in labor is approaching the transition stage and already has an epidural in place. An additional dose of medication has been prescribed and administered to the patient. Which priority intervention should be performed in order to evaluate the clinical response to treatment? A. Obtain a pain scale response from the patient based on a 0 to 10 scale. B. Document maternal blood pressure and fetal heart rates following medication administration and observe for any variations. C. Document intake and output on the electronic health record (EHR). D. Increase the flow rate of prescribed parenteral fluid to maintain hydration.

ANS: B RATIONALE: Association of Women's Health, Obstetric and Neonatal Nurses (AWHONN) evidence-based practice guidelines note that maternal blood pressure and fetal heart tones should be assessed following any bolus of additional medication via the epidural route. Obtaining a pain scale response is not typically used for the laboring patient but used for postoperative and/or chronic pain patients. Intake and output should be documented as part of the clinical recordbut is not the priority intervention based on this patient's situation. Increasing the flow rate of parenteral fluids requires a physician's order, and there is no clinical evidence that this isneeded. Giving parenteral fluids in excess can lead to fluid retention and fluid volume excess.

A labor patient, gravida 2, para 1, at term has received meperidine (Demerol) for pain control during labor. Her most recent dose was 15 minutes ago and birth is now imminent. Maternal vital signs have been stable and the EFM tracing has not shown any baseline changes. Which medication does the nurse anticipate would be required in the birth room for administration? A. Oxytocin (Pitocin) B. Naloxone (Narcan) C. Bromocriptine (Parlodel) D. Oxygen

ANS: B RATIONALE: Because birth is imminent, and considering that the patient has had a recent dose of narcotics, the nurse anticipates that naloxone (Narcan) will be administered to the newborn to combat the effects of the opioid. Although Pitocin will be given following birth of the placenta, the newborn will be delivered prior to that and will receive priority intervention. Parlodel is not typically given in the labor and birth area any more. It was previously used to suppress lactation. At present, there is no need for the administration of oxygen because there is no evidence that the mother is showing any signs of respiratory depression.

Childbirth pain is different from other types of pain in that it is A. less intense. B. associated with a physiologic process. C. more responsive to pharmacologic management. D. designed to make one withdraw from the stimulus.

ANS: B RATIONALE: Childbirth pain is part of a normal process, whereas other types of pain usually signify an injury or illness. Childbirth pain is not less intense than other types of pain. Pain management during labor may affect the course and length of labor. The pain with childbirth is a normal process; however, it is not caused by the type of injury as when withdrawal from the stimulus occurs.

A laboring patient has asked the nurse to assist her in utilizing a cutaneous stimulation strategy for pain management. The nurse would A. assist her into the shower. B. apply a heat pack to lower back. C. help her to create a relaxing mental scene. D. encourage cleansing breaths and slow-paced breathing.

ANS: B RATIONALE: Cutaneous stimulation includes self-massage, massage by others, counterpressure, touch, thermal stimulation, and acupressure. A shower, tub, and whirlpool are forms of hydrotherapy; creating a relaxed mental scene is mental stimulation. The use of cleansing breaths and patterned breathing is part of breathing techniques for labor.

Which patient will most likely have increased anxiety and tension during labor? A. Gravida 2 who refused any medication B. Gravida 2 who delivered a stillborn baby last year C. Gravida 1 who did not attend prepared childbirth classes D. Gravida 3 who has two children younger than 3 years

ANS: B RATIONALE: If a previous pregnancy had a poor outcome, the patient will probably be more anxious during labor and birth. The patient without childbirth education classes is not prepared for labor and will have increased anxiety during labor; however, the patient with a poor previous outcome is more likely to experience a greater degree of anxiety. A gravida 2 has previous experience and can anticipate what to expect. By refusing any medication, she is taking control over her situation and will have less anxiety. This gravida 3 has previous experience and is aware of what to expect.

Which physiologic event is the key indicator of the commencement of true labor? A. Bloody show B. Cervical dilation and effacement C. Fetal descent into the pelvic inlet D. Uterine contractions every 7 minutes

ANS: B RATIONALE: The conclusive distinction between true and false labor is that contractions of true labor cause progressive change in the cervix. Bloody show can occur before true labor. Fetal descent can occur before true labor. False labor may have contractions that occur this frequently but is usually inconsistent.

Which nursing action is correct when initiating electronic fetal monitoring? A. Lubricate the tocotransducer with an ultrasound gel. B. Securely apply the tocotransducer with a strap or belt. C. Inform the patient that she should remain in the semi-Fowler position D. Determine the position of the fetus before attaching the electrode to the maternal abdomen.

ANS: B RATIONALE: The tocotransducer should fit snugly on the abdomen to monitor uterine activity accurately. The tocotransducer does not need gel to operate appropriately. The patient should be encouraged to move around during labor. The tocotransducer should be placed at the fundal area of the uterus.

The health care provider for a laboring patient makes the following entry into the patient'srecord: 3/50%/+1. What instruction will the nurse implement with the patient? A. "You will need to remain in bed attached to the electronic fetal monitor." B. "Breathe with me slowly, in through your nose and out through your mouth." C. "I will begin the administration of 1000 mL of IV fluid so you can have an epidural." D. "Your partner will need to change into scrub attire to attend the imminent birth."

ANS: B RATIONALE: This patient is in the latent phase of the first stage of labor. Use slow, deep chest breathing patterns early in labor to conserve energy for the upcoming process. There is no mention in the stem that the membranes are ruptured, which may prohibit the patient from ambulating. Ambulating during early labor uses gravity to facilitate fetal descent. This is desired because the head is at 1 station. Epidural placement during early labor may slow down the labor process and should be delayed. There is no indication that birth is imminent because the patient is only 3 cm dilated.

What is the most likely cause for this fetal heart rate pattern? A. Administration of an epidural for pain relief during labor B. Cord compression C. Breech position of fetus D. Administration of meperidine (Demerol) for pain relief during labor

ANS: B RATIONALE: Variable deceleration patterns are seen in response to head compression or cord compression. A breech presentation would not be likely to cause this fetal heart rate pattern. Similarly, administration of medication and/or an epidural would not cause this fetal heart rate pattern.

A laboring patient states to the nurse, "I have to push!" What is the next nursing action? A. Contact the health care provider. B. Examine the patient's cervix for dilation. C. Review with her how to bear down with contractions. D. Ask her partner to support her head with each push.

ANS: B RATIONALE: When the cervix is completely dilated, the head can descend through the pelvis and stimulate the pushing, reflex. Cervical dilation must first be confirmed because premature pushing efforts may result in cervical edema and corresponding delay in dilation. Once complete dilation has been confirmed, the nurse can notify the health care provider. Teaching positioning and pushing efforts is accomplished once complete dilation has been confirmed.

When the mother's membranes rupture during active labor, the fetal heart rate should beobserved for the occurrence of which periodic pattern? A. Early decelerations B. Variable decelerations C. Nonperiodic accelerations D. Increase in baseline variability

ANS: B RATIONALE: When the membranes rupture, amniotic fluid may carry the umbilical cord to a position where it will be compressed between the maternal pelvis and the fetal presenting part, resulting in a variable deceleration pattern. Early declarations are considered reassuring; they are not a concern after rupture of membranes. Accelerations are considered reassuring; they are not a concern after rupture of membranes. Increase in baseline variability is not an expected occurrence after the rupture of membranes.

The results of a contraction stress test (CST) are positive. Which intervention is necessary based on this test result? A. Repeat the test in 1 week so that results can be trended based on this baseline result. B. Contact the health care provider to discuss birth options for the patient. C. Send the patient out for a meal and repeat the test to confirm that the results are valid. D. Ask the patient to perform a fetal kick count assessment for the next 30 minutes and then reassess the patient.

ANS: B Rationale: A positive CST test is an abnormal finding, and the provider should be notified so that birth options can be initiated. A positive CST indicates possible fetal compromise. Intervention should not be delayed by 1 week and results do not have to be trended. Because this is an abnormal result, there is no need to repeat the test. Sending the patient out for a meal will delay treatment options and may interfere with possible birth interventions if anesthesia is needed. Fetal kick count assessment is not needed at this time and will further delay treatment interventions for this abnormal result, which indicates fetal compromise.

What does optimal nursing care after an amniocentesis include? A. Pushing fluids by mouth B. Monitoring uterine activity C. Placing the patient in a supine position for 2 hours D. Applying a pressure dressing to the puncture site

ANS: B Rationale: A risk with amniocentesis is the onset of spontaneous contractions. Hydration is important; however, the woman has not been NPO, so this should not be a problem. The supine position may decrease uterine blood flow; the side-lying position is preferred. Pressure dressings are not necessary.

A pregnant patient has received the results of her triple-screen testing and it is positive. She provides you with a copy of the test results that she obtained from the lab. What would thenurse anticipate as being implemented in the patient's plan of care? A. No further testing is indicated at this time because results are normal. B. Refer to the physician for additional testing. C. Validate the results with the lab facility. D. Repeat the test in 2 weeks and have the patient return for her regularly scheduled prenatal visit.

ANS: B Rationale: Additional genetic testing is indicated to provide the patient with treatment options. A positive result on a triple-screen test is considered to be an abnormal finding so the patient should be referred to the physician for additional genetic testing. Validation of the test with a lab facility is not necessary because the patient provided you with a copy of the test results. There is no need to repeat the clinical test because the findings have already been determined.

A pregnant woman of normal weight enters her 13th week of pregnancy. If the patient eats and exercises as directed, what will the nurse anticipate as the ongoing weight gain for the remaining trimesters? A. 0.3 lb every week B. 1 lb every week C. 1.8 lb every week D. 2 lb every week

ANS: B Rationale: After the first 12 weeks (first trimester), the pregnant woman should gain 0.35 to 0.5 kg (0.8 to 1 lb) per week for the remainder of the pregnancy.

You are performing assessments for an obstetric patient who is 5 months pregnant with her third child. Which finding would cause you to suspect that the patient was at risk? A. Patient states that she doesn't feel any Braxton Hicks contractions like she had in her prior pregnancies. B. Fundal height is below the umbilicus. C. Cervical changes, such as Goodell's sign and Chadwick's sign, are present. D. She has increased vaginal secretions.

ANS: B Rationale: Based on gestational age (20 weeks), the fundal height should be at the umbilicus. This finding is abnormal and warrants further investigation about potential risk. With subsequent pregnancies, multiparas may not perceive Braxton Hicks contractions as being evidentcompared with their initial pregnancy. Cervical changes such as Goodell's and Chadwick'ssigns should be present and are considered a normal finding. Increased vaginal secretions are normal during pregnancy as a result of increased vascularity.

The nurse is reviewing the procedure for alpha-fetoprotein (AFP) screening with a patient at16 weeks' gestation. The nurse determines that the patient understands the teaching when shestates that will be collected for the initial screening process? A. Urine B. Blood C. Saliva D. Amniotic fluid

ANS: B Rationale: Initial screening is completed with blood. AFP can be detected in amniotic fluid; however, that procedure is more costly and invasive. Procedures progress from least invasive to most invasive.

Which physiologic finding is consistent with normal pregnancy? A. Systemic vascular resistance increases as blood pressure decreases. B. Cardiac output increases during pregnancy. C. Blood pressure remains consistent independent of position changes. D. Maternal vasoconstriction occurs in response to increased metabolism.

ANS: B Rationale: Cardiac output increases during pregnancy as a result of increased stroke volume and heart rate. Systemic vascular resistance decreases while blood pressure remains the same. Maternal blood pressure changes in response to patient positioning. In response to increased metabolism, maternal vasodilation is seen during pregnancy.

An expectant patient in her third trimester reports that she developed a strong tie to her babyfrom the beginning and now is really in tune to her baby's temperament. The nurse interpretsthis as the development of which maternal task of pregnancy? A. Learning to give of herself B. Developing attachment with the baby C. Securing acceptance of the baby by others D. Seeking safe passage for herself and her baby

ANS: B Rationale: Developing a strong tie in the first trimester and progressing to be in tune is the process of commitment, attachment, and interconnection with the infant. This stage begins in the first trimester and continues throughout the neonatal period. Learning to give of herself is the task that occurs during pregnancy as the woman allows her body to give space to the fetus. She continues with giving to others in the form of food and presents. Securing acceptance of the baby is a process that continues throughout pregnancy as the woman reworks relationships. Seeking safe passage is the task that ends with birth. During this task, the woman seeks health care and carries out cultural practices.

Which type of cutaneous stimulation involves massage of the abdomen? A. Imagery B. Effleurage C. Mental stimulation D. Thermal stimulation

ANS: B Rationale: Effleurage is massage usually performed on the abdomen during contractions. Imagery exercises enhance relaxation by teaching the woman to imagine herself in a relaxing setting. Mental stimulation is a group of methods to decrease pain by increasing mental stimulation. Thermal stimulation decreases pain by using applications of heat and cold.

A patient who is 7 months pregnant states, "I'm worried that something will happen to mybaby." Which is the nurse's best response? A. "Your baby is doing fine." B. "Tell me about your concerns." C. "There is nothing to worry about." D. "The doctor is taking good care of you and your baby."

ANS: B Rationale: Encouraging the patient to discuss her feelings is the best approach. The nurse should notdisregard or belittle the patient's feelings. Responding that your baby is doing fine disregards the patient's feelings and treats them as unimportant. Responding that there is nothing toworry about does not answer the patient's concerns. Saying that the doctor is taking good care of you and your baby is belittling the patient's concerns.

Which mechanism of labor occurs when the largest diameter of the fetal presenting part passes the pelvic inlet? A. Extension B. Engagement C. Internal rotation D. External rotation

ANS: B Rationale: Engagement occurs when the presenting part fully enters the pelvic inlet. Extension occurs when the fetal head meets resistance from the tissues of the pelvic floor and the fetal neck stops under the symphysis. This causes the fetal head to extend. Internal rotation occurs when the fetus enters the pelvic inlet. The rotation allows the longest fetal head diameter to conform to the longest diameter of the maternal pelvis. External rotation occurs after the birth of the head. The head then turns to the side so the shoulders can internally rotate and are positioned with their transverse diameter in the anteroposterior diameter of the pelvic outlet.

Which comment made by a new mother to her own mother is most likely to encourage thegrandmother's participation in the infant's care? A. "Could you help me with the housework today?" B. "The baby is spitting up a lot. What should I do?" C. "I know you are busy, so I'll get John's mother to help me." D. "The baby has a stomachache. I'll call the nurse to find out what to do."

ANS: B Rationale: Looking to the grandmother for advice encourages her to become involved in the care of theinfant. Housework does not encourage the grandmother to participate in the infant's care. Getting John's mother to help and calling the nurse about advice excludes the grandmother.

Which analysis of maternal serum is the best predictor of chromosomal abnormalities in the fetus? A. Biophysical profile B. Multiple-marker screening C. Lecithin-to-sphingomyelin ratio D. Blood type and crossmatch of maternal and fetal serum

ANS: B Rationale: Maternal serum can be analyzed for abnormal levels of alpha-fetoprotein (AFP), human chorionic gonadotropin (hCG), inhibin A, and estriol. The multiple-marker screening may predict chromosomal defects in the fetus. The biophysical profile is used to evaluate fetal status during the antepartum period. Five variables are used; however, none are concerned with chromosomal problems. The lecithin-to-sphingomyelin ratio is used to determine fetal lung maturity. The blood type and crossmatch will not predict chromosomal defects in the fetus.

A patient in her first trimester complains of nausea and vomiting. The patient asks, "Why is this happening?" What is the nurse's best response? A. "It is due to an increase in gastric motility." B. "It may be due to changes in hormones." C. "It is related to an increase in glucose levels." D. "It is caused by a decrease in gastric secretions."

ANS: B Rationale: Nausea and vomiting are believed to be caused by increased levels of hormones, decreased gastric motility, and hypoglycemia. Gastric motility decreases during pregnancy. Glucose levels decrease in the first trimester. Gastric secretions decrease, but this is not the main cause of nausea and vomiting.

Use Nägele's rule to determine the EDD (estimated day of birth) for a patient whose last menstrual period started on April 12. A. February 19 B. January 19 C. January 21 D. February 7

ANS: B Rationale: Nägele's rule subtracts 3 months from the month of the last menstrual period (month 4 - 3 = January) and adds 7 days to the day that the last menstrual period started (April 12 + 7 days = April 19), so the correct answer is January 19 of the following calendar year.

Which complication could occur as a result of percutaneous umbilical blood sampling (PUBS)? A. Postdates pregnancy B. Fetal bradycardia C. Placenta previa D. Uterine rupture

ANS: B Rationale: PUBS is an invasive test whereby a needle is inserted into the umbilical cord to obtain blood as the basis for diagnostic testing with the guidance of ultrasound technology. The most common complication is fetal bradycardia, which is temporary. PUBS has no effect on extending the gestation of pregnancy, the development of placenta previa, or uterine rupture.

Changes in the diet of the pregnant patient who has phenylketonuria would include A. adding foods high in vitamin C. B. eliminating drinks containing aspartame. C. restricting protein intake to <20 g a day. D. increasing caloric intake to at least 1800 cal/day.

ANS: B Rationale: Use of aspartame by women with phenylketonuria can result in fetal brain damage because these women lack the enzyme to metabolize aspartame. Adding vitamin C, restricting protein, and increasing caloric intake are not necessary for the pregnant patient with phenylketonuria.

A pregnant patient arrives for her first prenatal visit at the clinic. She informs the nurse that she has been taking an additional 400 mcg of folic acid prior to becoming pregnant. Based onthe patient's history, she has reached 8 weeks' gestation. Which recommendation would the nurse provide regarding folic acid supplementation? A. Have the patient continue to take 400 mcg folic acid throughout her pregnancy. B. Tell the patient that she no longer has to take additional folic acid because it will be included in her prenatal vitamins. C. Have the patient increase her folic acid intake to 1000 mcg throughout the rest of her pregnancy. D. Schedule the patient to go for an AFP (alpha-fetoprotein) test.

ANS: B Rationale: Prenatal vitamins include adequate folic acid supplementation, so patients should not take additional supplementation as long as they continue their prenatal vitamins. During pregnancy, the recommendation is to increase the folic acid intake to 600 mcg. 1000 mcg offolic acid would be an excessive dose. The AFP test should be done at 15 to 18 weeks' gestation. This is not clinically indicated because the patient is at 8 weeks' gestation.

An expectant mother says to the nurse, "When my sister's baby was born, it was covered in a cheese-like coating. What is the purpose of this coating?" The correct response by the nurse is to explain that the purpose of vernix caseosa is to a. regulate fetal temperature. b. protect the fetal skin from amniotic fluid. c. promote normal peripheral nervous system development. d. allow the transport of oxygen and nutrients across the amnion.

ANS: B Rationale: Prolonged exposure to amniotic fluid during the fetal period could result in breakdown of the skin without the protection of the vernix caseosa. The amniotic fluid aids in maintaining fetal temperature. Normal peripheral nervous system development is dependent on the nutritional intake of the mother. The amnion is the inner membrane that surrounds the fetus. It is not involved in the oxygen and nutrient exchange.

A patient who smokes one pack of cigarettes daily has a positive pregnancy test. The nurse will explain that smoking during pregnancy increases the risk of which condition? A. Congenital anomalies B. Death before or after birth C. Neonatal hypoglycemia D. Neonatal withdrawal syndrome

ANS: B Rationale: Smoking during pregnancy increases the risk for spontaneous abortion, low birth weight, abruptio placentae, placenta previa, preterm birth, perinatal mortality, and SIDS. Smoking does not appear to cause congenital anomalies, hypoglycemia, or withdrawal syndrome.

A biophysical profile is performed on a pregnant patient. The following assessments are noted: nonreactive stress test (NST), three episodes of fetal breathing movements (FBMs), limited gross movements, opening and closing of hang indicating the presence of fetal tone, and adequate amniotic fluid index (AFI) meeting criteria. Which answer would be the correct interpretation of this test result? A. A score of 10 would indicate that the results are equivocal. B. A score of 8 would indicate normal results. C. A score of 6 would indicate that birth should be considered as a possible treatment option. D. A score of 9 would indicate reassurance.

ANS: B Rationale: The biophysical profile is used to assess fetal well-being. Five categories of assessment are used in this combination test: fetal monitoring NST, evaluation of FBMs, gross movements, fetal tone, and calculation of the amniotic fluid index (AFI). A maximum of 2 points is used if criteria are met successfully in each category; thus a score in the range of 8 to 10 indicates a normal or reassuring finding. A score of 6 provides equivocal results and further testing or observation is necessary. A score of 4 or less requires immediate intervention, and birth may be warranted. The provided assessments indicate a score of 8 as the only area that has not met the stated criteria in the NST.

When planning a healthy diet with a pregnant patient, what should the nurse's first action be? A. Teach the patient about MyPlate. B. Review the patient's current dietary intake. C. Instruct the patient to limit the intake of fatty foods. D. Caution the patient to avoid large doses of vitamins, especially those that are fat-soluble.

ANS: B Rationale: The first action should be to assess the patient's current dietary pattern and practices becauseinstruction should be geared to what she already knows and does. Teaching the food guide MyPlate is important but not the first action when planning a diet with a pregnant patient.Limiting intake of fatty foods is important in a pregnant patient's diet; however, not the firstaction. Caution regarding about excessive fat-soluble vitamins is important; however, not the first action.

An expected change during pregnancy is a darkly pigmented vertical midabdominal line. The nurse recognizes this alteration as A. epulis. B. linea nigra. C. melasma. D. striae gravidarum.

ANS: B Rationale: The linea nigra is a dark pigmented line from the fundus to the symphysis pubis. Epulis refers to gingival hypertrophy. Melasma is a different kind of dark pigmentation that occurs on the face. Striae gravidarum (stretch marks) are lines caused by lineal tears that occur in connective tissue during periods of rapid growth.

The nurse's role in diagnostic testing is to provide which of the following? A. Advice to the couple B. Information about the tests C. Reassurance about fetal safety D. Assistance with decision making

ANS: B Rationale: The nurse should provide the couple with all necessary information regarding a procedure sothat the couple can make an informed decision. The nurse's role is to inform, not to advice.Ensuring fetal safety is not possible with all the diagnostic tests. To offer this is to give false reassurance to the parents. The nurse can inform the couple about potential problems so they can make an informed decision. Decision making should always lie with the couple involved. The nurse should provide information so that the couple can make an informed decision.

19. Which patient is most at risk for a low-birth-weight infant? A. 22-year-old, 60 inches tall, normal prepregnant weight B. 18-year-old, 64 inches tall, body mass index is <18.5 C. 30-year-old, 78 inches tall, prepregnant weight is 15 lb above the norm D. 35-year-old, 75 inches tall, total weight gain in previous pregnancies was 33 lb

ANS: B Rationale: The patient who has a low prepregnancy weight is associated with preterm labor and low-birth-weight infants. Women who are underweight should gain more during pregnancy to meet the needs of pregnancy as well as their own need to gain weight; patients who have a normal prepregnancy weight, who start pregnancy overweight, or who have a history of excessive weight gain in pregnancy are not at risk for low-birth-weight infants.

A pregnant patient asks the nurse if she can double her prenatal vitamin dose because she doesnot like to eat vegetables. What is the nurse's response regarding the danger of takingexcessive vitamins? A. Increases caloric intake B. Has toxic effects on the fetus C. Increases absorption of all vitamins D. Promotes development of pregnancy-induced hypertension (PIH)

ANS: B Rationale: The use of vitamin supplements in addition to food may increase the intake of some nutrients to doses much higher than the recommended amounts. Overdoses of some vitamins have been linked to fetal defects. Vitamin supplements do not contain calories. Vitamin supplements do not have better absorption than natural vitamins and minerals. There is no relationship between vitamin supplements and PIH.

What is the term for the step in maternal role attainment that relates to the woman giving up certain aspects of her previous life? A. Fantasy B. Grief work C. Role playing D. Looking for a fit

ANS: B Rationale: The woman experiences sadness as she realizes that she must give up certain aspects of her previous self and that she can never go back. This is called grief work. Fantasies allow the woman to try on a variety of possibilities or behaviors. This usually deals with how the child will look and the characteristics of the child. Role playing involves searching for opportunities to provide care for infants in the presence of another person. Looking for a fit is when the woman observes the behaviors of mothers and compares them with her own expectations.

Which of these findings would indicate a potential complication related to renal function during pregnancy? A. Increase in glomerular filtration rate (GFR) B. Increase in serum creatinine level C. Decrease in blood urea nitrogen (BUN) D. Mild proteinuria

ANS: B Rationale: With pregnancy, one would expect the serum creatinine and BUN levels to decrease. An elevation in the serum creatinine level should be investigated. With pregnancy, the GFR increases because of increased renal blood flow and is thus a normal expected finding. A decrease in the blood urea nitrogen level and mild proteinuria is expected findings in pregnancy.

You are preparing a patient for epidural placement by a nurse anesthetist in the LDR. Which interventions should be included in the plan of care? (Select all that apply.) A. Administer a bolus of 500 to 1000 mL of D5 normal saline prior to catheter placement. B. Have ephedrine available at bedside during catheter placement. C. Monitor blood pressure of patient frequently during catheter insertion and for the first 15 minutes of epidural administration. D. Insert a Foley catheter prior to epidural catheter placement. E. Monitor the patient for hypertension in response to epidural insertion.

ANS: B,C RATIONALE: A bolus of nondextrose fluid is recommended prior to epidural administration to prevent maternal hypotension. Ephedrine should be available at the bedside in case maternal hypotension is exhibited. Blood pressure should be monitored frequently during insertion and for the first 15 minutes of therapy. It is not necessary to insert a Foley catheter prior to epidural catheter placement. Hypertension is not a common clinical response to this treatment but hypotension is.

The clinical nurse educator is providing instruction to a group of new nurses during labor orientation. Which information regarding the factors that have a role in the initiation of labor should the educator include in this teaching session? (Select all that apply.) A. Progesterone levels become higher than estrogen levels. B. Natural oxytocin in conjunction with other substances plays a role. C. Stretching, pressure, and irritation of the uterus and cervix increase. D. The secretion of prostaglandins from the fetal membranes decreases.

ANS: B,C RATIONALE: Factors that appear to have a role in starting labor include the following: (1) natural oxytocinplays a part in labor's initiation in conjunction with other substances; and (2) stretching, pressure, and irritation of the uterus and cervix increase as the fetus reaches term size. The progesterone levels drop and estrogen levels increase. There is an increase in the secretion of prostaglandins from the fetal membranes.

The nurse is explaining fetal circulation to a group of nursing students. Which information should be included in the teaching session? (Select all that apply.) A. After birth the ductus venosus remains open, but the other shunts close. B. The foramen ovale shunts blood from the right atrium to the left atrium. C. The ductus venosus shunts blood from the liver to the inferior vena cava. D. The ductus arteriosus shunts blood from the right ventricle to the left ventricle.

ANS: B,C Rationale: The foramen ovale shunts oxygenated blood from the right atrium to the left atrium, bypassing the lungs. The ductus venosus shunts oxygenated blood from the liver to the inferior vena cava. All shunts close after birth. The ductus arteriosus shunts blood from the right ventricleto the aorta.

The nurse is teaching a patient taking prenatal vitamins how to avoid constipation. Which should the nurse plan to include in the teaching session? (Select all that apply.) A. Advise taking a daily laxative for constipation. B. Recommend a diet high in fruits and vegetables. C. Encourage an increase in fluid consumption during the day. D. Increase the intake of whole grains and whole grain products. E.Suggest increasing the intake of dairy products, especially cheeses.

ANS: B,C,D Rationale: Common sources of dietary fiber include fruits and vegetables (with skins when possible—apples, strawberries, pears, carrots, corn, potatoes with skins, and broccoli), whole grains, and whole grain products—whole wheat bread, bran muffins, bran cereals, oatmeal, brown rice, and whole wheat pasta. Increased intake of fluids can help prevent constipation. A pregnant patient should not take a daily laxative unless prescribed by her health care provider. Increased intake of dairy products, especially cheese, may increase constipation.

The nurse recognizes that fetal scalp stimulation may be prescribed to evaluate the response of the fetus to tactile stimulation. Which conditions contraindicate the use of fetal scalp stimulation? (Select all that apply.) A. Post-term fetus B. Maternal fever C. Placenta previa D. Induction of labor E. Prolonged rupture of membranes

ANS: B,C,E RATIONALE: Fetal scalp stimulation is not done when there is maternal fever (possibility of introducing microorganisms into the uterus), placenta previa (placenta overlies the cervix, and hemorrhage is likely), or prolonged rupture of membranes (risk of infection). Fetal scalp stimulation may be used to evaluate a post-term fetus' response to stimulation. It is also used to evaluate a fetus when labor is being induced.

The nurse is planning a prenatal class on fetal development. Which characteristics of prenatal development should the nurse include for a fetus of 24 weeks, based on fertilization age? (Select all that apply.) A. Ear cartilage firm B. Skin wrinkled and red C. Testes descending toward the inguinal rings D. Surfactant production nears mature levels E. Fetal movement becoming progressively more noticeable

ANS: B,C,E Rationale: A fetus of 24 weeks, based on fertilization age, will have wrinkled and red skin, testes descending toward inguinal rings, and the fetal movement becoming progressively more noticeable. Surfactant production nearing the mature levels does not occur until 32 weeks and ear cartilage is not firm until 38 weeks.

A woman who is 36 weeks pregnant asks the nurse to explain the vibroacoustic stimulator (VAS) test. Which should the nurse include in the response? (Select all that apply.) A. The test is invasive. B. The test uses sound to elicit fetal movements. C. The test may confirm nonreactive nonstress test results. D. The test can only be performed if contractions are present. E. Vibroacoustic stimulation can be repeated at 1-minute intervals up to three times.

ANS: B,C,E Rationale: Also referred to as VAS or acoustic stimulation, the vibroacoustic stimulator (similar to an electronic larynx) is applied to the maternal abdomen over the area of the fetal head. Vibration and sound are emitted for up to 3 seconds and may be repeated. A fetus near term responds by increasing the number of gross body movements, which can be easily seen and felt. The procedure can confirm reassuring NST findings and shorten the length of time necessary to obtain NST data. The test is noninvasive and contractions do not need to be present to perform the test.

Along with gas exchange and nutrient transfer, the placenta produces many hormones necessary for normal pregnancy, including which of the following? (Select all that apply.) A. Insulin B. Estrogen C. Progesterone D. Testosterone E. Human chorionic gonadotropin (hCG)

ANS: B,C,E Rationale: HCG causes the corpus luteum to persist and produce the necessary estrogens andprogesterone for the first 6 to 8 weeks. Estrogens cause enlargement of the woman's uterusand breasts and growth of the ductal system in the breasts and, as term approaches, plays a role in the initiation of labor. Progesterone causes the endometrium to change, providing early nourishment. Progesterone also protects against spontaneous abortion by suppressing maternal reactions to fetal antigens and reduces unnecessary uterine contractions. Other hormones produced by the placenta include hCT, hCA, and a number of growth factors. Insulin and testosterone are not secreted by the placenta.

Which assessment finding would cause a concern for a patient who had delivered vaginally? A. Estimated blood loss (EBL) of 500 mL during the birth process B. White blood cell count of 28,000 mm3 postbirth C. Patient complains of fingers tingling D. Patient complains of thirst

ANS: C RATIONALE: A patient's complaint of fingers tingling may represent respiratory alkalosis due tohyperventilation breathing patterns during labor. As such it requires intervention by the nurse to have the patient slow breathing down and restore normal carbon dioxide levels.

The nerve block used in labor that provides anesthesia to the lower vagina and perineum is referred to as a(n) A. local. B. epidural. C. pudendal. D. spinal block.

ANS: C RATIONALE: A pudendal block anesthetizes the lower vagina and perineum to provide anesthesia for an episiotomy and use of low forceps, if needed. A local provides anesthesia for the perineum at the site of the episiotomy. An epidural provides anesthesia for the uterus, perineum, and legs. A spinal block provides anesthesia for the uterus, perineum, and down the legs.

A patient presents to the labor and birth area for emergent birth. Vaginal exam reveals that the patient is fully dilated, vertex, +2 station, with ruptured membranes. The patient is extremely apprehensive because this is her first childbirth experience and asks for an epidural to be administered now. What is the priority nursing response based on this patient assessment? A. Use contact anesthesia for an epidural and prepare the patient per protocol. B. Tell the patient that she will not need any pain medication because the birth will be over in a matter of minutes and the pain will stop. C. Assist the patient with nonpharmacologic methods of pain distraction during this time as you prepare for vaginal birth. D. Call the physician for admitting orders.

ANS: C RATIONALE: By assisting the patient with nonpharmacologic methods of pain distraction, the nurse isfocusing on the patient's needs while still preparing for vaginal birth. The patient presents inan emergent situation with birth being imminent. Thus there is not enough time to administer an epidural. Telling the patient that she will not need any pain medication because the birthwill be over soon does not address the patient's concerns of apprehension and therefore is nottherapeutic. Because this is an emergency birth situation, the nurse should be attending to the patient. If needed, another nurse and/or supervisor can contact the physician.

To relieve a mild postdural puncture headache, the nurse should encourage the intake of A. milk. B. orange juice. C. tea or coffee. D. beef or chicken bouillon.

ANS: C RATIONALE: Caffeine is an oral therapy that is beneficial in relieving postdural puncture headache. Milk, juices, and bouillon will add oral hydration but lack the added benefit of the caffeine. Some patients prefer a cold caffeinated soft drink over coffee or tea.

Which assessment finding indicates that cervical dilation and/or effacement has occurred? A. Onset of irregular contractions B. Cephalic presentation at 0 station C. Bloody mucus drainage from vagina D. Fetal heart tones (FHTs) present in the lower right quadrant

ANS: C RATIONALE: Cervical dilation and/or effacement results in loss of the mucus plug as well as rupture of small capillaries in the cervix; irregular contractions, cephalic presentation, and FHTs in the lower right quadrant do not indicate the onset of cervical ripening.

Which of the following is the priority intervention for a supine patient whose monitor strip shows decelerations that begin after the peak of the contraction and return to the baseline after the contraction ends? A. Increase IV infusion. B. Elevate lower extremities. C. Reposition to left side-lying position. D. Administer oxygen per face mask at 4 to 6 L/minute.

ANS: C RATIONALE: Decelerations that begin at the peak of the contractions and recover after the contractions end are caused by uteroplacental insufficiency. When the patient is in the supine position, the weight of the uterus partially occludes the vena cava and descending aorta, resulting in hypotension and decreased placental perfusion. Increasing the IV infusion, elevating the lower extremities, and administering O2 will not be effective as long as the patient is in a supine position.

Which physiologic effect may occur in the presence of increased maternal pain perception during labor? A. Increase in uterine contractions in response to catecholamine secretion B. Decrease in blood pressure in response to alpha receptors C. Decreased perfusion to the placenta in response to catecholamine secretion D. Increased uterine blood flow, causing increase in maternal blood pressure

ANS: C RATIONALE: Decreased perfusion to and from the placenta occurs as result of catecholamine secretion. A decrease in uterine contractions is seen in response to catecholamine secretion. Maternal blood pressure is increased in response to alpha receptors. Decreased uterine blood flow causes an increase in maternal blood pressure.

The nurse is assessing the duration of a patient's labor contractions. Which method does thenurse implement to assess the duration of labor contractions? A. Assess the strongest intensity of each contraction. B. Assess uterine relaxation between two contractions. C. Assess from the beginning to the end of each contraction. D. Assess from the beginning of one contraction to the beginning of the next.

ANS: C RATIONALE: Duration of labor contractions is the average length of contractions from beginning to end. Assessing the strongest intensity of each contraction assesses the strength or intensity of the contractions. Assessing uterine relaxation between two contractions is the interval of the contraction phase. Assessing from the beginning of one contraction to the beginning of the next is the frequency of the contractions.

The nurse admits a laboring patient at term. On review of the prenatal record, the patient'spregnancy has been unremarkable and she is considered low risk. In planning the patient'scare, at what interval will the nurse intermittently auscultate (IA) the fetal heart rate during the first stage of labor? A. Every 10 minutes B. Every 15 minutes C. Every 30 minutes D. Every 60 minutes

ANS: C RATIONALE: Evaluate the fetal monitoring strip systematically for the elements noted. The following are recommended assessment and documentation intervals for IA and EFM (although facility policies may be different): low-risk women, every 30 minutes during the active phase and every 15 minutes during the second stage.

To clarify the fetal condition when baseline variability is absent, the nurse should first A. monitor fetal oxygen saturation using fetal pulse oximetry. B. notify the physician so that a fetal scalp blood sample can be obtained. C. apply pressure to the fetal scalp with a glove finger using a circular motion. D. increase the rate of nonadditive IV fluid to expand the mother's blood volume.

ANS: C RATIONALE: Fetal scalp stimulation helps identify whether the fetus responds to gentle massage. An acceleration in response to the massage suggests that the fetus is in normal oxygen and acid-base balance. Monitoring fetal oxygen saturation using fetal pulse oximetry is no longer available in the United States. Obtaining a fetal scalp blood sample is invasive and the results are not immediately available. Increasing the rate of nonadditive IV fluid would not clarify the fetal condition.

Which of the following therapeutic applications provides the most accurate information related to uterine contraction strength? A. External fetal monitoring (EFM) B. Internal fetal monitoring C. Intrauterine pressure catheter (IUPC) D. Maternal comments based on perception

ANS: C RATIONALE: IUPC is a clinical tool that provides an accurate assessment of uterine contraction strength. EFM provides evidence of contraction pattern and fetal heart rate but only estimates uterine contraction strength. Internal fetal monitoring provides direct evidence of fetal heart rate and contraction pattern. It only estimates uterine contraction strength. Maternal comments related to pain may not be related to uterine contraction strength and thus are influenced by thepatient's own pain perception.

If the position of a fetus in a cephalic presentation is right occiput anterior, the nurse should assess the fetal heart rate in which quadrant of the maternal abdomen? A. Right upper B. Left upper C. Right lower D. Left lower

ANS: C RATIONALE: If the fetus is in a right occiput anterior position, the fetal spine will be on the mother's rightside. The best location to hear the fetal heart rate is through the fetal shoulder, which would be in the right lower quadrant. The right upper, left upper, and left lower areas are not the best locations for assessing the fetal heart rate in this case.

The nurse assesses a laboring patient's contraction pattern and notes the frequency at every 3to 4 minutes, duration 50 to 60 seconds, and the intensity is moderate by palpation. What is the most accurate documentation for this contraction pattern? A. Stage 1, latent phase B. Stage 2, latent phase C. Stage 1, active phase D. Stage 2, active phase

ANS: C RATIONALE: In the active phase of stage 1, contractions are about 2 to 5 minutes apart, with a duration of about 40 to 60 seconds and an intensity that ranges from moderate to strong. During the latent phase of stage 1, the interval between contractions shortens until contractions are about 5 minutes apart. Duration increases to 30 to 40 seconds by the end of the latent phase. During stage 2, latent phase, the woman is resting and preparing to push; she likely has not experienced the pushing reflex (sometimes referred to as the Ferguson reflex). She is actively bearing down during the active phase of the second stage.

If a notation on the patient's health record states that the fetal position is LSP, this indicatesthat the A. head is in the right posterior quadrant of the pelvis. B. head is in the left anterior quadrant of the pelvis. C. buttocks are in the left posterior quadrant of the pelvis. D. buttocks are in the right upper quadrant of the abdomen.

ANS: C RATIONALE: LSP explains the position of the fetus in the maternal pelvis. L = left side of the pelvis, S = sacrum (fetus is in breech presentation), P = posterior quadrants of the pelvis. When the head is in the right posterior quadrant of the pelvis, the position is ROP. When the head is in the left anterior quadrant of the pelvis, the position is ROA. When the buttocks are in the upper quadrant of the abdomen, the position would be ROA, ROP, LOA, LOP, LOT, or ROT.

The patient presenting at 38 weeks' gestation, gravida 1, para 0, vaginal exam 4 cm, 100% effaced, +1 station vertex. What is the most likely intervention for this fetal heart rate pattern? A. Continue oxytocin (Pitocin) infusion. B. Contact the anesthesia department for epidural administration. C. Change maternal position. D. Administer Narcan to patient and prepare for immediate vaginal delivery.

ANS: C RATIONALE: Late decelerations indicate fetal compromise (uteroplacental insufficiency) and are considered to be a significant event requiring immediate assessment and intervention. Of all the options listed, changing maternal position may increase placental perfusion. In the presence of late decelerations, Pitocin infusion should be stopped. Contacting anesthesia for epidural administration will not solve the existing problem of late decelerations. There are no data to support the administration of Narcan and because patient is still in early labor, birth is not imminent.

The method of anesthesia in labor that is considered the safest for the fetus is A. epidural block. B. pudendal block. C. local infiltration. D. spinal (subarachnoid) block.

ANS: C RATIONALE: Local infiltration of the perineum rarely has any adverse effects on the mother or the fetus. With an epidural, pudendal, or spinal block the fetus can be affected by maternal side effects and maternal hypotension.

The fetal heart rate baseline increases 20 bpm after vibroacoustic stimulation. The best interpretation of this is that the fetus is showing A. a worsening hypoxia. B. progressive acidosis. C. an expected response. D. parasympathetic stimulation.

ANS: C RATIONALE: The fetus with adequate reserve for the stress of labor will usually respond to vibroacoustic stimulation with a temporary increase in the fetal heart rate (FHR) baseline. An increase in the FHR with stimulation does not indicate hypoxia. An increase in the FHR after stimulation is an anticipated response and does not indicate acidosis. An increase in the FHR after stimulation is a normal pattern, and does not indicate problems with the parasympathetic nervous system. A Category I pattern is normal and strongly predictive of adequate fetal acid-base status.

Which patient will be most receptive to teaching about nonpharmacologic pain control methods? A. Gravida 1, para 0, in transition B. Gravida 2, para 1, admitted at 8 cm C. Gravida 1, para 0, dilated 2 cm, 80% effaced D. Gravida 3, para 2, complaining of intense perineal pressure

ANS: C RATIONALE: The latent phase of labor is the best time for intrapartum teaching; the latent phase of labor is the first centimeter of cervical dilation. Patients in the transition phase (8 to 10 cm) are experiencing intense pain and are not receptive to teaching. A multigravida complaining of intense perineal pressure indicates a patient whose birth is imminent.

Which statement is true with regard to the type of pain associated with childbirth experience? A. Pain is constant throughout the labor experience. B. Labor pain during childbirth is considered to be an abnormal response. C. Pain associated with childbirth is self-limiting. D. Pain associated with childbirth does not allow for adequate preparation.

ANS: C RATIONALE: The pain associated with childbirth is self-limiting in that it typically stops once the child is delivered. Pain is intermittent during the labor experience. Labor pain is considered to be a normal response during childbirth. Pregnant woman can prepare for the expected pain of childbirth by taking prepared childbirth classes and using relaxation techniques during the course of labor.

When the deceleration pattern of the fetal heart rate mirrors the uterine contraction, which nursing action is indicated? A. Reposition the patient. B. Apply a fetal scalp electrode. C. Record this normal pattern. D. Administer oxygen by nasal cannula.

ANS: C RATIONALE: The periodic pattern described is early deceleration that is not associated with fetal compromise and requires no intervention. This is a Category I tracing which is a normal pattern. Repositioning the patient, applying a fetal scalp electrode, or administering oxygen would be interventions performed for Category II or III patterns.

Proper placement of the tocotransducer for electronic fetal monitoring is A. Inside the uterus. B. On the fetal scalp. C. Over the uterine fundus. D. Over the mother's lower abdomen.

ANS: C RATIONALE: The tocotransducer monitors uterine activity and should be placed over the fundus, where the most intensive uterine contractions occur. The tocotransducer is for external use. The tocotransducer monitors uterine contractions. The most intensive uterine contractions occur at the fundus; this is the best placement area.

A patient in labor reports a feeling of burning pain during the second stage of labor. This type of pain is associated with A. visceral pain. B. tissue ischemia. C. somatic pain. D. cervical dilation.

ANS: C RATIONALE: This is an example of somatic pain experienced as a result of distention of the vagina and perineum during the second stage of labor. Visceral pain occurs in response to pressure on pelvic structures. Pain associated with ischemic tissue is a result of decreased blow flow to the uterus. The pain of cervical dilation is a major pain source during labor but, during the second stage of labor, the patient is already fully dilated so this would not be a factor.

The process of labor places significant metabolic demands on the obstetric patient. Which physiologic findings would be expected? A. Decreased maternal blood pressure as a result of stimulation of alpha receptors B. Uterine vasoconstriction as a result of stimulation of beta receptors C. Increased maternal demand for oxygen D. Increased blood flow to placenta because of catecholamine release

ANS: C RATIONALE: With regard to labor, one would expect to see an increase in maternal blood pressure because of stimulation of alpha receptors. Uterine vasoconstriction would occur in response to stimulation of alpha receptors. One would expect to see a decrease in blood flow to the placenta. The maternal metabolic rate is increased during labor, along with an increase in maternal demand for oxygen.

Pregnant patients can usually tolerate the normal blood loss associated with childbirth because of which physiologic adaptation to pregnancy? A. A higher hematocrit B. Increased leukocytes C. Increased blood volume D. A lower fibrinogen level

ANS: C RATIONALE: Women have a significant increase in blood volume during pregnancy. After birth, the additional circulating volume is no longer necessary. The hematocrit decreases with pregnancy due to the higher fluid volume. Leukocyte levels increase during labor; however, that is not the reason for the toleration of blood loss. Fibrinogen levels increase with pregnancy.

A pregnant woman in labor is quite anxious and has been breathing rapidly during contractions. She now complains of a tingling sensation in her fingers. What is the priority nursing intervention at this time? A. Perform a vaginal exam to denote progress. B. Reposition the patient to a side lying position. C. Instruct the patient to breathe into her cupped hands. D. Notify the physician about current findings.

ANS: C RATIONALE:This patient is exhibiting signs of hyperventilation associated with a rapid breathing pattern, which can occur during the labor process. The nurse should instruct the patient to breathe into her cupped hands to retain carbon dioxide that is being lost from the hyperventilation process. A vaginal exam is not indicated because there is no evidence of fetal distress and/or change in labor progress. Repositioning the patient may be an option but is not the priority intervention at this time. Notifying the physician is not appropriate at this time because the RN should attend to actions that are readily available to her based on her scope of practice and standard

The nurse is caring for the newborn of a mother with diabetes. For which signs of hypoglycemia should the nurse assess the newborn? Select all that apply. 1. Pallor 2. Irritability 3. Hypotonia 4. Ineffective sucking 5. Excessive birth weight

Correct 1. Pallor 2. Irritability 3. Hypotonia 4. Ineffective sucking

The primary reason for evaluating alpha-fetoprotein (AFP) levels in maternal serum is to determine whether the fetus has which condition? A. Hemophilia B. Sickle cell anemia C. A neural tube defect D. Abnormal lecithin-to-sphingomyelin ratio

ANS: C Rationale: An open neural tube allows a high level of AFP to seep into the amniotic fluid and enter the maternal serum. Hemophilia is a genetic defect and is best detected with chromosomal studies, such as chorionic villus sampling or amniocentesis. Sickle cell anemia is a genetic defect and is best detected with chromosomal studies such as chorionic villus sampling or amniocentesis. L/S ratios are determined with an amniocentesis and are usually performed in the third trimester.

During vital sign assessment of a pregnant patient in her third trimester, the patient complains of feeling faint, dizzy, and agitated. Which nursing intervention is most appropriate? A. Have the patient stand up and retake her blood pressure. B. Have the patient sit down and hold her arm in a dependent position. C. Have the patient turn to her left side and recheck her blood pressure in 5 minutes. D. Have the patient lie supine for 5 minutes and recheck her blood pressure on both arms.

ANS: C Rationale: Blood pressure is affected by positioning during pregnancy. The supine position may cause occlusion of the vena cava and descending aorta. Turning the pregnant woman to a lateral recumbent position alleviates pressure on the blood vessels and quickly corrects supine hypotension. Pressures are significantly higher when the patient is standing. This would cause an increase in systolic and diastolic pressures. The arm should be supported at the same level of the heart. The supine position may cause occlusion of the vena cava and descending aorta, creating hypotension.

A patient, gravida 2, para 1, comes for a prenatal visit at 20 weeks of gestation. Her fundus is palpated 3 cm below the umbilicus. This finding is A. appropriate for gestational age. B. a sign of impending complications. C. lower than normal for gestational age. D. higher than normal for gestational age.

ANS: C Rationale: By 20 weeks, the fundus should reach the umbilicus. The fundus should be at the umbilicu s at 20 weeks, so 3 cm below the umbilicus is an inappropriate height and needs further assessment. This is lower than expected at this date. It may be a complication, but it may also be because of incorrect dating of the pregnancy.

A pregnant woman is scheduled to undergo chorionic villus sampling (CVS) based on genetic family history. Which medication does the nurse anticipate will be administered? A. Magnesium sulfate B. Prostaglandin suppository C. RhoGAM if the patient is Rh-negative D. Betamethasone

ANS: C Rationale: CVS can increase the likelihood of Rh sensitization if a woman is Rh-negative. There is no indication for magnesium sulfate because it is used to stop preterm labor. There is no indication for administration of a prostaglandin suppository. Betamethasone is given to pregnant women in preterm labor to improve fetal lung maturity.

When is the earliest interval that chorionic villus sampling (CVS) can be performed during pregnancy? A. 4 weeks B. 8 weeks C. 10 weeks D. 14 weeks

ANS: C Rationale: CVS is normally performed between 10 and 13 weeks gestation. The fetal villus tissue can be analyzed directly for chromosomal, metabolic, or DNA abnormalities. It is too early to perform CVS at 4 or 8 weeks of pregnancy. The test can no longer be performed a 14 weeks gestation. Results are available within 24 to 48 hours.

Which situation best describes a man trying on fathering behaviors? A. Reading books on newborn care B. Spending more time with his siblings C. Coaching a little league baseball team D. Exhibiting physical symptoms related to pregnancy

ANS: C Rationale: Coaching a little league baseball team shows interaction with children and assuming the behavior and role of a father. This best describes a man trying on the role of being a father. Men do not normally read information that is provided in advance. The nurse should be prepared to present information after the baby is born, when it is more relevant. The man will normally seek closer ties with his father. Exhibiting physical symptoms related to pregnancy is called couvade.

The primary side effect of maternal narcotic analgesia in the newborn is A. tachypnea. B. bradycardia. C. acrocyanosis. D. respiratory depression.

ANS: D RATIONALE: An infant delivered within 1 to 4 hours of maternal analgesic administration is at risk for respiratory depression from the sedative effects of the narcotic. The infant who is having a side effect to maternal analgesics normally would have a decrease in respirations, not an increase. Bradycardia is not the anticipated side effect of maternal analgesics. Acrocyanosis is an expected finding in a newborn and is not related to maternal analgesics.

For the pregnant patient who is a vegan, what combination of foods will the nurse advise to meet the nutritional needs for all essential amino acids? A. Eggs and beans B. Fruits and vegetables C. Grains and legumes D. Vitamin and mineral supplements

ANS: C Rationale: Combining incomplete plant proteins with other plant foods that have complementary amino acids allows intake of all essential amino acids. Dishes that contain grains (e.g., wheat, rice, corn) and legumes (e.g., garbanzo, navy, kidney, or pinto beans, peas, peanuts) are combinations that provide complete proteins. Eggs are not consumed by vegans. Fruits and vegetables alone will not provide the essential amino acids. Vitamin and mineral supplements do not provide amino acids.

A gravida 1 patient at 32 weeks of gestation reports that she has severe lower back pain. Whatshould the nurse's assessment include? A. Palpation of the lumbar spine B. Exercise pattern and duration C. Observation of posture and body mechanics D. Ability to sleep for at least 6 hours uninterrupted

ANS: C Rationale: Correct posture and body mechanics can reduce lower back pain caused by increasing lordosis. Pregnancy should not cause alterations in the spine. Any assessment for malformation should be done early in pregnancy. Certain exercises can help relieve back pain. Rest is important for overall well-being; however, the primary concern related to back pain is a thorough evaluation of posture and body mechanics.

The nurse is conducting a staff in-service on multifetal pregnancy. Which statement regarding dizygotic twin development should the nurse include in the teaching session? A. Dizygotic twins arise from two fertilized ova and are the same sex. B. Dizygotic twins arise from a single fertilized ovum and are always of the same sex. C. Dizygotic twins arise from two fertilized ova and may be the same sex or different sexes. D. Dizygotic twins arise from a single fertilized ovum and may be the same sex or different sexes.

ANS: C Rationale: Dizygotic twins arise from two ova that are fertilized by different sperm. They may be the same or different gender, and they may not have similar physical traits. Monozygotic twins are always the same sex. A single fertilized ovum that produces twins is called monozygotic. Dizygotic twins are from two fertilized ova and may or may not be the same sex.

The nurse is explaining the physiology of uterine contractions to a group of nursing students. Which statement best explains the maternal-fetal exchange of oxygen and waste products during a contraction? A. Little to no affect B. Increases as blood pressure decreases C. Diminishes as the spiral arteries are compressed D. Continues except when placental functions are reduced

ANS: C Rationale: During labor contractions, the maternal blood supply to the placenta gradually stops as the spiral arteries supplying the intervillous space are compressed by the contracting uterine muscle. The exchange of oxygen and waste products is affected by contractions. The exchange of oxygen and waste products decreases. The maternal blood supply to the placenta gradually stops with contractions.

Which information is covered by early pregnancy classes offered in the first and second trimesters? A. Methods of pain relief B. The phases and stages of labor C. Coping with common discomforts of pregnancy D. Prebirth and postbirth care of a patient having a cesarean birth

ANS: C Rationale: Early pregnancy classes focus on the first two trimesters and cover information on adapting to pregnancy, dealing with early discomforts, and understanding what to expect in the months ahead. Methods of pain relief are discussed in a childbirth preparation class. The phases and stages of labor are usually covered in a childbirth preparation class. Cesarean birth preparation classes discuss prebirth and postbirth of a patient having a cesarean birth.

A patient at 8 weeks' gestation complains to the nurse, "I feel sick almost every morning. And I throw up at least two or three times a week." What is the nurse's best guidance for thispatient? A. "Do you like cheese?" B. "Try eating four meals a day instead of three meals a day." C. "Try eating peanut butter on whole wheat bread right before going to bed." D. "If you can eat enough throughout the day, you don't have to worry about being sick."

ANS: C Rationale: Eating a bedtime protein snack helps maintain glucose levels throughout the night. Cheese is high in fat and can aggravate nausea. Small and frequent meals is the optimal recommendation. Four meals a day would not be ideal for a patient experiencing nausea, sheneeds to eat more frequently. Consumption is not the patient's stated concern—it is the nausea and vomiting.

A patient with an IUD in place has a positive pregnancy test. When planning care, the nurse will base decisions on which anticipated action? A. A therapeutic abortion will need to be scheduled since fetal damage is inevitable. B. Hormonal analyses will be done to determine the underlying cause of the false-positive test result. C. The IUD will need to be removed to avoid complications such as miscarriage or infection. D. The IUD will need to remain in place to avoid injuring the fetus.

ANS: C Rationale: Pregnancy with an intrauterine device (IUD) in place is unusual; however, it can occur and cause complications such as spontaneous abortion and infection. A therapeutic abortion is not indicated unless infection occurs.

A patient relates a story of how her boyfriend is feeling her aches and pains associated with her pregnancy. She is concerned that her boyfriend is making fun of her concerns. How would you respond to this patient statement? A. Tell her not to worry because it is natural for her boyfriend to make her feel better by identifying with her pregnancy. B. Refer the patient to a psychologist for counseling to deal with this problem because it is clearly upsetting her. C. Explain that her boyfriend may be experiencing couvade syndrome and that this is a normal finding seen with male partners. D. Ask the patient specifically to define her concerns related to her relationship with her boyfriend and suggest methods to stop this type of behavior by her significant other.

ANS: C Rationale: Provide factual information that will help reduce stress and modify acceptance. Telling her not to worry does not address the possibility that her boyfriend may be experiencing couvade syndrome. The patient is expressing concern but does not have all the facts related to couvade syndrome and requires education, rather than referral. Couvade syndrome is not an abnormal condition and should be treated with acceptance and understanding.

What is the best explanation that the nurse can provide to a patient who is concerned that shehas "pseudoanemia" of pregnancy? A. Have her write down her concerns and tell her that you will ask the physician to respond once the lab results have been evaluated. B. Tell her that this is a benign self-limiting condition that can be easily corrected by switching to a high-iron diet. C. Inform her that because of the pregnancy, her blood volume has increased, leading to a substantial dilution effect on her serum blood levels, and that most women experience this condition. D. Contact the physician and get a prescription for iron pills to correct this condition.

ANS: C Rationale: Providing factual information based on physiologic mechanisms is the best option. Although having the patient write down her concerns is reasonable, the nurse should not refer thisconversation to the physician but rather address the patient's specific concerns. Switching to ahigh-iron diet will not correct this condition. This physiologic pattern occurs during pregnancy as a result of hemodilution from excess blood volume. Iron medication is not indicated for correction of this condition. There is no need to contact the physician for a prescription.

The health care provider reports that the primigravida's fundus can be palpated at the umbilicus. Which priority question will the nurse include in the patient's assessment? A. "Have you noticed that it is easier for you to breathe now?" B. "Would you like to hear the baby's heartbeat for the first time?" C. "Have you felt a fluttering sensation in your lower pelvic area yet?" D. "Have you recently developed any unusual cravings, such as for chalk or dirt?"

ANS: C Rationale: Quickening is the first maternal sensation of fetal movement and is often described as a fluttering sensation. Quickening is detected at approximately 20 weeks in the primigravidaand as early as 16 weeks in the multigravida. The fundus is at the umbilicus at 20 weeks'gestation. Lightening is associated with descent of the fetal head into the maternal pelvis and is associated with improved lung expansion. Lightening occurs approximately 2 weeks before birth in the primipara. Fetal heart tones can be detected by Doppler as early as 9 to 12 weeks of gestation. Pica is the craving for nonnutritive substances such as chalk, dirt, clay, or sand. It can develop at any time during pregnancy. It can be associated with malnutrition and thehealth care provider should monitor the patient's hematocrit/hemoglobin, zinc, and ironlevels.

When explaining the recommended weight gain to your patient, the nurse's teaching should include which statement? A. "All pregnant women need to gain a minimum of 25 to 35 lb." B. "The fetus, amniotic fluid, and placenta require 15 lb of weight gain." C. "Weight gain in pregnancy is based on the patient's prepregnant body mass index." D. "More weight should be gained in the first and second trimesters and less in the third."

ANS: C Rationale: Recommendations for weight gain in pregnancy are based on the woman's prepregnancyweight for her height (body mass index). Depending on the prepregnant weight, recommendation for weight gain may be more or less than 25 to 35 lb. The combination of the fetus, amniotic fluid, and placenta averages about 11 lb in the patient who has a normal BMI. Less weight should be gained in the first trimester, when the fetus needs fewer nutrients for growth, and more in the third trimester, when fetal growth is accelerated.

4. What is the gravida and para for a patient who delivered triplets 2 years ago and is now pregnant again? a. 2,3 b. 1,2 c. 2,1 d. 1,3

ANS: C Rationale: She has had two pregnancies (gravida 2); para refers to the outcome of the pregnancy rather than the number of infants from that pregnancy. She is pregnant now, so that would make her a gravida 2. She is para 1 because she had one pregnancy that progressed to the age of viability.

Some of the embryo's intestines remain within the umbilical cord during the embryonic period because the a. intestines need this time to grow until week 15. b. nutrient content of the blood is higher in this location. c. abdomen is too small to contain all the organs while they are developing. d. umbilical cord is much larger at this time than it will be at the end of pregnancy.

ANS: C Rationale: The abdominal contents grow more rapidly than the abdominal cavity, so part of their development takes place in the umbilical cord. By 10 weeks, the abdomen is large enough to contain them. The intestines remain within the umbilical cord only until about week 10. Blood supply is adequate in all areas; intestines stay in the umbilical cord for about 10 weeks because they are growing faster than the abdomen. Intestines begin their development within the umbilical cord, but only because the liver and kidneys occupy most of the abdominal cavity, not because of the size of the umbilical cord.

An expectant father asks the nurse, "Which part of the mature sperm contains the male chromosome?" What is the correct response by the nurse? a. X-bearing sperm b. The tail of the sperm c. The head of the sperm d. The middle portion of the sperm

ANS: C Rationale: The head of the sperm contains the male chromosomes that will join the chromosomes of the ovum. If an X-bearing sperm fertilizes the ovum, the baby will be female. The tail of the sperm helps propel the sperm toward the ovum. The middle portion of the sperm supplies energy for the tail's whip-like action.

Which patient has correctly increased her caloric intake from her recommended pregnancy intake to the amount necessary to sustain breastfeeding in the first 6 postpartum months? A. From 1800 to 2200 calories per day B. From 2000 to 2500 calories per day C. From 2200 to 2530 calories per day D. From 2500 to 2730 calories per day

ANS: C Rationale: The increased calories necessary for breastfeeding are 500, with 330 calories coming from increased caloric intake and 170 calories from maternal stores. An increase of 230 calories is insufficient for breastfeeding. An increase of 400 and 500 calories is above the recommended amount.

What is the term for a nonstress test in which there are two or more fetal heart rate accelerations of 15 or more beats per minute (BPM) with fetal movement in a 20-minute period? a. Positive b. Negative c. Reactive d. Nonreactive

ANS: C Rationale: The nonstress test (NST) is reactive (normal) when there are two or more fetal heart rate accelerations of at least 15 BPM (each with a duration of at least 15 seconds) in a 20-minute period. A positive result is not used with an NST. The contraction stress test (CST) uses positive as a result term. A negative result is not used with an NST. The CST uses negative as a result term. A nonreactive result means that the heart rate did not accelerate during fetal movement.

In preparing a pregnant patient for a nonstress test (NST), which of the following should be included in the plan of care? A. Have the patient void prior to being placed on the fetal monitor because a full bladder will interfere with results. B. Maintain NPO status prior to testing. C. Position the patient for comfort, adjusting the tocotransducer belt to locate fetal heart rate. D. Have an infusion pump prepared with oxytocin per protocol for evaluation.

ANS: C Rationale: The nurse must adjust the tocotransducer to find the best location to pick up and record the fetal heart rate. Positioning the patient for comfort during testing is a prime concern. Although a full bladder may affect patient comfort, it will not interfere with testing results. NPO status is not required for an NST. Instead, a pregnant patient should maintain her normal nutritional intake to provide energy to herself and the fetus. An infusion pump with oxytocin is required for a contraction stress test (CST).

The nurse is conducting a prenatal nutrition education class for a group of nursing students. Which statement best describes the condition known as pica? A. Iron-deficiency anemia B. Intolerance to milk products C. Ingestion of nonfood substances D. Episodes of anorexia and vomiting

ANS: C Rationale: The practice of eating substances not normally thought of as food is called pica. Clay, dirt, and solid laundry starch are the substances most commonly ingested. Pica may produce iron-deficiency anemia if proper nutrition is decreased. Intolerance to milk products is termedlactose intolerance. Pica is not related to anorexia and vomiting.

A patient at 36 weeks gestation is undergoing a nonstress (NST) test. The nurse observes the fetal heart rate baseline at 135 beats per minute (bpm) and four nonepisodic patterns of the fetal heart rate reaching 160 bpm for periods of 20 to 25 seconds each. How will the nurse record these findings? A. NST positive, nonreassuring B. NST negative, reassuring C. NST reactive, reassuring D. NST nonreactive, nonreassuring

ANS: C Rationale: The presence of at least three accelerations of at least 15 beats, over at least 15 seconds, over a duration of at least 20 minutes, is considered reactive and reassuring. Nonreactive testing reveals no or fewer accelerations over the same or longer period. The NST test is not recorded as positive or negative.

A pregnant woman complains of frequent heartburn. The patient states that she has never had these symptoms before and wonders why this is happening now. The most appropriate response by the nurse is to A. examine her dietary intake pattern and tell her to avoid certain foods. B. tell her that this is a normal finding during early pregnancy and will resolve as she gets closer to term. C. explain to the patient that physiologic changes caused by the pregnancy make her more likely to experience these types of symptoms. D. refer her to her health care provider for additional testing because this is an abnormal finding.

ANS: C Rationale: The presentation of heartburn is a normal abnormal finding that can occur in pregnant woman because of relaxation of the lower esophageal sphincter as a result of the physiologic effects of pregnancy. Although foods may contribute to the heartburn, the patient is asking why this presentation is occurring, so the nurse should address the cause first. It is independent of gestation. There is no need to refer to the physician at this time because this is a normal abnormal finding. There is no evidence of complications ensuing from this presentation.

What is the physiologic reason for vascular volume increasing by 40% to 60% during pregnancy? A. Prevents maternal and fetal dehydration B. Eliminates metabolic wastes of the mother C. Provides adequate perfusion of the placenta D. Compensates for decreased renal plasma flow

ANS: C Rationale: The primary function of increased vascular volume is to transport oxygen and nutrients to the fetus via the placenta. Preventing maternal and fetal dehydration is not the primary reason for the increase in volume. Assisting with pulling metabolic wastes from the fetus for maternal excretion is one purpose of the increased vascular volume. Renal plasma flow increases during pregnancy.

The nurse evaluates a pattern on the fetal monitor that appears similar to early decelerations. The deceleration begins near the acme of the contraction and continues well beyond the end of the contraction. Which nursing action indicates the proper evaluation of this situation? A. This pattern reflects variable decelerations. No interventions are necessary at this time. B. Document this Category I fetal heart rate pattern and decrease the rate of the intravenous (IV) fluid. C. Continue to monitor these early decelerations, which occur as the fetal head is compressed during a contraction. D. This deceleration pattern is associated with uteroplacental insufficiency. The nurse must act quickly to improve placental blood flow and fetal oxygen supply.

ANS: D RATIONALE: A pattern similar to early decelerations, but the deceleration begins near the acme of the contraction and continues well beyond the end of the contraction, describes a late deceleration. Oxygen should be given via a snug face mask. Position the patient on her left side to increase placental blood flow. Variable decelerations are caused by cord compression. A vaginal examination should be performed to identify this potential emergency. This is not a normal pattern, rather it is a Category III tracing, predictive of abnormal fetal acid status at the time of observation. The IV rate should be increased in order to add to the mother's bloodvolume. These are late decelerations, not early; therefore interventions are necessary.

The nurse is concerned that a patient's uterine activity is too intense and that her obesity ispreventing accurate assessment of the actual intrauterine pressure. Based on this information, which action should the nurse take? A. Reposition the tocotransducer. B. Reposition the Doppler transducer. C. Obtain an order from the health care provider for a spiral electrode. D. Obtain an order from the health care provider for an intrauterine pressure catheter.

ANS: D RATIONALE: An intrauterine pressure catheter can measure actual intrauterine pressure. The tocotransducer measures the uterine pressure externally; this would not be accurate with an obese patient, even with repositioning. A Doppler auscultates the FHR. A scalp electrode (or spiral electrode) measures the fetal heart rate (FHR).

A major advantage of nonpharmacologic pain management is A. a more rapid labor is likely. B. more complete pain relief is possible. C. the woman remains fully alert at all times. D. there are no side effects or risks to the fetus.

ANS: D RATIONALE: Because nonpharmacologic pain management does not include analgesics, adjunct drugs, or anesthesia, it is harmless to the mother and the fetus. There is less pain relief with nonpharmacologic pain management during childbirth. Pain management may or may not alter the length of labor. At times, when pain is decreased, the mother relaxes and laborprogresses at a quicker pace. The woman's alertness is not altered by medication, but theincrease in pain will decrease alertness.

The laboring patient asks the nurse how the labor contractions cause the cervix to dilate. The nurse responds that labor contractions facilitate cervical dilation by A. promoting blood flow to the cervix. B. contracting the lower uterine segment. C. enlarging the internal size of the uterus. D. pulling the cervix over the fetus and amniotic sac.

ANS: D RATIONALE: Effective uterine contractions pull the cervix upward at the same time the fetus and amniotic sac are pushed downward. Blood flow decreases to the uterus during a contraction. The contractions are stronger at the fundus. The internal size becomes smaller with the contractions; this helps push the fetus down.

Which method of pain management would be safest for a gravida 3, para 2, admitted at 8 cm cervical dilation? A. Narcotics B. Spinal block C. Epidural anesthesia D. Breathing and relaxation techniques

ANS: D RATIONALE: Nonpharmacologic methods of pain management may be the best option for a woman in advanced labor. At 8 cm cervical dilation there is probably not enough time remaining to administer spinal anesthesia or epidural anesthesia. A narcotic given at this time may reach its peak at about the time of birth and result in respiratory depression in the newborn.

Childbirth preparation can be considered successful if which of the following outcomes is achieved? A. Labor was pain-free. B. The birth experiences of friends and families were ignored. C. Only nonpharmacologic methods for pain control were used. D. The patient rehearsed labor and practiced skills to master pain.

ANS: D RATIONALE: Preparation allows the woman to rehearse for labor and to learn new skills to cope with the pain of labor and the expected behavioral changes. Childbirth preparation does not guarantee a pain-free labor. A woman should be prepared for pain and anesthesia-analgesia realistically. Friends and families can be an important source of support if they convey realisticinformation about labor pain. Women will not always achieve their desired level of pain control by using nonpharmacologic methods alone.

A patient in active labor requests an epidural for pain management. What is the nurse's mostappropriate intervention at this juncture? A. Assess the fetal heart rate pattern over the next 30 minutes. B. Take the patient's blood pressure every 5 minutes for 15 minutes. C. Determine the patient's contraction pattern for the next 30 minutes. D. Initiate an IV infusion of lactated Ringer's solution at 2000 mL/hour over 30 minutes.

ANS: D RATIONALE: Rapid infusion of a nondextrose IV solution, often warmed, such as lactated Ringer's ornormal saline, before initiation of the block fills the vascular system to offset vasodilation. Preload IV quantities are at least 500 to 1000 mL infused rapidly. Vasodilation with corresponding hypotension can reduce placental perfusion and is most likely to occur within the first 15 minutes after the initiation of the epidural. Determining the fetal heart rate every 30 minutes is the standard of care. The patient is in active labor, which indicates a contraction pattern resulting in cervical dilation.

Observation of a fetal heart rate pattern indicates an increase in heart rate from the prior baseline rate of 152 bpm. Which physiologic mechanisms would account for this situation? A. Inhibition of epinephrine B. Inhibition of norepinephrine C. Stimulation of the vagus nerve D. Sympathetic stimulation

ANS: D RATIONALE: Sympathetic nerve innervation would result in an increase in fetal heart rate. The release of epinephrine as a result of sympathetic innervation would lead to an increase in fetal heart rate. The release of norepinephrine as a result of sympathetic innervation would lead to an increase in fetal heart rate. Stimulation of the vagus nerve would indicate parasympathetic innervation and result in a decreased heart rate.

A patient in labor presents with a breech presentation. The nurse understands that a breech presentation is associated with A. more rapid labor. B. a high risk of infection. C. maternal perineal trauma. D. umbilical cord compression.

ANS: D RATIONALE: The umbilical cord can compress between the fetal body and maternal pelvis when the body has been born but the head remains within the pelvis. Breech presentation is not associated with a more rapid labor. There is no higher risk of infection with a breech birth. There is no higher risk for perineal trauma with a breech birth. Most breech presentations are now delivered by caesarean birth.

Why is continuous electronic fetal monitoring generally used when oxytocin is administered? A. Fetal chemoreceptors are stimulated. B. The mother may become hypotensive. C. Maternal fluid volume deficit may occur. D. Uteroplacental exchange may be compromised.

ANS: D RATIONALE: The uterus may contract more firmly and the resting tone may be increased with oxytocin use. This response reduces the entrance of freshly oxygenated maternal blood into the intervillous spaces, depleting fetal oxygen reserves. Oxytocin affects the uterine muscles. Hypotension is not a common side effect of oxytocin. All laboring women are at risk for fluid volume deficit; oxytocin administration does not increase the risk.

To improve placental blood flow immediately after the injection of an epidural anesthetic, the nurse should A. give the woman oxygen. B. turn the woman to the right side. C. decrease the intravenous infusion rate. D. place a wedge under the woman's right hip.

ANS: D RATIONALE: Tilting the woman's pelvis to the left side relieves compression of the vena cava and compensates for a lower blood pressure without interfering with dispersal of the epidural medication. Oxygen administration will not improve placental blood flow. The woman needs to maintain the supine position for proper dispersal of the medication. Placing a wedge under the hip will relieve compression of the vena cava. The intravenous infusion rate needs to be increased to prevent hypotension.

When a pattern of variable decelerations occur, the nurse should immediately A. administer O2 at 8 to 10 L/minute. B. place a wedge under the right hip. C. increase the IV fluids to 150 mL/hour. D. position patient in a knee-chest position.

ANS: D RATIONALE: Variable decelerations are caused by conditions that reduce flow through the umbilical cord. The patient should be repositioned when the FHR pattern is associated with cord compression. The knee-chest position uses gravity to shift the fetus out of the pelvis to relieve cord compression. Administering oxygen will not be effective until cord compression is relieved. Increasing the IV fluids and placing a wedge under the right hip are not effective interventions for cord compression.

The nurse is monitoring a patient in labor and notes this fetal heart rate pattern on the electronic fetal monitoring strip (see figure). Which is the most appropriate nursing action at this time? A. Decrease the rate of the IV fluids. B. Document the fetal heart rate pattern. C. Explain to the patient that the pattern is normal. D. Perform a vaginal exam to detect a prolapsed cord.

ANS: D RATIONALE: Variable decelerations do not have the uniform appearance of early and late decelerations. Their shape, duration, and degree of fall below baseline rate vary. They fall and rise abruptly (within 30 seconds) with the onset and relief of cord compression, unlike the gradual fall and rise of early and late decelerations. A vaginal examination may identify a prolapsed cord, which may cause variable decelerations, bradycardia, or both as it is compressed. A vaginalexamination also evaluates the woman's labor status, which helps the birth attendant decide if labor should continue. This is a Category III tracing (abnormal); therefore the IV rate should be increased and an intervention needs to occur.

The nurse is teaching a childbirth education class. Which information regarding excessive pain in labor should the nurse include in the session? A. It usually results in a more rapid labor. B. It has no effect on the outcome of labor. C. It is considered to be a normal occurrence. D. It may result in decreased placental perfusion.

ANS: D RATIONALE: When experiencing excessive pain, the woman may react with a stress response that diverts blood flow from the uterus and the fetus. Excessive pain may prolong the labor because of increased anxiety in the woman. It may affect the outcome of the labor, depending on the cause and the effect on the woman. Pain is considered normal for labor. However, excessive pain may be an indication of other problems and must be assessed.

Which patient would require additional calories and nutrients? A. A 36-year-old female gravida 2, para 1, in her first trimester of pregnancy B. An 18-year-old female who delivered a 7-lb baby and is bottle feeding C. A 23-year-old female who had a cesarean birth and is bottle feeding D. A 20-year-old female who had a vaginal birth 5 months ago and is breastfeeding

ANS: D Rationale: A patient who is breastfeeding will require more calories and nutrients than women who are pregnant. The type of birth has no impact on nutrient intake. A patient who is bottle-feeding does not require additional calories.

When documenting a patient encounter, which term will the nurse use to describe the woman who is in the 28th week of her first pregnancy? a. Multigravida b. Multipara c. Nullipara d. Primigravida

ANS: D Rationale: A primigravida is a woman pregnant for the first time. A multigravida has been pregnant more than once. A nullipara is a woman who has never been pregnant or has not completed a pregnancy of 20 weeks or more. A primipara has delivered one pregnancy of at least 20 weeks. A multipara has delivered two or more pregnancies of at least 20 weeks.

Which comment made by a patient in her first trimester indicates ambivalent feelings? A. "My body is changing so quickly." B. "I haven't felt well since this pregnancy began." C. "I'm concerned about the amount of weight I've gained." D. "I wanted to become pregnant, but I'm scared about being a mother."

ANS: D Rationale: Ambivalence refers to conflicting feelings. Expressing a concern about being a mother indicates possible ambivalent feelings. Not feeling well since the pregnancy began does not reflect conflicting feelings. The woman is trying to confirm the pregnancy when she is stating the rapid changes to her body. She is not expressing conflicting feelings. By expressing concerns over gaining weight, which is normal, the woman is trying to confirm the pregnancy.

The nurse is explaining the function of the placenta to a pregnant patient. Which statement indicates to the nurse that further clarification is necessary? a. "My baby gets oxygen from the placenta." b. "The placenta functions to help excrete waste products." c. "The nourishment that I take in passes through the placenta." d. "The placenta helps maintain a stable temperature for my baby."

ANS: D Rationale: Amniotic fluid and not the placenta helps with thermoregulation. The remaining statements are correct regarding placental function.

Which suggestion is most helpful for the pregnant patient who is experiencing heartburn? A. Drink plenty of fluids at bedtime. B. Eat only three meals a day so the stomach is empty between meals. C. Drink coffee or orange juice immediately on arising in the morning. D. Use Tums or Rolaids to obtain relief, as directed by the health care provider.

ANS: D Rationale: Antacids high in calcium (e.g., Tums, Rolaids) can provide temporary relief. Fluids overstretch the stomach and may precipitate reflux when lying down. Instruct the patient to eat five or six small meals per day rather than three full meals. Coffee and orange juice stimulate acid formation in the stomach and may need to be eliminated from the diet.

A pregnant woman has come to the emergency department with complaints of nasal congestion and epistaxis. Which is the correct interpretation of these symptoms by the health care provider? A. Nasal stuffiness and nosebleeds are caused by a decrease in progesterone. B. These conditions are abnormal. Refer the patient to an ear, nose, and throat specialist. C. Estrogen relaxes the smooth muscles in the respiratory tract, so congestion and epistaxis are within normal limits. D. Estrogen causes increased blood supply to the mucous membranes and can result in congestion and nosebleeds.

ANS: D Rationale: As capillaries become engorged, the upper respiratory tract is affected by the subsequent edema and hyperemia, which causes these conditions, seen commonly during pregnancy. Progesterone is responsible for the heightened awareness of the need to breathe in pregnancy. Progesterone levels increase during pregnancy. The patient should be reassured that these symptoms are within normal limits. No referral is needed at this time. Relaxation of the smooth muscles in the respiratory tract is affected by progesterone.

A patient in her third trimester of pregnancy is asking about safe travel. Which statement should the nurse provide regarding safe travel during pregnancy? A. "Only travel by car during pregnancy." B. "Avoid use of the seat belt during the third trimester." C. "You can travel by plane until your 38th week of gestation." D. "If you are traveling by car stop to walk every 1 to 2 hours."

ANS: D Rationale: Car travel is safe during normal pregnancies. Suggest that the woman stop to walk every 1 to 2 hours so she can empty her bladder. Walking also helps decrease the risk of thrombosis that is elevated during pregnancy. Seat belts should be worn throughout the pregnancy. Instruct the woman to fasten the seat belt snugly, with the lap belt under her abdomen and across her thighs and the shoulder belt in a diagonal position across her chest and above the bulge of her uterus. Travel by plane is generally safe up to 36 weeks if there are no complications of the pregnancy, so only traveling by car is an inaccurate statement.

The nurse is meeting with a patient with an elevated BMI regarding an optimal diet for pregnancy. Which major source of nutrients should be a significant component of thispatient's diet? A. Fats B. Fiber C. Simple sugars D. Complex carbohydrates

ANS: D Rationale: Complex carbohydrates supply the pregnant woman with vitamins, minerals, and fiber. Fats provide 9 calories in each gram, in contrast to carbohydrates and proteins, which provide only 4 calories in each gram. Fiber is supplied primarily by complex carbohydrates. The most common simple carbohydrate is table sugar, which is a source of energy but does not provide any nutrients.

Which physiologic adaptation of pregnancy may lead to increased constipation during the pregnancy? A. Increased emptying time in the intestines B. Abdominal distention and bloating C. Decreased absorption of water D. Decreased motility in the intestines

ANS: D Rationale: Decreased motility in the intestines leading to increased water absorption would cause constipation. Increased emptying time in the intestines leads to increased nutrient absorption. Abdominal distention and bloating are a result of increased emptying time in the intestines. Decreased absorption of water would not cause constipation.

Which is the most important reason for evaluating the pattern of weight gain in pregnancy? A. Prevents excessive adipose tissue deposits B. Determines cultural influences on the woman's diet C. Assesses the need to limit caloric intake in obese women D. Identifies potential nutritional problems or complications of pregnancy

ANS: D Rationale: Deviations from the recommended pattern of weight gain may indicate nutritional problems or developing complications. Excessive adipose tissue may occur with excess weight gain but is not the reason for monitoring the weight gain pattern. The pattern of weight gain is not affected by cultural influences. It is important to monitor the pattern of weight gain for the developing complications.

A pregnant client arrives on the birthing unit from the emergency department with frank blood running down both legs and a reported low blood pressure. What is the priority nursing intervention? 1. Assessing fetal heart tones 2. Assessing for a prolapsed cord 3. Starting an intravenous (IV) infusion 4. Inserting a uterine pressure catheter

Correct 1. Assessing fetal heart tones

What is the purpose of amniocentesis for a patient hospitalized at 34 weeks of gestation with pregnancy-induced hypertension? A. Determine if a metabolic disorder exists. B. Identify the sex of the fetus. C. Identify abnormal fetal cells. D. Determine fetal lung maturity.

ANS: D Rationale: During the third trimester, amniocentesis is most often performed to determine fetal lung maturity. In cases of pregnancy-induced hypertension, preterm birth may be necessary because of changes in placental perfusion. The test is done in the early portion of the pregnancy if a metabolic disorder is genetic. Amniocentesis is done early in the pregnancy to do genetic studies and determine the sex. Identification of abnormal cells is done during the early portion of the pregnancy.

The nurse is directing an unlicensed assistive personnel (UAP) to obtain maternal vital signs between contractions. Which statement is the appropriate rationale for assessing maternal vital signs between contractions rather than at another interval? A. Vital signs taken during contractions are inaccurate. B. During a contraction, assessing fetal heart rate is the priority. C. Maternal blood flow to the heart is reduced during contractions. D. Maternal circulating blood volume increases temporarily during contractions.

ANS: D Rationale: During uterine contractions, blood flow to the placenta temporarily stops, causing a relativeincrease in the mother's blood volume, which in turn temporarily increases blood pressure and slows the pulse. Vital signs are altered by contractions but are considered accurate for a period of time. It is important to monitor the fetal response to contractions, but the question is concerned with the maternal vital signs. Maternal blood flow is increased during a contraction.

Which clinical finding is associated with inadequate maternal weight gain during pregnancy? A. Prolonged labor B. Preeclampsia C. Gestational diabetes D. Low-birth-weight infant

ANS: D Rationale: Inadequate maternal weight gain during pregnancy can manifest in the birth of a low-birth-weight infant. Prolonged labor and gestational diabetes are associated with excess weight gain during pregnancy. Preeclampsia is based on maternal hypertension, proteinuria, and edema states.

The patient has just learned that she is pregnant and overhears the gynecologist saying that shehas a positive Chadwick's sign. When the patient asks the nurse what this means, how would the nurse respond? A. "Chadwick's sign signifies an increased risk of blood clots in pregnant women because of a congestion of blood." B. "That sign means the cervix has softened as the result of tissue changes that naturally occur with pregnancy." C. "This means that a mucus plug has formed in the cervical canal to help protect you from uterine infection." D. "This sign occurs normally in pregnancy, when estrogen causes increased blood flow in the area of the cervix."

ANS: D Rationale: Increasing levels of estrogen cause hyperemia (congestion with blood) of the cervix, resulting in the characteristic bluish purple color that extends to include the vagina and labia. This discoloration, referred to as Chadwick's sign, is one of the earliest signs of pregnancy.Although Chadwick's sign occurs with hyperemia (congestion with blood), the sign does not signify an increased risk of blood clots. The softening of the cervix is called Goodell's sign, not Chadwick's sign. Although the formation of a mucus plug protects from infection, it is notcalled Chadwick's sign.

A pregnant patient with significant iron-deficiency anemia is prescribed iron supplements. The patient explains to the nurse that she cannot take iron because it makes her nauseous. What is the best response by the nurse? A. "Iron will be absorbed more readily if taken with orange juice." B. "It is important to take this drug regardless of this side effect." C. "Taking the drug with milk may decrease your symptoms." D. "Try taking the iron at bedtime on an empty stomach."

ANS: D Rationale: Iron taken at bedtime may be easier to tolerate. All the answers are true statements; however, only the option that states that iron taken at bedtime may be easier to tolerate addresses both optimal absorption of iron and alleviation of nausea, which will not be noticeable during sleep. It is true that taking iron with milk will decrease the symptoms; however, it will also decrease absorption.

Which comment made by a new mother exhibits understanding of her toddler's response to a new sibling? A. "I can't believe he is sucking his thumb again." B. "He is being difficult and I don't have time to deal with him." C. "When we brought the baby home, we made Michael stop sleeping in the crib." D. "My husband is going to stay with the baby so I can take Michael to the park tomorrow."

ANS: D Rationale: It is important for a mother to seek time alone with her toddler to reassure him that he is loved. It is normal for a child to regress when a new sibling is introduced into the home. The toddler may have feelings of jealousy and resentment toward the new baby taking attention away from him. Frequent reassurance of parental love and affection is important. Changes in sleeping arrangements should be made several weeks before the birth so the child does not feel displaced by the new baby.

Which factors should be considered a contraindication for transcervical chorionic villus sampling? A. Rh-negative mother B. Gestation less than 15 weeks C.Maternal age younger than 35 years D. Positive for group B Streptococcus

ANS: D Rationale: Maternal infection is a risk with this procedure, and it is contraindicated if the patient has an active infection in the cervix, vagina, or pelvic area. This procedure is done between 10 and 12 weeks. This procedure is usually done for women older than 35; however, if the woman is at high risk for fetal anomalies, her age is not a contraindication. The procedure can still be performed; however, Rh sensitization may occur if the mother is Rh-negative. Rho(D) immune globulin can be administered following the procedure.

A patient who is 16 weeks pregnant with her first baby asks how long it will be before she feels the baby move. Which is the nurse's best answer? a. "You should have felt the baby move by now." b. "The baby is moving, but you can't feel it yet." c. "Some babies are quiet and you don't feel them move." d. "Within the next month you should start to feel fluttering sensations."

ANS: D Rationale: Maternal perception of fetal movement (quickening) usually begins between 17 and 20 weeks after conception. Because this is her first pregnancy, movement is felt toward the later part of the 17 to 20 weeks. "The baby is moving, but you can't feel it yet" may be alarming to the woman. "Some babies are quiet and you don't feel them move" is a true statement; the fetus' movements are not strong enough to be felt until 17 to 20 weeks; however, this statement does not answer the woman's concern. Fetal movement should be felt between 17 and 20 weeks; if movement is not perceptible by the end of that time, further assessment will be necessary.

Which of the patient health behaviors in the first trimester would the nurse identify as a risk factor in pregnancy? A. Sexual intercourse two or three times weekly B. Moderate exercise for 30 minutes daily C. Working 40 hours a week as a secretary in a travel agency D. Relaxing in a hot tub for 30 minutes a day, several days a week

ANS: D Rationale: Pregnant women should avoid activities that might cause hyperthermia. Maternal hyperthermia, particularly during the first trimester, may be associated with fetal anomalies. She should not be in a hot tub for more than 10 minutes at less than 100F. Sexual intercourse is generally safe for the healthy pregnant woman; moderate exercise during pregnancy can strengthen muscles, reduce backache and stress, and provide a feeling of well-being; working during pregnancy is acceptable as long as the woman is not continually on her feet or exposed to environmental toxins and industrial hazards.

The nurse is reviewing a list of foods high in folic acid with a patient who is considering becoming pregnant. The nurse determines that the patient understands the teaching when the patient states she will include which list of foods in her diet? A. Peaches, yogurt, and tofu B. Strawberries, milk, and tuna C. Asparagus, lemonade, and chicken breast D. Spinach, orange juice, and fortified bran flakes

ANS: D Rationale: Prepregnant, the recommendation for folic acid is 800 mcg. Foods high in folic acid are dark green leafy vegetables, legumes (beans, peanuts), orange juice, asparagus, spinach, and fortified cereal and pasta. In the United States, folic acid is added to orange juice and wheat-based products.

Which physiologic findings related to gallbladder function may lead to the development of gallstones during pregnancy? A. Decrease in alkaline phosphatase levels compared with nonpregnant women B. Increase in albumin and total protein as a result of hemodilution C. Hypertonicity of gallbladder tissue D. Prolonged emptying time

ANS: D Rationale: Prolonged emptying time is seen during pregnancy and may lead to the development of gallstones. In pregnancy, there is a twofold to fourfold time increase in alkaline phosphatase levels as compared with those in nonpregnant woman. During pregnancy, a decrease in albumin level and total protein is seen as a result of hemodilution. Gallbladder tissue becomes hypotonic during pregnancy.

Uncontrolled maternal hyperventilation during labor results in A. metabolic acidosis. B. metabolic alkalosis. C. respiratory acidosis. D. respiratory alkalosis.

ANS: D Rationale: Rapid deep respirations cause the laboring woman to lose carbon dioxide through exhalation, resulting in respiratory alkalosis. Hyperventilation does not cause respiratory acidosis, metabolic acidosis, or metabolic alkalosis.

A relaxation technique that can be used during the childbirth experience to decrease maternal pain perception is A. using increased environmental stimulation as a method of distraction. B. restricting family and friends from visiting during the labor period to keep the patient focused on breathing techniques. C. medicating the patient frequently to reduce pain perception. D. assisting the patient in breathing methods aimed at taking control of pain perception based on the contraction pattern.

ANS: D Rationale: Relaxation techniques are aimed at incorporating mind and body activities to maintain control over pain. Additional environmental stimuli may have the opposite effect and increase patient anxiety, which will affect pain perception. Restricting visitors may have the opposite effect, leading to increased anxiety because of isolation. Medicating a patient may not decrease pain perception but may place the patient at risk for adverse reactions and/or complications of pregnancy related to medications.

A pregnant patient's diet may not meet her need for folate. Which food choice is an excellentsource of this nutrient? A. Chicken B. Cheese C. Potatoes D. Green leafy vegetables

ANS: D Rationale: Sources of folate include green leafy vegetables, whole grains, fruits, liver, dried peas, and beans. Chicken is a good source of protein, but poor in folate. Cheese is an excellent source of calcium, but poor in folate. Potatoes contain carbohydrates and vitamins but are poor in folate.

What is the purpose of initiating contractions in a contraction stress test (CST)? A. Increase placental blood flow. B. Identify fetal acceleration patterns. C. Determine the degree of fetal activity. D. Apply a stressful stimulus to the fetus.

ANS: D Rationale: The CST involves recording the response of the fetal heart rate to stress induced by uterine contractions. The CST records the fetal response to stress. It does not increase placental blood flow. The NST looks at fetal heart accelerations with fetal movements. The NST and biophysical profiles look at fetal movements.

Which is the method of childbirth that helps prevent the fear-tension-pain cycle by using slow abdominal breathing in early labor and rapid chest breathing in advanced labor? a. Bradley b. Lamaze c. Leboyer d. Dick-Read

ANS: D Rationale: The Dick-Read method helps prevent the fear-tension-pain cycle by using slow abdominal breathing in early labor and rapid chest breathing in advanced labor. The Lamaze method involves concentration and conditioning to help the woman respond to contractions with relaxation to decrease pain. Viewing childbirth as a traumatic experience, the Leboyer method uses decreased light and noise to help the newborn adapt to extrauterine life more easily. The Bradley method teaches women to use abdominal muscles to increase relaxation and breath control; it emphasizes avoidance of all medications and interventions.

A pregnant client is diagnosed with partial placenta previa. In explaining the diagnosis, the nurse tells the client that the usual treatment for partial placenta previais which of the following? a. Activity limited to bed rest b. Platelet infusion c. Immediate cesarean delivery d. Labor induction with oxytocin

Answer is A. Rationale: Treatment of partial placenta previa includes bed rest, hydration, and careful monitoring of the client's bleeding.

What does a birth plan help the parents accomplish? A. Avoidance of an episiotomy B. Determining the outcome of the birth C. Assuming complete control of the situation D. Taking an active part in planning the birth experience

ANS: D Rationale: The birth plan helps the woman and her partner look at the available options and plan the birth experience to meet their personal needs. A birth plan cannot dictate the need for or avoidance of an episiotomy. The outcome of the birth is not an absolute determinant. A birth plan does not assume complete control of the situation; it allows for expanding communication. Parents who prepare a birth plan should be educated that flexibility is essential as each labor and delivery is unique and may present unexpected complications.

The nurse is explaining the process of cell division during the preembryonic period to a group of nursing students. Which statement best describes the characteristics of the morula? a. Fertilized ovum before mitosis begins b. Double layer of cells that becomes the placenta c. Flattened, disk-shaped layer of cells within a fluid-filled sphere d. Solid ball composed of the first cells formed after fertilization

ANS: D Rationale: The morula is so named because it resembles a mulberry. It is a solid ball of 12 to 16 cells that develops after fertilization. The fertilized ovum is called the zygote. The placenta is formed from two layers of cells—the trophoblast, which is the other portion of the fertilized ovum, and the decidua, which is the portion of the uterus where implantation occurs. The flattened, disk-shaped layer of cells is the embryonic disk; it will develop into the body.

Which finding is a positive sign of pregnancy? A. Amenorrhea B. Breast changes C. Fetal movement felt by the woman D. Visualization of fetus by ultrasound

ANS: D Rationale: The only positive signs of pregnancy are auscultation of fetal heart tones, visualization of the fetus by ultrasound, and fetal movement felt by the examiner. Amenorrhea is a presumptive sign of pregnancy. Breast changes are a presumptive sign of pregnancy. Fetal movement is a presumptive sign of pregnancy.

A patient reports to the clinic nurse that she has not had a period in over 12 weeks, she istired, and her breasts are sore all of the time. The patient's urine test is positive for hCG. Whatis the correct nursing action related to this information? A. Ask the patient if she has had any nausea or vomiting in the morning. B. Schedule the patient to be seen by a health care provider within the next 4 weeks. C. Send the patient to the maternity screening area of the clinic for a routine ultrasound. D. Determine if there are any factors that might prohibit her from seeking medical care.

ANS: D Rationale: The patient has presumptive and probable indications of pregnancy. However, she has not sought out health care until late in the first or early in the second trimester. The nurse must assess for barriers to seeking health care, physical or emotional, because regular prenatal care is key to a positive pregnancy outcome. Asking if the patient has nausea or vomiting will only add to the list of presumptive signs of pregnancy, and this information will not add to the assessment data to determine whether the patient is pregnant. The patient needs to see a health care provider before the next 4 weeks because she is late in seeking early prenatal care. Ultrasound testing must be prescribed by a health care provider.

A pregnant patient comes into the medical clinic stating that her family and friends are telling her that she is always talking about the pregnancy and nothing else. She is concerned that something is wrong with her. What psychological behavior is she exhibiting? A. Antepartum obsession B. Ambivalence C. Uncertainty D. Introversion

ANS: D Rationale: The patient is exhibiting behaviors associated with introversion and/or narcissism. These are normal findings during pregnancy as long as they do not become obsessive to the exclusion of everything else. The patient is talking about the pregnancy but there is no evidence that it is affecting her perception of reality and/or ability to perform ADLs. It is normal for pregnant women to focus on the self as being of prime importance in their life initially during the pregnancy. Some women may feel ambivalent about their pregnancy, which is a normalreaction. However, this patient's behavior does not support this finding. Some women react with uncertainty at the news of being pregnant, which is a normal reaction. However, thispatient's behavior does not support this finding.

Which technique would provide the best pain relief for a pregnant woman with an occiput posterior position? A. Neuromuscular disassociation B. Effleurage C. Psychoprophylaxis D. Sacral pressure

ANS: D Rationale: The use of sacral pressure may provide relief for patients who are experiencing back labor. The presentation of the fetus in a posterior position indicates this. Neuromuscular dissociation is used as a conditioned response to affect pain relief based on the mother tensing one group of muscles and focusing on releasing tension in the rest of her body. Effleurage is the process of using circular massage to effect pain relief. Psychoprophylaxis is another name for the Lamaze method of prepared childbirth.

A pregnant woman notices that she is beginning to develop dark skin patches on her face. She denies using any different type of facial products as a cleansing solution or makeup. What would the priority nursing intervention be in response to this situation? A. Refer the patient to a dermatologist for further examination. B. Ask the patient if she has been eating different types of foods. C. Take a culture swab and send to the lab for culture and sensitivity (C&S). D. Let the patient know that this is a common finding that occurs during pregnancy.

ANS: D Rationale: This condition is known as chloasma or melasma (mask of pregnancy) and is a result of pigmentation changes relative to hormones. It can be exacerbated by exposure to the sun. There is no need to refer to a dermatologist. Intake of foods is not associated with exacerbation of this process. There is no need for a C&S to be taken. The patient should be assured that this is a normal finding of pregnancy.

Determine the obstetric history of a patient in her fifth pregnancy who has had twospontaneous abortions in the first trimester, one infant at 32 weeks' gestation, and one infant at 38 weeks' gestation. a. G5T1P2A2L2 b. G5T1P1A1L2 c. G5T0P2A2L2 d. G5T1P1A2L2

ANS: D Rationale: This patient is in her fifth pregnancy, which is G5, she had one viable term infant (between 38and 42 weeks' gestation), which is T1, she had one viable preterm infant (between 20 and 37 weeks' gestation), which is P1, two spontaneous abortions (before 20 weeks' gestation),which is A2, and she has two living children, which is L2.

What is the rationale for a woman in her first trimester of pregnancy to expect to visit her health care provider every 4 weeks? A. Problems can be eliminated. B. She develops trust in the health care team. C. Her questions about labor can be answered. D. The conditions of the expectant mother and fetus can be monitored.

ANS: D Rationale: This routine allows for monitoring maternal health and fetal growth and ensures that problems will be identified early. All problems cannot be eliminated because of prenatal visits; however, they can be identified early. Developing a trusting relationship should be established during these visits, but that is not the primary reason. Most women do not have questions concerning labor until the last trimester of the pregnancy.

While providing education to a primiparous patient regarding the normal changes of pregnancy, what is an important information for the nurse to share regarding Braxton Hicks contractions? A. These contractions may indicate preterm labor. B. These are contractions that never cause any discomfort. C. Braxton Hicks contractions only start during the third trimester. D. These occur throughout pregnancy, but you may not feel them until the third trimester.

ANS: D Rationale: Throughout pregnancy, the uterus undergoes irregular contractions called Braxton Hicks contractions. During the first two trimesters, the contractions are infrequent and usually not felt by the woman until the third trimester. Braxton Hicks contractions do not indicate preterm labor. Braxton Hicks contractions can cause some discomfort, especially in the third trimester. Braxton Hicks contractions occur throughout the whole pregnancy.

To determine cultural influences on a patient's diet, what is the nurse's primary action? A. Evaluate the patient's weight gain during pregnancy. B. Assess the socioeconomic status of the patient. C. Discuss the four food groups with the patient. D. Identify the food preferences and methods of food preparation common to thepatient's culture.

ANS: D Rationale: Understanding the patient's food preferences and how she prepares food will assist the nurse in determining whether the patient's culture is adversely affecting her nutritional intake. Evaluating a patient's weight gain during pregnancy should be included for all patients, not just for those who are culturally different. The socioeconomic status of the patients may alter the nutritional intake, but not the cultural influence. Teaching the food groups to the patient should come after assessing food preferences.

A nurse documents that the fetal heart rate variability is marked. This indicates that the range is greater than how many beats per minute? Record your answer as a whole number. _____ bpm

ANS:25 RATIONALE: There are four categories of fetal heart rate variability: Absent: Amplitude range is visually undetectableMinimal: Detectable to less than or equal to 5 beats/minuteModerate (normal): 6 to 25 beats/minute Marked: Range >25 beats/minute

The capacity of the uterus in a term pregnancy is how many times its prepregnant capacity? Record your answer as a whole number. ______ times

ANS:500 Rationale: The prepregnant capacity of the uterus is about 10 mL, and it reaches 5000 mL (5 L) by the end of the pregnancy, which reflects a 500-fold increase.

Calculate the estimated date of birth (EDD) in October using Nägele's rule for a patient whoselast normal menstrual period (LNMP) began on January 1. Record your answer as a whole number. _______

ANS:8 Nägele's rule is often used to establish the EDD. This method involves subtracting 3 monthsfrom the date that the LNMP began, adding 7 days, and then correcting the year, if appropriate. Subtracting 3 months from January 1 gives you the month of October and adding 7 days = October 8.

57. The nurse is caring for a laboring client with sickle cell anemia. Which therapy should the nurse anticipate the physician to order? a. Oxygen b. Antibiotics c. Magnesium sulfate d. Antihypertensives

Answer is A. Rationale: Low oxygen concentration causes sickling of RBCs which cannot pass through small arteries & capillaries; they clump together & occlude vessels (sickle cell crisis) Antibiotics, mag sulfate, & antihypertensives are not indicated r/t sickle cell

9. During a nonstress test (NST), the electronic tracing displays a relatively flat line for fetal movement, making it difficult to evaluate the fetal heart rate (FHR). To mark the strip, the nurse in charge should instruct the client to push the control button at which time? a. At the beginning of each fetal movement b. At the beginning of each contraction c. After every three fetal movements d. At the end of fetal movement

Answer is A. Rationale: The test is intended to monitor accelerations or decelerations related to fetal movement. Examiner needs to know when the movement began.

7. A patient is in her last trimester of pregnancy. Nurse Jane should instruct the patient to notify her primary health care provider immediately if she notices: a. Blurred vision b. Hemorrhoids c. Increased vaginal mucus d. Shortness of breath on exertion

Answer is A. Rationale: Blurred vision is an indication of preeclampsia which requires immediate attention. Hemorrhoids, increased vaginal mucus, & SOB are normal findings in the 3rdtrimester of pregnancy.

53. The nurse is teaching a prenatal client about chorionic villussampling. The nurse correctly teaches the client that the procedure carries which of the following advantages? a. Diagnosis of genetic abnormalities as early as 10-12 weeks' gestation b. Determination of fetal lung maturity c. Diagnosis on the basis on a maternal blood test d. Evaluation of uterine muscle tone

Answer is A. Rationale: CVS may be performed by transcervical or transabdominal approach between 10 & 12 weeksgestation. Fetal lung maturity would be determined close to term with L/S ratio. Maternal serum AFP is screening test only, not diagnostic.

69. A nurse is teaching a group of first trimester prenatal clients about the discomforts of pregnancy. A client asks the nurse, "What causes my nausea and vomiting?" The nurse knows that _________________ is the primary contributing factor to first trimester emesis. a. human chorionic gonadotropin b. estrogen c. progesterone d. prostaglandins

Answer is A. Rationale: HCG is thought to be the primary cause for nausea & vomiting during pregnancy. Estrogen & progesterone may to a lesser extent cause N/V. Prostaglandins are associated cervical ripening @ term & onset of labor.

15. When administering magnesium sulfate to a client with preeclampsia, the nurse understands that this drug is given to: a. Prevent seizures b. Reduce blood pressure c. Slow the process of labor d. Increase diuresis

Answer is A. Rationale: Mag sulfate is a central nervous system depressant & is administered to preeclamptic patients to decrease the risk of seizures. The drug will reduce BP & slow process of labor, but that is not the rationale for use in preeclampsia. An important nursing intervention is to monitoring I & O, as the drug actually decreasesurinary output.

67. The nurse is caring for a prenatal client at 38 weeks' gestation whose ultrasound reveals approximately 3000 mLof amniotic fluid. She complains of shortness of breath and has 2+ pitting edema in her lower extremities. The nurse anticipates preparation for: a. delivery by cesarean. b. amniocentesis. c. intravenous antibiotics. d. amnioinfusion.

Answer is A. Rationale: Term pregnancy; anticipate delivery by Cesarean section. Symptoms indicate possible maternal cardiovascular problems. Too late for amnioto diagnosis fetal anomalies. No indication of infection to warrant abx. Amnioinfusionused in cases of meconiumstaining or possibly cord compression.

39. The posttermneonate with meconium-stained amniotic fluid needs care designed to especially monitor for which of the following? a. Respiratory problems b. Gastrointestinal problems c. Integumentary problems d. Elimination problems

Answer is A. Rationale: The primary concern with meconium staining is the risk for aspiration & pneumonia. Other systems are not affected.

41. A client at 8 weeks' gestation calls complaining of slight nausea in the morning hours. Which of the following client interventions should the nurse question? a. Taking 1 teaspoon of bicarbonate of soda in an 8-ounce glass of water PRN b. Eating a few low-sodium crackers before getting out of bed c. Avoiding the intake of liquids with meals d. Eating six small meals a day instead of thee large meals

Answer is A. Rationale: Use of sodium bicarbonate may lead to electrolyte imbalances. All other interventions are appropriate for hyperemesis gravidarum.

31.When uterine rupture occurs, which of the following would be the priority? a. Limiting hypovolemic shock b. Obtaining blood specimens c. Instituting complete bed rest d. Inserting a urinary catheter

Answer is A. Rationale: Uterine rupture results in hemorrhage which can lead to hypovolemic shock. Prevention of shock is best; earlier the intervention, better the outcome. Blood specimens would be obtained, butis not a priority. Bed rest is not relevant @ this time. A urinary catheter would likely be inserted to monitor I & O which can be an indicator of shock, butis not the priority.

Which of the following would the nurse most likely expect to find when assessing a pregnant client with abruptio placentae? a. Excessive vaginal bleeding b. Rigid, boardlikeabdomen c. Tetanic uterine contractions d. Premature rupture of membranes

Answer is B. Rational: The most common assessment finding in a client with abruption placenta is a rigid or boardlikeabdomen. Pain, usually reported as a sharp stabbing sensation high in the uterine funduswith the initial separation, also is common.

6. The nurse is developing a teaching plan for a patient who is 8 weeks pregnant. The nurse should tell the patient that she can expect to feel the fetus move at which time? a. Between10 and 12 weeks' gestation b. Between16 and 20 weeks' gestation c. Between21 and 23 weeks' gestation d. Between24 and 26 weeks' gestation

Answer is B. Rationale: A pregnant woman usually can detect fetal movement (quickening) between 16 and 20 weeks'gestation. Before 16 weeks, the fetus is not developed enough for the woman to detect movement. After 20 weeks, the fetus continues to gain weight steadily, the lungs start to produce surfactant, the brain is grossly formed, and myelination of the spinal cord begins.

13. A 39-year-old at 37 weeks' gestation is admitted to the hospital with complaints of abdominal pain & vaginal bleeding following the use of cocaine 1 hour earlier. Which complication is most likely causing the client's complaints? a. Placenta previa b. Abruptio placentae c. Ectopic pregnancy d. Spontaneous abortion

Answer is B. Rationale: Abruption often caused by severe vasoconstriction (cocaine) or abdominal trauma. Placenta previa usually involves painless bleeding. Too late in pregnancy for abortion or ectopic pregnancy.

16. What's the approximate time from fertilization to uterine implantation? a. 2-5 days b. 7-10 days c. 12-16 days d. 14 weeks

Answer is B. Rationale: Approximately 3-5 days after fertilization, the dividing embryo enters the uterine cavity. Full implantation is complete approximately 7-10 days after fertilization.

A client at 36 weeks' gestation is schedule for a routine ultrasound prior to an amniocentesis. After teaching the client about the purpose for the ultrasound, which of the following client statements would indicate to the nurse in charge that the client needs further instruction? a.The ultrasound will help to locate the placenta b. The ultrasound identifies blood flow through the umbilical cord c. The test will determine where to insert the needle d. The US will locate pools of amniotic fluid

Answer is B. Rationale: Before amniocentesis, a routine ultrasound is valuable in locating the placenta, locating a pool of amniotic fluid, and showing the physician where to insert the needle. Color Doppler imaging ultrasonographyidentifies blood flow through the umbilical cord. A routine ultrasound does not accomplish this.

10. When evaluating a client's knowledge of symptoms to report during her pregnancy, which statement would indicate to the nurse in charge that the client understands the information given to her? a. "I'll report increased frequency of urination." b. "If I have blurred or double vision, I should call the clinic immediately." c. "If I feel tired after resting, I should report it immediately." d. "Nausea should be reported immediately."

Answer is B. Rationale: Blurred or double vision is symptom of preeclampsia. Increased urination, fatigue, & nausea are expected findings (unless excessive.)

8. The nurse in charge is reviewing a patient's prenatal history. Which finding indicates a genetic risk factor? The patient: a. Is 15 years old b. has a child with cystic fibrosis c. was exposed to rubella at 36 weeks' gestation d. has a history of preterm labor at 32 weeks' gestation

Answer is B. Rationale: CF is a chronic progressive autosomal recessive genetic disease which affects the respiratory, digestive & endocrine systems. Infant must inherit two defective CF genes- one from each parent. If two carriers of CF gene conceive = 25% chance infant will have disease & 50% chance of being a carrier.

50. The nurse is reviewing four prenatal charts. Which client would be an appropriate candidate for a contraction stress test (CST)? A client with: a. hx of PTL b. multiple gestation c. an incompetent cervix d. placenta previa

Answer is B. Rationale: CST might be appropriate for a multiple gestation pregnancy. CST is contraindicated in women who have had or are @ risk for preterm labor. Also contraindicated with PROM, extensive uterine surgery or classical uterine incision, & placenta previa.

42. The nurse documents positive ballottement in the patient's prenatal record. The nurse understands that this indicates which of the following? a. Palpable contractions on the abdomen b. Passive movement of the unengaged fetus c. Fetal kicking felt by the patient d. Enlargement and softening of the uterus

Answer is B. Rationale: Examiner taps the fetus transcervically& it rebounds against the tap (like a balloon filled with air). Occurs if fetus is unengaged. Fetal movement felt by the patient is quickening. Softening of the lower uterine segment is Hegar'ssign.

36. A newborn who has an asymmetrical Moro reflex response should be further assessed for which of the following? a. Talipes equinovarus b. Fractured clavicle c. Congenital hypothyroidism d. Increased intracranial pressure

Answer is B. Rationale: Fractured clavicle will impair movement of the upper extremity on the affected side. Talipes equinovarus is a combination of downward & inward fixed position of flexion of the feet (clubfoot). Congenital hypothyroidism likely asymptomatic @ birth. Neonatal S/S of increased ICP include drowsiness, separated sutures, bulging fontanelle, & vomiting.

28. Which of the following factors would the nurse suspect as predisposing a client to placenta previa? a. Multiple gestation b. Uterine anomalies c. Abdominal trauma

Answer is B. Rationale: Misshaped uterus may lend toward abnormal implantation site. Other risk factors include previous cesarean birth, previous placenta previa, previous pregnancy termination, multiparity, AMA, smoking & cocaine use. Additionally may be r/t a short interval between pregnancies, living at a high altitude & Asian ethnicity (86% increased risk). d. Renal or vascular disease

38. A newborn weighing 3000 grams and feeding every 4 hours needs 120 calories/kg of body weight every 24 hours for proper growth and development. How many ounces of 20 cal/oz formula should this newborn receive at each feeding to meet nutritional needs? a. 2 ounces b. 3 ounces c. 4 ounces d. 6 ounces

Answer is B. Rationale: Neonate weighs 3 kg. x 120 kcals = 360 kcals every 24 hours. He feeds every 4 hours (6 times in 24 hours) 360 divided by 6 = 60 kcals/feeding Formula delivers 20 kcals/oz 3 ozs. X 20 kcals = 60 kcals

54.A prenatal client at 30 weeks' gestation is scheduled for an amniocentesis to determine fetal lung maturity. The nurse expects the lecithin/sphingomyelin(L/S) ratio to be: a. 5:1 b. 1:1 c. 2:1 d. 3:1

Answer is C. Rationale: L/S ratio of 2:1 generally indicates adequate surfactant & fetal lung maturity. Lecithin is a phospholipid component of fetal lung fluid & surfactant Sphingomyelin- general amniotic membrane lipid Present in equal amounts until about 30 weeks gestation, then only lecithin increases

18. When preparing a teaching plan for a client who is to receive a rubella vaccine during the postpartum period, the nurse in charge should include which of the following? a. The vaccine prevents a future fetus from developing congenital anomalies b. Pregnancy should be avoided for 3 months after the immunization c. The client should avoid contact with children diagnosed with rubella d. The injection will provide immunity against the 7-day measles

Answer is B. Rationale: Postpartum patient will need to sign consent acknowledging requirement to prevent pregnancy for 3 months, as CRS could occur from vaccine. No vaccine is 100% nor can guarantee absence of congenital defects. No rationale for avoiding children with rubella postpartum; only during pregnancy. Rubella vaccine only protects against rubella.

30. Which of the following would the nurse assess in a client experiencing abruptio placentae? a. Bright red, painless vaginal bleeding b. Concealed or external dark red bleeding c. Palpable fetal outline d. Soft and nontenderabdomen

Answer is B. Rationale: S/S of an abruption include rigid, board-like abdomen, & concealed or obvious dark red bleeding. Bright red, painless bleeding is r/t placenta previa. Palpable fetal outline is expected by performing Leopold's maneuvers.

40. A client with severe preeclampsia is admitted with of BP 160/110, proteinuria, and severe pitting edema. Which of the following would be most important to include in the client's plan of care? a. Daily weights b. Seizure precautions c. Right lateral positioning d. Stress reduction

Answer is B. Rationale: Seizure precautions are priority. Daily weights are good indication of fluid volume status & weight gain or loss. Right or left lateral positioning prevents aortocaval syndrome. Stress reduction is always good!

27. Which of the following would the nurse identify as a classic sign of PIH? a. Edema of the feet and ankles b. Edema of the hands and face c. Weight gain of 1 lb/week d. Early morning headache

Answer is B. Rationale: Swelling of the hands & face is a classic symptom of preeclampsia. Swelling of the feet & ankles is r/t poor circulation due to the weight of the uterus on vessels in the lower extremities. Weight gain of 0.88 to 1 lb. per week is normal in the 3rdtrimester. Headaches may be associated with preeclampsia; not necessarily just in the morning.

63.The patient with an incompetent cervix has been advised regarding treatment for her condition. This would involve: a. D & C b. Cerclage c. Complete bed rest d. Magnesium Sulfate

Answer is B. Rationale: Treatment for incompetent cervix includes cerclage, which is a pursestringsuture placed in the cervix to prevent dilation prior to term. The suture is removed when fetus is matured & ready for delivery. D & C would abort the pregnancy. Bedrest might help, butwill not guarantee a continued pregnancy. Mag Sulfate is not even related to the problem.

44. Which of the following would be the nurse's most appropriate response to a client who asks why she must have a cesarean delivery if she has a complete placenta previa? a. "You will have to ask your physician when he returns." b. "You need a cesarean to prevent hemorrhage." c. "The placenta is covering most of your cervix." d. "The placenta is covering the opening of the uterus and blocking your baby's delivery."

Answer is B. Rationale: Vaginal delivery is not an option with complete placenta previa due to hemorrhage & placental separation resulting in death of neonate. Passing the buck to the physician increases patient anxiety. Telling the patient that the placenta covers the cervix may not be in terms she understands. The condition doesn't "block fetal exit"- see risks in answer B.

59. The nurse is reviewing the lab tests of four prenatal clients. Which lab finding would support the diagnosis of hyperemesis gravidarum? a. Hypercalcemia b. Hyperkalemia c. Hypokalemia d. Hypocalcemia

Answer is C. Rationale: Hypokalemiais associated with loss of fluid from upper GI tract r/t vomiting or gastric suction, diuretic use Hypocalcemiais r/t low albumin levels, , pancreatitis, hypoparathyroidism, renal failure, & severe malnutrition Hypercalcemiais r/t hyperparathyroidism, malignancies, metastatic bone disease, immobilization, & thiazidediuretics Hyperkalemiais r/t renal failure, rapid infusion of K+ replacement

62. Your patient in her fourth month of her pregnancy has just been told she has an incompetent cervix. What is the best way to explain this to her? a. You have a severe cervical infection. b. The cervix will not open. c. The cervix is opening prematurely. d. You have a bleeding area on the cervix.

Answer is C. Rationale: Incompetent cervix refers to an inability of the to remain closed long enough during pregnancy for sufficientfetal maturity.

60. A client at 36 weeks' gestation is admitted to the labor and birth unit. Her chief complaint is abdominal cramping with a sudden gush of clear fluid. What is the priority nursing diagnosis for this prenatal client? a. Risk for ineffective coping related to unknown outcome of pregnancy. b. Knowledge deficit related to unfamiliarity with loss of vaginal fluids. c. Risk for infection related to premature rupture of membranes. d. Impaired physical mobility related to strict bed rest.

Answer is C. Rationale: Priority is concern for potential infection due to PROM. Ineffective coping r/t unknown outcome may be an issue. No evidence of knowledge deficit; she came to the facility with c/o fluid leakage. BR has not been ordered; may decide to deliver.

64. A prenatal nurse is assessing a client at 34 weeks' gestation who complains of watery vaginal discharge. What should be the nurse's initial action? a. Prepare for a non-stress test. b. Obtain vaginal cultures for STIs. c. Test the fluid with nitrazinepaper. d. Test the urine for bacteria.

Answer is C. Rationale: Priority is to R/O PROM by nitrazinepaper test; if positive, next action would be to R/O prolapsed cord If negative might check for UTI or STIs NST not really indicated- more concerned about fetal lung maturity (especially if male fetus)

A patient with pregnancy-induced hypertension (preeclampsia) probably exhibits which of the following symptoms? a. Proteinuria, headaches, vaginal bleeding b. Headaches, double vision, vaginal bleeding c. Proteinuria, headaches, double vision d. Glycosuria, double vision, facial swelling, & uterine contractions

Answer is C. Rationale: A patient with PIH complains of headache, double vision, facial/finger swelling and sudden weight gain. A urine specimen reveals proteinuria. Vaginal bleeding and uterine contractions are not associated with this condition.

33. A patient at 24 weeks gestation has gained 6 pounds in 4 weeks. Which of the following would be the priority assessment? a. Glucosuria b. Depression c. Hand/face edema d. Dietary intake

Answer is C. Rationale: Based on the excessive weight gain, preeclampsia is a possibility, & the priority consideration. Assess for other s/s including hand/facial edema, blurred vision, headache, proteinuria, elevated BP. Glucosuria may indicate the renal threshold has been exceeded & she is now diabetic. Depression (overeating) & dietary intake may relate to excessive weight gain.

52. A prenatal client at 17 weeks' gestation is scheduled for an alpha- fetoprotein test and asks the nurse, "What is this test for?" The nurse correctly responds that the test is used to determine: a. lung maturity. b. placental function c. down's syndrome. d. hydrops fetalis.

Answer is C. Rationale: Low levels of MAFP may indicate Down's syndrome. (High levels may indicate neural tube defect.) Lung maturity is indicated by L/S ratio more toward term. (This patient is only 17 wks.) Placental function is indicated by NST or CST. Hydrops fetalis is a severe condition that is r/t Rh incompatibility & results in generalized fetal edema & heart failure. Rh incompatibility is diagnosed during pregnancy with an indirect Coombs'test.

49. The physician orders an ultrasound for a prenatal client prior to an amniocentesis. The nurse explains to the client that the purpose of the ultrasound is to: a. determine the gestational sac volume. b. measure the fetus' crown-rump length. c. locate the placenta. d. measure the fetus' biparietal diameter.

Answer is C. Rationale: Prior to an amnio, US is used to locate the placenta & pools of amniotic fluid, & to avoid critical structures when inserting the needle. Determining gestational sac volume, crown-rump length, & biparietal diameter is done during routine ultrasound.

35. The nurse assesses the vital signs of a client, 4 hours' postpartum that are as follows: BP 90/60; temperature 100.4ºF; pulse 100 weak, thready; R 20 per minute. Which of the following should the nurse do first? a. Report the temperature to the physician b. Recheck the blood pressure with another cuff c. Assess the uterus for firmness and position d. Determine the amount of lochia

Answer is C. Rationale: The VS indicate patient may be bleeding. The likely etiology is a soft, boggy fundus. Assess & massage until firm. The temp could be normal, based on recent delivery. Blood pressure should have been assessed with appropriate cuff initially; pulse of 100, weak & threadystill indicate bleeding. Amount of lochia may be relevant, but the fundal assessment is priority.

12. The patient presents to the clinic with hxof a positive pregnancy test two weeks ago. She is now c/o dizziness, vaginal spotting, & abdominal pain which radiates to her right shoulder. You recognize the need for emergent evaluation as she likely has a(n): a. abruptioplacentae b. placentaprevia c. ectopic pregnancy d. missed abortion

Answer is C. Rationale: Too early for abruption or previa. Classic symptom of ectopic pregnancy is pain radiating to shoulder which is indicative of tubal rupture. Pain is caused by internal bleeding that irritates the diaphragm.

25. A pregnant client states that she "waddles" when she walks. The nurse's explanation is based on which of the following as the cause? a. The large size of the newborn b. Pressure on the pelvic muscles c. Relaxation of the pelvic joints d. Excessive weight gain

Answer is C. Rationale: Waddling is due to relaxation of the pelvic joints caused by the presence of the hormone, relaxin.

46. A client at 28 weeks' gestation is admitted to the labor and birth unit. Which test would be used to assess the client's fetal status? a. Ultrasound for physical structure b. Contraction stress test (CST) c. Biophysical profile (BPP) d. Amniocentesis

Answer is C. Rationale: Best indicator would be biophysical profile as this includes NST & US. Too early to perform CST. Too late for amniocentesis.

19. A patient with eclampsia begins to experience a seizure. Which of the following would the nurse in charge do first? a. Pad the side rails b. Place a pillow under the left buttock c. Insert a padded tongue blade into the mouth d. Maintain a patent airway

Answer is D. Rationale: •Airway is always a priority •Tongue blades are no longer used •Patient would be turned on her side to prevent aspiration in the event of emesis •Side rails should be padded for any patient likely to experience seizure activity, but are not a priority during the event

23. A pregnant woman states that she frequently ingests laundry starch. When assessing the client, for which of the following should the nurse be alert? a. Muscle spasms. b. Lactose intolerance. c. Diabetes mellitus. d. Anemia.

Answer is D. Rationale: All pregnant clients should be screened for pica, or the ingestion of nonfood substances, such as clay, dirt, or laundry starch. Commonly, clients who practice pica are anemic.

24. The nurse enlists the aid of an interpreter when caring for a primiparousclient from Mexico who speaks very little English and delivered a viable term neonate 8 hours ago. When developing the postpartum dietary plan of care for the client, the nurse would encourage the client's intake of which of the following? a. Tomatoes. b. Potatoes. c. Corn products. d. Meat products.

Answer is D. Rationale: Because the diet of Hispanic Americans from Mexico and Central America commonly includes beans, corn products, tomatoes, chili peppers, potatoes, milk, cheeses, and eggs, the nurse needs to encourage an intake of meats, dark green leafy vegetables, and other high-protein products that are rich in iron. Doing so helps to compensate for the significant blood loss and subsequent iron loss that occurs during the postpartum period. Additionally, fresh fruits, meats, and green leafy vegetables may be scarce, possibly resulting in deficiencies of vitamin A, vitamin D, and iron.

65. The nurse is assessing a prenatal client at 12 weeks' gestation who was admitted to the hospital with complaints of severe nausea and vomiting, & elevated alpha- fetoprotein. What diagnosis should the nurse anticipate? a. Abruptio placentae b. Oligohydramnios c. Placenta previa d. Multiple pregnancy

Answer is D. Rationale: Increased N/V & elevated MAFP are associated with multiples pregnancy. Oligohydramnios would not affect these factors. Abruptio placentae would be associated a rigid, board-like abdomen & possibly dark red vaginal bleeding. S/S of placenta previa would include painless, bright red vaginal bleeding.

66. The nurse is performing a pelvic exam on a laboring client and discovers a loop of cord in the vagina. What is the initial nursing action? a. Administer oxygen at 5 L per minute. b. Call the physician or nurse-midwife. c. Place client in a side-lying position. d. Leave gloved fingers in the vagina.

Answer is D. Rationale: Problem is prolapsed cord. Keep the fingers in the vagina & attempt to keep the presenting part off the cord. Other action would be to put patient in knee-chest position.

55. The nurse is caring for a laboring client with Type I diabetes. What are the signs and symptoms of hypoglycemia for which the nurse should assess? a. Headache and anorexia b. Dry skin and blurred vision c. Frequent urination and headache d. Diaphoresis and disorientation

Answer is D. Rationale: S/S of hypoglycemia include: shakiness, sweating, pallor, cold/clammy skin, disorientation, irritability, headache, hunger, blurred vision S/S of hyperglycemia include: fatigue, flushed/hot skin, dry mouth, excessive thirst, frequent urination, rapid/deep respirations, fruity odor to breath (ketones), drowsiness, headache, depressed reflexes

22. After instructing a primigravidaabout desired weight gain during pregnancy, the nurse determines that the teaching has been successful when the client states which of the following? a. "A total weight gain of approximately 20 pounds is recommended." b. "A weight gain of 6.6 pounds in the second and third trimester is considered normal." c. "A weight gain of about 12 pounds every trimester is recommended." d. "Although it varies, a gain of 25 to 35 pounds is about average."

Answer is D. Rationale: The National Academy of Sciences Institute of Medicine recommends that women gain between 25 and 35 poundsduring pregnancy. Typically, women should gain 3.5 pounds during the first trimester and then 1 pound/week during the remainder of the pregnancy (24 weeks) for a total of about 27 to 28 pounds.

51. A prenatal client at 22 weeks' gestation is scheduled for an amniocentesis. What would be an appropriate nursing action to prepare this client for the procedure? a. Cleanse the client's abdominal skin with alcohol. b. AdministerRhimmune globulin to the client. c. Encourage the client to take fluids. d. Position the client in a left lateral tilt.

Answer is D. Rationale: Lateral tilt would help prevent compression of aorta & inferior vena cava. Abdomen would likely be cleaned with betadinesolution, unless allergy exists. Rhogamwould only be administered if the amnioresulted in an abortion. (Routinely administered @ 28 wks. Gestation & within 72 hours of delivery or abortion.) Taking PO fluids has no relation to procedure.

45. A patient is in labor and has just been told she has a breech presentation. The nurse should be particularly alert for which of the following? a. Lightening b. Ophthalmia neonatorum c. Pica d. Prolapsed umbilical cord

Answer is D. Rationale: If the fetus is not a cephalic presentation, increased risk for prolapsed cord. Lightening occurs before labor in the primigravida& during labor in the multigravida. This refers to the moving down of the presenting part into the pelvis. Ophthalmianeonatorumis a fetal eye infection caused by gonorrhea. Pica would occur during the pregnancy.

48. The nurse is preparing a prenatal client for a transvaginal ultrasound. What nursing action should the nurse include in the preparations? a. Advise the client not to empty her bladder. b. Encourage the client to drink 1.5 quarts of fluid. c. Apply transmission gel over the client's abdomen. d. Place client in lithotomyposition.

Answer is D. Rationale: Lithotomy position facilitates the procedure. A full bladder & gel to the abdomen is needed only for an abdominal ultrasound.

34. The patient who is 5 weeks pregnant has just had her first ultrasound & is informed that it is an ectopic pregnancy. Which of the following would be the priority nursing diagnosis? a. Risk for infection b. Chronic Pain c. Knowledge Deficit d. Anticipatory Grieving

Answer is D. Rationale: The patient now knows that this pregnancy will need to be aborted, so she will anticipate grieving the loss. Infection is not a priority at this time. Chronic pain is defined as that which lasts for 6 months or longer.

When caring for a 3-day-old neonate who is receiving phototherapy to treat jaundice, the nurse in charge would expect to do which of the following? a. Turn the neonate every 6 hours b. Encourage the mother to discontinue breastfeeding c. Notify the physician if the skin becomes bronze in color d. Check the vital signs every 2 to 4 hours

Answer is D. Rationale: Neonate should be turned every 2 hours to expose all body parts & promoting breakdown of bilirubin, preventing pressure areas, & providing stimulation. Breastfeeding is encouraged to maintain adequate fluid status. (If bililevels > 15 mg/dL, physician may order temporary cessation of breastfeeding as this may increase jaundice) Phototherapy may cause minor transient rash, but not bronzing. Body temperature is major concern.

37. Immediately after birth the nurse notes the following on a male newborn: respirations 78; apical heart rate 160 BPM, nostril flaring; mild intercostal retractions; and grunting at the end of expiration. Which of the following should the nurse do? a. Call the assessment data to the physician's attention b. Start oxygen per nasal cannulaat 2 L/min. c. Suction the infant's mouth and nares d. Recognize this as normal first period of reactivity

Answer is D. Rationale: S/S expected during initial period of reactivity is characterized by a fully alert & active neonate. They have a strong desire to suck. Adaptation to extrauterinelife may involve tachypnea(up to 80 breaths per minute), tachycardia (up to 180 BPM), mild to moderate chest wall retractions, nasal flaring, periodic breathing (pauses of < 15 seconds), expiratory grunting, & acrocyanosis. May need Oxygen- deliver by hood, not NC. Physician would need to be notified if condition not resolved.

70. A nurse is researching the topic of edema during pregnancy. Which of the following contributes to fluid retention? a. Increased serum protein. b. Decreased intracapillary pressure and permeability. c. Decreased nitrogen retention. d. Increased levels of steroid sex hormones.

Answer is D. Rationale: Steroid hormones secreted by the placenta include estrogen & progesterone. All steroids cause edema. Decreased intracapillary pressure & permeability actually reducerisk of edema. Increased serum protein assists in holding fluid in the vascular space. Nitrogen is not relevant.

21. Which foods would supply your pregnant patient with the best sources of iron? a. Fish, chicken, and lean beef. b. Broccoli, spinach, and fish. c. Eggs, legumes, and dried fruit. d. Lean beef, organ meats, and dried fruits.

Answer is D. Rationale: The best sources of iron include lean beef, organ meats, and dried fruit.

68. A nurse is receiving shift reports on four patients in the birthing unit. In which of the following is the physician most likely to induce labor? A patient at: a. 39 weeks' gestation with a transverse fetal lie b. 41 weeks' gestation with an active vaginal herpes infection c. 38 weeks' gestation with a prolapsed umbilical cord d. 40 weeks' gestation with gestational hypertension

Answer is D. Rationale: The only patient that would qualify for induction would be individual with gestational HTN. Contraindications to induction include transverse lie, active herpetic infection, & a prolapsed umbilical cord.

A client is concerned about gaining weight during pregnancy. What should the nurse tell the client is the cause of the greatest weight gain during pregnancy? 1. Fetal growth 2. Fluid retention 3. Metabolic alterations 4. Increased blood volume

Correct 1. Fetal growth

11. During a prenatal visit at 4 months gestation, a pregnant client asks whether tests can be done to identify fetal abnormalities. Between 18 and 40 weeks' gestation, which procedure is routinely used to detect fetal anomalies? a. Amniocentesis. b. Chorionic villisampling. c. Fetotomy. d. Ultrasound.

Answer is D. Rationale: Ultrasound is safely performed throughout pregnancy. CVS is performed @ 10-12 weeks. Amniois usually performed @ 15-20 weeks to determine defects, butmay be done up to 40 weeks to determine fetal lung maturity (L/S ratio).

What's the term for: Low blood oxygen and high blood and tissue carbon dioxide levels

Asphyxia

What's the term for: Less than normal muscle tone

Atony

What's the term for: Stimulation to improve effectiveness of spontaneous labor contractions

Augmentation

A nurse is teaching breast care to a client who is breastfeeding. Which statement by the client indicates that the teaching has been effective? 1. "I should air-dry my nipples after each feeding." 2. "I should use a mild soap when I wash my breasts." 3. "I'll have to line my breast pads with plastic shields." 4. "I need to take off my bra before I go to bed at night."

Correct 1. "I should air-dry my nipples after each feeding."

A nurse is teaching clients how to determine the estimated time of ovulation by taking their basal body temperature. What is the expected change in the basal temperature during ovulation? 1. A slight drop followed by an increase 2. A sudden rise followed by a decrease 3. A marked increase after which the temperature remains high 4. A marked decrease after which the temperature remains lower

Correct 1. A slight drop followed by an increase

A woman in active labor arrives at the birthing unit. She tells the nurse that she was told that she had a chlamydial infection the last time she visited the clinic; however, she stopped taking the antibiotic after 3 days because she "felt better." In light of this history what would the nurse anticipate as part of the plan of care? 1. Administration of antibiotics before delivery 2. Oxytocin infusion to augment labor 3. Epidural anesthesia to relieve difficult labor discomfort 4. Magnesium sulfate infusion to prevent a precipitous birth

Correct 1. Administration of antibiotics before delivery

A client in her tenth week of pregnancy exhibits presumptive signs of pregnancy. Which clinical findings may the nurse determine upon assessment? Select all that apply. 1. Amenorrhea 2. Breast changes 3. Urinary frequency 4. Abdominal enlargement 5. Positive urine pregnancy test

Correct 1. Amenorrhea 2. Breast changes 3. Urinary frequency

A nurse is caring for a client in preterm labor who is receiving a course of corticosteroids to promote fetal lung maturity. What test may be used to most accurately determine fetal lung maturity? 1. Amniocentesis 2. Ultrasonography 3. Radioreceptor assay 4. Chorionic villus sampling

Correct 1. Amniocentesis

A nurse in the prenatal clinic is assessing a woman at 34 weeks' gestation. The client's blood pressure is 166/100 mm Hg and her urine is +3 for protein. She states that she has a severe headache and occasional blurred vision. Her baseline blood pressure was 100/62 mm Hg. What is the priority nursing action? 1. Arranging transportation to the hospital 2. Obtaining a prescription for an antihypertensive 3. Rechecking the blood pressure within 30 minutes 4. Obtaining a prescription for acetaminophen to relieve the headache

Correct 1. Arranging transportation to the hospital

On the second postpartum day a client mentions that her nipples are becoming sore from breastfeeding. What is the nurse's initial action in response to this information? 1. Assess her breastfeeding techniques to identify possible causes. 2. Provide a nipple shield to keep the infant's mouth off the nipples. 3. Instruct her to apply warm compresses 10 minutes before she begins to breastfeed. 4. Explain that she should limit breastfeeding to 5 minutes per side until the soreness subsides.

Correct 1. Assess her breastfeeding techniques to identify possible causes.

A client at 43 weeks' gestation has just given birth to an infant with typical postmaturity characteristics. Which signs of postmaturity does the nurse identify? Select all that apply. 1. Cracked and peeling skin 2. Long scalp hair and fingernails 3. Red, puffy appearance of face and neck 4. Vernix caseosa covering the back and buttocks 5. Creases covering the neonate's full soles and palms

Correct 1. Cracked and peeling skin 2. Long scalp hair and fingernails 5. Creases covering the neonate's full soles and palms

A client with mild preeclampsia is admitted to the high-risk prenatal unit because of a progressive increase in her blood pressure. The nurse reviews the primary healthcare provider's prescriptions. Which prescriptions does the nurse expect to receive for this client? Select all that apply. 1. Daily weight 2. Side-lying bed rest 3. 2 g/day sodium diet 4. Deep tendon reflexes 5. Glucose tolerance test

Correct 1. Daily weight 2. Side-lying bed rest 4. Deep tendon reflexes

The laboratory blood tests of a client at 10 weeks' gestation reveal that she has anemia. The client refuses iron supplements. The nurse teaches her that the best source of iron is liver. What other foods does the nurse encourage the client to eat? Select all that apply. 1. Dark leafy green vegetables 2. Legumes 3. Dried fruits 4. Broiled halibut 5. Ground beef patty

Correct 1. Dark leafy green vegetables 2. Legumes 3. Dried fruits 5. Ground beef patty

A man and a woman want to use the calendar (rhythm) method of contraception but do not understand how it works. Based on an average 28-day cycle, during what time frame does the nurse explain that they should refrain from intercourse? 1. Days 10 to 17 of the menstrual cycle 2. 7 days before the end of the menstrual cycle 3. 7 days after completion of the menstrual period 4. 14 days after completion of the menstrual period

Correct 1. Days 10 to 17 of the menstrual cycle

A client with class I heart disease has reached 34 weeks' gestation. Which problem should the nurse anticipate now that the client is in her third trimester? 1. Dyspnea at rest 2. Vasovagal syncope 3. Progressive dependent edema 4. Shortness of breath on exertion

Correct 1. Dyspnea at rest

A client asks the nurse about the use of an intrauterine device (IUD) for contraception. Which information should the nurse include in the response? Select all that apply. 1. Expulsion of the device 2. Occasional dyspareunia 3. Delay of return to fertility 4. Risk for perforation of the uterus 5. Increased number of vaginal infections

Correct 1. Expulsion of the device 2. Occasional dyspareunia 4. Risk for perforation of the uterus

A newborn experiences a hypothermic period while being bathed and having clothing changed. Once the hypothermic episode has been identified and treated, what is the next nursing action? 1. Feeding the infant 2. Requesting a complete blood count 3. Monitoring the infant for hyperthermia 4. Allowing the infant to rest undisturbed

Correct 1. Feeding the infant

A nonstress test is scheduled for a client with preeclampsia. During the nonstress test the nurse concludes that if accelerations of the fetal heart rate occur with fetal movement, this probably indicates what? 1. Fetal well-being 2. Fetal head compression 3. Uteroplacental insufficiency 4. Umbilical cord compression

Correct 1. Fetal well-being

A client is admitted to the birthing unit with uterine tenderness and minimal dark-red vaginal bleeding. She has a marginal abruptio placentae. The priority assessment includes fetal status, vital signs, skin color, and urine output. What additional assessment is essential? 1. Fundal height 2. Obstetric history 3. Time of the last meal 4. Family history of bleeding disorders

Correct 1. Fundal height

A pregnant client is admitted to the high-risk unit with uterine tenderness and some dark-red vaginal bleeding. Abruptio placentae is diagnosed. Which priority assessment should be included with vital signs, skin color, urine output, and fetal heart rate? 1. Fundal height 2. Obstetric history 3. Time of last meal 4. History of bleeding tendencies

Correct 1. Fundal height

A client is admitted to the birthing suite with a blood pressure of 150/90 mm Hg, 3+ proteinuria, and edema of the hands and face. A diagnosis of severe preeclampsia is made. What other clinical findings support this diagnosis? Select all that apply. 1. Headache 2. Constipation 3. Abdominal pain 4. Vaginal bleeding 5. Visual disturbances

Correct 1. Headache 3. Abdominal pain 5. Visual disturbances

A primary healthcare provider prescribes carboprost to be administered to a postpartum client with intractable vaginal bleeding. Which prior condition noted in the client's history should alert the nurse to question this prescription? 1. History of asthma 2. Homan sign 3. Increased blood pressure 4. Absence of the Babinski reflex

Correct 1. History of asthma

A nurse is obtaining the health history from a 21-year-old client who is seeking contraceptive information. What factor in the client's history is a contraindication to the use of oral contraceptives? 1. History of hypertension 2. Pack-a-day cigarette habit 3. Older than 30 years of age 4. Several multiple pregnancies

Correct 1. History of hypertension

A nurse suspects that a newborn has toxoplasmosis, one of the TORCH infections. How and when may it have been transmitted to the newborn? 1. In utero through the placenta 2. In the postpartum period through breast milk 3. During birth through contact with the maternal vagina 4. After the birth through a blood transfusion given to the mother

Correct 1. In utero through the placenta

The nurse is caring for a pregnant client with type 1 diabetes. Which complication is the result of type 1 diabetes? 1. Increased risk of hypertensive states 2. Abnormal placental implantation 3. Excessive weight gain because of increased appetite 4. Decreased amount of amniotic fluid as the pregnancy progresses

Correct 1. Increased risk of hypertensive states

During a home visit the nurse obtains information regarding a postpartum client's behavior and suspects that she is experiencing postpartum depression. Which assessments support this conclusion? Select all that apply. 1. Lethargy 2. Ambivalence 3. Emotional lability 4. Increased appetite 5. Long periods of sleep

Correct 1. Lethargy 2. Ambivalence 3. Emotional lability

The nurse is teaching a prenatal class regarding the risks of smoking during pregnancy. What neonatal consequence of maternal smoking should the nurse include in the teaching? 1. Low birthweight 2. Facial abnormalities 3. Chronic lung problems 4. Hyperglycemic reactions

Correct 1. Low birthweight

What is the priority nursing intervention for the postpartum client whose fundus is three fingerbreadths above the umbilicus, boggy, and midline? 1. Massaging the uterine fundus 2. Helping the client to the bathroom 3. Assessing the peripad for the amount of lochia 4. Administering intramuscular methylergonovine (Methergine) 0.2 mg

Correct 1. Massaging the uterine fundus

In specific situations gloves are used to handle newborns whether or not they are positive for human immunodeficiency virus (HIV). When is it unnecessary for the nurse to wear gloves while caring for a newborn? 1. Offering a feeding 2. Changing the diaper 3. Giving an admission bath 4. Suctioning the nasopharynx

Correct 1. Offering a feeding

The nurse is caring for a new mother who has a chlamydial infection. For which complications should the nurse assess the client's neonate? Select all that apply. 1. Pneumonia 2. Preterm birth 3. Microcephaly 4. Conjunctivitis 5. Congenital cataracts

Correct 1. Pneumonia 2. Preterm birth 4. Conjunctivitis

Why is it important for the nurse to know the infant's gestational age and how it compares with the birth weight? 1. Potential problems may be identified. 2. Infants lose weight during the first few days of life. 3. Hospitals need this information to calculate census. 4. Infants' weight must be included on the admission record.

Correct 1. Potential problems may be identified.

A client with severe preeclampsia in the high-risk unit is receiving an infusion of magnesium sulfate. If eclampsia were to occur, which action would be the highest priority for the nurse to initiate? 1. Prevent injury 2. Assess fetal heart tones 3. Maintain an open airway 4. Increase the infusion rate

Correct 1. Prevent injury

During the assessment of a preterm neonate the nurse determines that the infant is experiencing hypothermia. Which action should the nurse take? 1. Rewarm gradually 2. Notify the practitioner 3. Assess for hyperglycemia 4. Record skin temperature hourly

Correct 1. Rewarm gradually

The nurse assesses a newborn and observes central cyanosis. What type of congenital heart defect usually results in central cyanosis? 1. Shunting of blood from right to left 2. Shunting of blood from left to right 3. Obstruction of blood flow from the left side of the heart 4. Shunting of blood from right to left

Correct 1. Shunting of blood from right to left

The nurse is assessing the newborn of a known opioid user for signs of withdrawal. What clinical manifestations does the nurse expect to identify? Select all that apply. 1. Sneezing 2. Hyperactivity 3. High-pitched cry 4. Exaggerated Moro reflex 5. Reduced deep tendon reflexes

Correct 1. Sneezing 2. Hyperactivity 3. High-pitched cry 4. Exaggerated Moro reflex

Intravenous magnesium sulfate therapy is instituted for a client with severe preeclampsia who has a blood pressure of 170/110 mm Hg, a pulse of 108 beats/min, and a respiratory rate of 24 breaths/min. Eight hours later her blood pressure is 150/110 mm Hg, the pulse is 98 beats/min, the respiratory rate is 10 breaths/min, and the knee-jerk reflex is absent. What should the nurse do next? 1. Stop the infusion of magnesium sulfate and notify the primary healthcare provider. 2. Administer calcium gluconate, because it is an antidote to magnesium sulfate. 3. Continue the magnesium sulfate infusion, because the blood pressure is still high. 4. Check vital signs and reflexes in 1 hour and then discontinue the infusion if necessary.

Correct 1. Stop the infusion of magnesium sulfate and notify the primary healthcare provider.

A client at 36 weeks' gestation is admitted to the high-risk unit with the diagnosis of severe preeclampsia, and antiseizure therapy is instituted. A fetal monitor and an electronic blood pressure machine are applied. Which complication of severe preeclampsia requires diligent monitoring of the blood pressure? 1. Stroke 2. Hemorrhage 3. Precipitous labor 4. Disseminated intravascular coagulation

Correct 1. Stroke

A client at 41 weeks' gestation is scheduled for a contraction stress test (CST). How should the nurse explain a CST result that is interpreted as negative? 1. The fetus at this time is likely to tolerate the stress of labor; however, the test should be repeated weekly. 2. The test should be repeated in 24 hours because the examination results indicate hyperstimulation. 3. Immediate birth should be considered because there is no fetal heart acceleration with fetal movement. 4. A trial induction should be started because fetal heart rate acceleration with movement indicates a false result.

Correct 1. The fetus at this time is likely to tolerate the stress of labor; however, the test should be repeated weekly.

A pregnant client is concerned that she may have been infected with human immunodeficiency virus (HIV). Which information should a nurse include when counseling this client regarding HIV testing? Select all that apply. 1. The risks of passing the virus to the fetus 2. What positive or negative test results indicate 3. The risk factors for contracting HIV 4. The need for pregnant women to be tested for HIV 5. The emotional, legal, and medical implications of test results

Correct 1. The risks of passing the virus to the fetus 2. What positive or negative test results indicate 5. The emotional, legal, and medical implications of test results

A client at 36 weeks' gestation is admitted to the high-risk unit with heavy bleeding because of complete placenta previa. Why does the nurse place the client in a lateral Trendelenburg position? 1. To prevent shock 2. To control bleeding 3. To keep pressure off the cervix 4. To move the placenta off the cervix

Correct 1. To prevent shock

The nurse in the birthing unit is caring for several postpartum clients. Which factor will increase the risk for hypotonic uterine dystocia? 1. Twin gestation 2. Gestational anemia 3. Hypertonic contractions 4. Gestational hypertension

Correct 1. Twin gestation

A client at 36-weeks' gestation arrives at the prenatal clinic for a routine examination. The nurse determines that the client's blood pressure has increased from 102/60 mm Hg to 134/88 mm Hg and becomes concerned she may be experiencing mild preeclampsia. What other sign of mild preeclampsia does the nurse anticipate? 1. Urine dipstick positive for protein 2. Mild ankle edema 3. Episodes of dizziness on arising 4. Weight gain of 2 lb (907 g) in 2 weeks

Correct 1. Urine dipstick positive for protein

A newborn with acquired herpes simplex virus infection is being discharged. Which facet of development should the nurse instruct the parents to monitor closely? 1. Visual clarity 2. Renal function 3. Long bone growth 4. Responses to sounds

Correct 1. Visual clarity

A nurse is teaching a breastfeeding mother about cleansing her nipples. What technique should the nurse emphasize? 1. Wash the breasts and nipples with water when bathing 2. Wipe the nipples with sterile water before each feeding 3. Swab the nipples with an alcohol sponge after each feeding 4. Rub the breasts and nipples with soapy water when showering

Correct 1. Wash the breasts and nipples with water when bathing

A client states that she wishes to use the calendar method of birth control. The nurse concludes that the client understands how to calculate the beginning of the fertile period when she makes which statement? 1. "I will subtract 11 days from the length of my longest cycle." 2. "I will subtract 18 days from the length of my shortest cycle." 3. "I will abstain from having sex after the 10th day of my cycle." 4. "I will abstain from sex from the 10th day before the middle of my average cycle."

Correct 2. "I will subtract 18 days from the length of my shortest cycle."

One minute after birth a nurse assesses a newborn and auscultates a heart rate of 90 beats/min. The newborn has a strong, loud cry, moves all extremities well, and has acrocyanosis but is otherwise pink. What is this neonate's Apgar score 1. 9 2. 8 3. 7 4. 6

Correct 2. 8

Twenty-four hours after an uncomplicated labor and birth a client's complete blood count reveals a white blood cell (WBC) count of 17,000/mm3. How should the nurse interpret this WBC count? 1. A normal decrease in WBCs 2. An expected response to the process of labor and birth 3. A sign of an acute sexually transmitted viral infection 4. A sign of a bacterial infection of the reproductive system

Correct 2. An expected response to the process of labor and birth

A nurse teaches a woman who is planning to breastfeed how to relieve breast engorgement. The nurse determines that further teaching is necessary when the woman states that she will do what? 1. Manually express breast milk 2. Breastfeed the infant less frequently 3. Apply warm compresses to both breasts 4. Place cold compresses on the breasts just after breastfeeding

Correct 2. Breastfeed the infant less frequently

The nurse is caring for a client in transitioning labor and notes an early deceleration on the fetal heart monitor. Which nursing intervention would be most appropriate at this time? 1. Administering oxygen at 12 L/min 2. Continuing to monitor fetal heart rate (FHR) tracing 3. Placing the client in a side-lying position 4. Stopping the oxytocin drip

Correct 2. Continuing to monitor fetal heart rate (FHR) tracing

A nurse on the high-risk unit is caring for a client with severe preeclampsia. Which intervention is the most effective in preventing a seizure? 1. Providing a plastic airway 2. Controlling external stimuli 3. Having emergency equipment available 4. Keeping calcium gluconate at the bedside

Correct 2. Controlling external stimuli

A multigravida of Asian descent weighs 104 lb (47.2 kg), having gained 14 pounds (6.4 kg) during the pregnancy. On her second postpartum day, the client is withdrawn and eating very little from the meals provided. Which intervention is most important for the nurse to implement? 1. Report these findings to the healthcare provider. 2. Encourage the family to bring in special foods preferred in their culture. 3. Order a high-protein milkshake to supplement between meals. 4. Call the dietitian to work with client to plan high calorie meals for the client to eat.

Correct 2. Encourage the family to bring in special foods preferred in their culture.

The nurse is caring for a client who is having a precipitous labor. For which complication should the nurse make a focused nursing assessment when a rapid descent of the fetus is experienced? 1. Microcephaly 2. Fetal head trauma 3. Fracture of the maternal coccyx 4. Prolonged retention of the placenta

Correct 2. Fetal head trauma

The parent of a preterm infant asks the nurse in the neonatal intensive care unit why the baby is in a bed with a radiant warmer. How does the nurse explain the increased risk for hypothermia in preterm infants? 1. Have a smaller body surface area than full-term newborns 2. Lack the subcutaneous fat that usually provides insulation 3. Perspire excessively, causing a constant loss of body heat 4. Have a limited ability to produce antibodies against infections

Correct 2. Lack the subcutaneous fat that usually provides insulation

An infant of a diabetic mother is admitted to the neonatal intensive care unit. What is the priority nursing intervention for this infant? 1. Clamping the cord a second time 2. Obtaining heel blood to test the glucose level 3. Starting an intravenous (IV) infusion of glucose in water 4. Instilling an ophthalmic antibiotic to prevent an eye infection

Correct 2. Obtaining heel blood to test the glucose level

Which clinical finding should the nurse evaluate before continuing the administration of intravenous (IV) magnesium sulfate therapy? 1. Temperature and respirations 2. Patellar reflexes and urinary output 3. Urinary glucose and specific gravity 4. Level of consciousness and funduscopic appearance

Correct 2. Patellar reflexes and urinary output

A client with heart disease is admitted to the birthing suite. Which nursing intervention may help prevent the development of cardiac decompensation during her labor? 1. Positioning her on the side with her head on a pillow 2. Positioning her on the side with her shoulders elevated 3. Administering the prescribed intravenous (IV) infusion of isotonic saline 4. Administering the prescribed IV piggyback infusion of oxytocin

Correct 2. Positioning her on the side with her shoulders elevated

Which clinical finding is most important for the nurse to assess if a client has had a precipitous birth? 1. Sudden chilling 2. Profuse bleeding 3. Decrease in heart rate 4. Increased blood pressure

Correct 2. Profuse bleeding

Rho(D) immune globulin (RhoGAM) is prescribed for an Rh-negative client who has just given birth. Before giving the medication, the nurse verifies the newborn's Rh factor and reaction to the Coombs test. Which combination of newborn Rh factor and Coombs test result confirms the need to give Rho(D) immune globulin? 1. Rh positive with a positive Coombs result 2. Rh positive with a negative Coombs result 3. Rh negative with a positive Coombs result 4. Rh negative with a negative Coombs result

Correct 2. Rh positive with a negative Coombs result

A pregnant client asks the clinic nurse how smoking will affect her baby. What information about cigarette smoking will influence the nurse's response? 1. It relieves maternal tension, and the fetus responds accordingly to the reduction in stress. 2. The resulting vasoconstriction affects both fetal and maternal blood vessels. 3. Substances contained in smoke diffuse through the placenta and compromise the fetus's well-being. 4. Effects are limited because fetal circulation and maternal circulation are separated by the placental barrier.

Correct 2. The resulting vasoconstriction affects both fetal and maternal blood vessels.

Which intervention should the nurse take immediately when an apnea monitor sounds an alarm 10 seconds after cessation of respirations? 1. Assess for changes in skin color 2. Use tactile stimuli on the chest or extremities 3. Check the monitor for signs of a malfunction 4. Resuscitate with a facemask and an Ambu bag

Correct 2. Use tactile stimuli on the chest or extremities

A nurse is caring for a postpartum client. Where does the nurse expect the fundus to be located if involution is progressing as expected 12 hours after birth? 1. 2 cm below the umbilicus 2. 3 cm above the umbilicus 3. 1 cm above the umbilicus 4. 3 cm below the umbilicus

Correct 3. 1 cm above the umbilicus

The nurse assures a breast-feeding mother that one way she will know that her infant is getting an adequate supply of breast milk is if the infant gains weight. What behavior does the infant exhibit if an adequate amount of milk is being ingested? 1. Has several firm stools daily 2. Voids six or more times a day 3. Spits out a pacifier when offered 4. Awakens to feed about every 4 hours

Correct 2. Voids six or more times a day

An intravenous infusion of magnesium sulfate is prescribed for a client with severe preeclampsia. The dosage is twice the usual adult dosage. When a nurse questions the dosage, the primary healthcare provider insists that it is the desired dosage and directs the nurse to administer the medication. How should the nurse respond to this directive? 1. Administer the dose and monitor the client. 2. Withhold the dose and notify the nurse manager. 3. Administer the dose and document it on the client's record. 4. Withhold the dose and notify the director of the obstetric department.

Correct 2. Withhold the dose and notify the nurse manager.

A client visiting the prenatal clinic for the first time tells the nurse that she has heard conflicting stories regarding sex during pregnancy and asks about continuing sexual activity. How should the nurse respond? 1. "You should discontinue intercourse after the second trimester." 2. "This information can be given only by your obstetrician or nurse-midwife." 3. "With an uncomplicated pregnancy, there are no limitations on sexual activity." 4. "Sexual activity should be avoided during the first and last six weeks of pregnancy."

Correct 3. "With an uncomplicated pregnancy, there are no limitations on sexual activity."

A client who is formula feeding her infant complains of discomfort from engorged breasts. What should the nurse recommend that the client do? 1. Use warm, moist towels as compresses. 2. Express milk from each breast manually. 3. Apply cold packs and a snugly fitting bra. 4. Restrict oral fluid intake to less than a quart a day.

Correct 3. Apply cold packs and a snugly fitting bra

After performing Leopold maneuvers on a laboring client, the nurse determines that the fetus is in the right occiput posterior (ROP) position. Where should the Doppler ultrasound transducer be placed to best auscultate fetal heart tones? 1. Above the umbilicus in the midline 2. Above the umbilicus on the left side 3. Below the umbilicus on the right side 4. Below the umbilicus near the left groin

Correct 3. Below the umbilicus on the right side

A client whose membranes have ruptured is admitted to the birthing unit. Her cervix is dilated 3 cm and 50% effaced. The amniotic fluid is clear, and the fetal heart rate is stable. Which outcome does the nurse anticipate? 1. A prolonged second stage of labor 2. A difficult birth resulting from delayed effacement 3. Birth of the fetus within a day 4. The stimulation of labor with an oxytocin infusion

Correct 3. Birth of the fetus within a day

A client who is admitted for surgery for a ruptured tubal pregnancy tells the nurse that she has shoulder pain. What does the nurse conclude that the pain is caused by? 1. Anxiety about the diagnosis 2. Cardiac changes from hypovolemia 3. Blood accumulation under the diaphragm 4. Rebound tenderness from the ruptured tube

Correct 3. Blood accumulation under the diaphragm

A primigravida at term is admitted to the birthing room in active labor. Later, when the client is dilated 8 cm, she tells the nurse that she has the urge to push. The nurse instructs her to pant-blow at this time because pushing can cause which of the following? 1. Prolapsed cord 2. Ruptured uterus 3. Cervical edema 4. Lead to a precipitous birth

Correct 3. Cervical edema

A newly arrived Russian immigrant attends the prenatal clinic for the first time. Although she states that she has had immunizations, she does not know which ones. Which immunizations should the nurse recommend? Select all that apply. 1. Mumps 2. Measles 3. Diphtheria 4. Hepatitis B 5. Chickenpox

Correct 3. Diphtheria 4. Hepatitis B

A client in labor is receiving an oxytocin (Pitocin) infusion. Which intervention is a priority for the nurse when repetitive late decelerations of the fetal heart rate are observed? 1. Administer oxygen. 2. Place the client on the left side. 3. Discontinue the oxytocin infusion. 4. Check the client's blood pressure.

Correct 3. Discontinue the oxytocin infusion.

A client had a rubella infection (German measles) during the fourth month of pregnancy. At the time of the infant's birth, the nurse places the newborn in the isolation nursery. Which type of infection control precautions should the nurse institute? 1. Enteric 2. Contact 3. Droplet 4. Standard

Correct 3. Droplet

A 24-year-old woman wants to use her basal body temperature (BBT) in natural family planning but is unsure when to take her temperature. When should the nurse explain is the best time for accurate BBT assessment? 1. Each night right before bed 2. On the first day of her next menstrual cycle 3. Each morning before getting out of bed or increasing her activity 4. At bedtime beginning on day 14 of her menstrual cycle and continuing until her next period

Correct 3. Each morning before getting out of bed or increasing her activity

Which content should the nurse emphasize in a prepared childbirth class? 1. Birth as a family experience 2. Labor without the use of analgesics 3. Education, exercise, and breathing techniques 4. Hydration, relaxation, and pain control during labor

Correct 3. Education, exercise, and breathing techniques

What should supportive nursing care at the beginning of the mother-infant relationship include? 1. Suggesting that the mother choose breastfeeding instead of formula feeding 2. Advising the mother to engage in rooming-in with the newborn at the bedside 3. Encouraging the mother to help out with simple aspects of her newborn's care 4. Observing the mother-infant interaction unobtrusively to evaluate the relationship

Correct 3. Encouraging the mother to help out with simple aspects of her newborn's care

A nurse on the high-risk unit assesses a client admitted with severe preeclampsia. The client has audible crackles in the lower left lobe, slight blurring of vision in the right eye, generalized facial edema, and epigastric discomfort. Which clinical manifestation indicates the potential for a seizure? 1. Audible crackles 2. Blurring of vision 3. Epigastric discomfort 4. Generalized facial edema

Correct 3. Epigastric discomfort

A nurse is assessing a client at 16 weeks' gestation. Where does the nurse expect the fundal height to be located? 1. Above the umbilicus 2. At the level of the umbilicus 3. Half the distance to the umbilicus 4. Slightly above the symphysis pubis

Correct 3. Half the distance to the umbilicus

A nurse in the prenatal clinic is caring for a client with heart disease who is in her second trimester. Which hemodynamic change of pregnancy is likely to affect the client at this time? 1. Decreased red blood cell count 2. Gradually increasing size of the uterus 3. Heart rate acceleration in the last half of pregnancy 4. Increase in cardiac output during the third trimester

Correct 3. Heart rate acceleration in the last half of pregnancy

A client measuring at 18 weeks' gestation visits the prenatal clinic stating that she is still very nauseated and vomits frequently. Physical examination reveals a brown vaginal discharge and a blood pressure of 148/90 mm Hg. What condition does the nurse suspect the client is experiencing? 1. Dehydration 2. Choriocarcinoma 3. Hydatidiform mole 4. Threatened abortion

Correct 3. Hydatidiform mole

A nurse is caring for a client with class III heart disease who is beginning the second stage of labor. For which medical intervention does the nurse prepare the client at this time? 1. Elective cesarean birth to conserve energy 2. Pudendal anesthesia to prevent restlessness 3. Instrument extraction to ease a vaginal birth 4. Intravenous tocolytic medication to weaken contractions

Correct 3. Instrument extraction to ease a vaginal birth

A nurse is assessing a newborn for signs of hyperbilirubinemia (pathologic jaundice). Which clinical finding confirms this complication? 1. Muscle irritability within 1 hour of birth 2. Neurologic signs during the first 24 hours 3. Jaundice that develops in the first 12 to 24 hours 4. Jaundice that develops between 48 and 72 hours after birth

Correct 3. Jaundice that develops in the first 12 to 24 hours

A 36-year-old multigravida who is at 14 weeks' gestation is scheduled for an alpha-fetoprotein test. She asks the nurse, "What does this test do?" The nurse responds that this test can reveal what? 1. Kidney defects 2. Cardiac anomalies 3. Neural tube defects 4. Urinary tract anomalies

Correct 3. Neural tube defects

A client is admitted to the birthing room in active labor. She is gravida 4, para 3. When she is at 8 cm of dilation, her membranes rupture spontaneously. What should the nurse do after assessing fetal well-being? 1. Notify the practitioner. 2. Document the occurrence. 3. Perform a vaginal exam. 4. Change the client's underpad.

Correct 3. Perform a vaginal exam.

A false-negative home pregnancy test may result if the woman does what when performing the test? 1. Saturates the test strip 2. Performs it in the first void of the morning 3. Performs it 3 days after intercourse took place 4. Performs it while taking a prescribed tranquilizer

Correct 3. Performs it 3 days after intercourse took place

A client who is 38 weeks pregnant presents to the labor unit for a nonstress test (NST). The resulting fetal monitor strip is shown. How does the nurse interpret this finding? 1. Negative because of the lack of contractions 2. Nonreassuring; fetal heart rate lacks variability 3. Reassuring; fetal heart rate accelerates with movement 4. Positive; demonstrates decelerations with fetal movement

Correct 3. Reassuring; fetal heart rate accelerates with movement

A breastfeeding mother asks the nurse what she can do to ease the discomfort caused by a cracked nipple. What should the nurse instruct the client to do? 1. Stop nursing for a few days and allow the nipple to heal. 2. Manually express milk and feed it to the baby in a bottle. 3. Start feedings on the unaffected breast until the affected breast heals. 4. Use a breast shield to keep the baby from making direct contact with the nipple.

Correct 3. Start feedings on the unaffected breast until the affected breast heals.

At the beginning of the first formula feeding a newborn begins to cough and choke, and the lips become cyanotic. What is the nurse's priority action in response to this situation? 1. Stimulate crying 2. Substitute sterile water for the formula 3. Suction and then oxygenate the newborn 4. Stop the feeding momentarily and then restart it

Correct 3. Suction and then oxygenate the newborn

Continuous positive-pressure ventilation therapy by way of an endotracheal tube is started in a newborn with respiratory distress syndrome (RDS). The nurse determines that the infant's breath sounds on the right side are diminished and that the point of maximum impulse (PMI) of the heartbeat is in the left axillary line. How should the nurse interpret these data? 1. These findings are expected because infants with this disorder often have some degree of atelectasis. 2. The inspiratory pressure on the ventilator is probably too low and needs to be increased for adequate ventilation. 3. These findings indicate that the infant may have a pneumothorax and that the health care provider should be contacted immediately. 4. The endotracheal tube needs to be pulled back to ventilate both lungs because it has probably slipped into the left main stem bronchus.

Correct 3. These findings indicate that the infant may have a pneumothorax and that the health care provider should be contacted immediately.

A vaginal examination reveals that a client in labor is dilated 8 cm. Soon afterward she becomes nauseated and has the hiccups. The bloody show increases. Which phase of labor does the nurse determine the client is entering? 1. Latent 2. Active 3. Transition 4. Early active

Correct 3. Transition

A mother asks the neonatal nurse why her infant must be monitored so closely for hypoglycemia when her type 1 diabetes was in excellent control during the entire pregnancy. How should the nurse best respond? 1. "A newborn's glucose level drops after birth, so we're being especially cautious with your baby because of your diabetes." 2. "A newborn's pancreas produces an increased amount of insulin during the first day of birth, so we're checking to see whether hypoglycemia has occurred." 3. "Babies of mothers with diabetes do not have large stores of glucose at birth, so it's difficult for them to maintain the blood glucose level within an acceptable range." 4. "Babies of mothers with diabetes have a higher-than-average insulin level because of the excess glucose received from the mothers during pregnancy, so the glucose level may drop."

Correct 4. "Babies of mothers with diabetes have a higher-than-average insulin level because of the excess glucose received from the mothers during pregnancy, so the glucose level may drop."

A client with active genital herpes has a cesarean birth. The nurse teaches the mother how to limit transmission of the virus to her newborn. The nurse concludes that the instructions have been understood when the mother makes what statement? 1. "I should avoid kissing the baby on the lips." 2. "I have to wear gloves when I'm holding the baby." 3. "I should wash my clothes and my baby's clothes separately." 4. "I have to wash my hands with soap and water before handling the baby."

Correct 4. "I have to wash my hands with soap and water before handling the baby."

A client seeking advice regarding contraception asks a nurse to explain how an intrauterine device (IUD) prevents pregnancy. How should the nurse respond? 1. "It covers the entrance to the cervical os." 2. "The openings to the fallopian tubes are blocked." 3. "The sperm are kept from reaching the vagina." 4. "It produces a spermicidal intrauterine environment."

Correct 4. "It produces a spermicidal intrauterine environment."

A mother is inspecting her newborn girl for the first time. The infant's breasts are edematous, and she has a pink vaginal discharge. How should the nurse respond when the mother asks what is wrong? 1. "You seem very concerned. I don't see anything unusual." 2. "Your baby appears to have a problem. I'll notify the pediatrician." 3. "The swelling and discharge will go away. It's nothing to worry about." 4. "The swelling and discharge are expected. They're a response to your hormones."

Correct 4. "The swelling and discharge are expected. They're a response to your hormones."

Placenta previa is diagnosed when a client at 24 weeks' gestation presents with painless vaginal bleeding. The client is concerned that she has done something to cause the bleeding. How should the nurse respond? 1. "It's not your fault; these things happen." 2. "Don't worry; it's just a sign that labor is beginning." 3. "Your uterus may be weak—that's what causes the vaginal bleeding." 4. "You have a low-lying placenta that separates when the cervix dilates."

Correct 4. "You have a low-lying placenta that separates when the cervix dilates."

The primary healthcare provider plans to perform a vaginal examination of a client with a partial placenta previa. What should the nurse have available when this examination is performed? 1. 1 unit of freeze-dried plasma 2. Vitamin K and a syringe for injection 3. Heparin sodium for intravenous infusion 4. 2 units of typed and crossmatched blood

Correct 4. 2 units of typed and crossmatched blood

The nurse is reviewing the documented results of a lecithin/sphingomyelin (L/S) ratio. Which finding is indicative of fetal lung maturity? 1. 1:1 2. 1.4:1 3. 1.8:1 4. 2:1

Correct 4. 2:1

A client who has type O Rh-positive blood gives birth. The neonate has type B Rh-negative blood. When the nurse assesses the neonate 11 hours after birth, the infant's skin appears yellow. What is the most likely cause? 1. Neonatal sepsis 2. Rh incompatibility 3. Physiologic jaundice 4. ABO incompatibility

Correct 4. ABO incompatibility

The nurse is obtaining the health history of a woman who is visiting the prenatal clinic for the first time. She states that she is 5 months pregnant. Which positive sign of pregnancy should the nurse evaluate in this client? 1. Quickening 2. Enlarged abdomen 3. Cervical color change 4. Audible fetal heartbeat

Correct 4. Audible fetal heartbeat

A woman in labor with her third child is dilated to 7 cm, and the fetal head is at station +1. The client's membranes rupture. What is the nurse's priority intervention? 1. Notify the practitioner. 2. Observe the vaginal opening for a prolapsed cord. 3. Reposition the client on a sterile towel on her left side. 4. Check the fetal heart rate while observing the color of the amniotic fluid.

Correct 4. Check the fetal heart rate while observing the color of the amniotic fluid.

External fetal uterine monitoring is started for a client in active labor. A nurse identifies fetal heart rate decelerations in a uniform wave shape that reflects the shape of the contraction. What is the nurse's next action? 1. Notifying the healthcare provider of possible head compression 2. Placing the client in a knee-chest position to avoid cord compression 3. Putting the client in a dorsal recumbent position to prevent compression of the vena cava 4. Continuing to monitor the client for the return of the fetal heart rate to baseline when each contraction ends

Correct 4. Continuing to monitor the client for the return of the fetal heart rate to baseline when each contraction ends

What is the safest and most reliable birth control method for the nurse to recommend to a client with type 1 diabetes? 1. Vaginal sponge 2. Oral contraceptive 3. Rhythm method with a condom 4. Diaphragm with a spermicidal gel

Correct 4. Diaphragm with a spermicidal gel

A newborn has just begun to breast-feed for the first time. Although the neonate has latched on to the mother's nipple, soon after beginning to suck the infant begins to choke, has an excessive quantity of frothy secretions, and exhibits unexplained episodes of cyanosis. How should the nurse best intervene at this time? 1. Tell the client to use the other breast and continue breast-feeding 2. Delay the feeding to allow more time for the infant to recover from the birthing process 3. Contact the lactation consultant to help the client learn a more successful breast-feeding technique 4. Halt the feeding and notify the healthcare provider to evaluate the infant for a tracheoesophageal fistula

Correct 4. Halt the feeding and notify the healthcare provider to evaluate the infant for a tracheoesophageal fistula

A client at term is admitted in active labor. She has tested positive for human immunodeficiency virus (HIV). Which intervention in the standard prescriptions should the nurse question? 1. Sonogram 2. Nonstress test 3. Sterile vaginal examination 4. Internal fetal scalp electrode

Correct 4. Internal fetal scalp electrode

A preterm neonate admitted to the neonatal intensive care nursery exhibits muscle twitching; seizures; cyanosis; abnormal respirations; and a short, shrill cry. Which complication does the nurse suspect? 1. Tetany 2. Spina bifida 3. Hyperkalemia 4. Intracranial hemorrhage

Correct 4. Intracranial hemorrhage

A newborn who has remained in the hospital because the mother had a cesarean birth is to be tested for phenylketonuria (PKU) on the morning of discharge. How should the nurse explain the purpose of PKU testing to this mother? 1. It detects thyroid deficiency 2. It reveals possible brain damage 3. It identifies chromosomal damage 4. It is used to measure protein metabolism

Correct 4. It is used to measure protein metabolism

Which nursing action best promotes parent-infant attachment behaviors? 1. Restricting visitors on the postpartum unit 2. Supporting rooming-in with parent-infant care 3. Encouraging the mother to choose breast-feeding 4. Keeping the new family together immediately after the birth

Correct 4. Keeping the new family together immediately after the birth

The nurse is caring for a high-risk pregnant client who has had a positive contraction stress test (CST). What would the nurse interpret the result to mean? 1. A nonstress test is needed. 2. An immediate cesarean birth is needed. 3. The fetal heart is within the expected limits for the average fetus. 4. Late decelerations of the fetal heart rate are occurring with each contraction.

Correct 4. Late decelerations of the fetal heart rate are occurring with each contraction.

The nurse is assessing the rate of involution of a client's uterus on the second postpartum day. Where does the nurse expect the fundus to be located? 1. At the level of the umbilicus 2. One fingerbreadth above the umbilicus 3. Above and to the right of the umbilicus 4. One or two fingerbreadths below the umbilicus

Correct 4. One or two fingerbreadths below the umbilicus

A grand multipara at 34 weeks' gestation is brought to the emergency department because of vaginal bleeding. The nurse suspects that the client has a placenta previa. Which characteristic typical of placenta previa supports the nurse's conclusion? 1. Painful vaginal bleeding in the first trimester 2. Painful vaginal bleeding in the third trimester 3. Painless vaginal bleeding in the first trimester 4. Painless vaginal bleeding in the third trimester

Correct 4. Painless vaginal bleeding in the third trimester

A newborn is experiencing cold stress while being admitted to the nursery. Which nursing goal has the highest immediate priority? 1. Minimize shivering 2. Prevent hyperglycemia 3. Limit oxygen consumption 4. Prevent metabolism of fat stores

Correct 4. Prevent metabolism of fat stores

A newborn is diagnosed as having neonatal abstinence syndrome (NAS) after exhibiting jitteriness, irritability, and a shrill cry. What is the priority nursing care? 1. Administering an opioid antagonist 2. Limiting fluid intake to inhibit vomiting 3. Assessing for age-appropriate developmental level 4. Reducing environmental stimuli to promote relaxation

Correct 4. Reducing environmental stimuli to promote relaxation

A laboring client has asked the nurse to help her use a nonpharmacologic strategy for pain management. Name the sensory simulation strategy. 1. Gentle massage of the abdomen 2. Biofeedback-assisted relaxation techniques 3. Application of a heat pack to the lower back 4. Selecting a focal point and beginning breathing techniques

Correct 4. Selecting a focal point and beginning breathing techniques

A client at 42 weeks' gestation is scheduled for induction of labor. The nurse begins the induction with a piggyback infusion of 15 units of oxytocin. Which clinical finding requires the nurse to discontinue the oxytocin infusion? 1. Contractions that occur every 3 minutes and lasting 60 seconds 2. Elevation of blood pressure from 110/70 to 135/85 mm Hg during the last 30 minutes 3. Rupture of membranes with amniotic fluid that contains threads of blood and mucus 4. Several late fetal heart rate decelerations that return to baseline after the contraction is over

Correct 4. Several late fetal heart rate decelerations that return to baseline after the contraction is over

Before teaching a client about breastfeeding, which information regarding hormonal influences should the nurse fully understand? 1. A high level of progesterone stimulates the secretion of oxytocin. 2. A high level of estrogen stimulates the secretion of lactogenic hormones. 3. Milk secretion is under the control of postpartum hormones starting immediately after birth. 4. Suckling stimulates the pituitary gland to release oxytocin, which initiates the let-down reflex.

Correct 4. Suckling stimulates the pituitary gland to release oxytocin, which initiates the let-down reflex.

How should the nurse screen the newborn of a diabetic mother for hypoglycemia? 1. Testing for glucose tolerance 2. Drawing blood for a serum glucose determination 3. Arranging for a fasting blood glucose determination 4. Testing heel blood with the use of a glucose-oxidase strip

Correct 4. Testing heel blood with the use of a glucose-oxidase strip

A pregnant client with diabetes is referred to the dietitian in the prenatal clinic for nutritional assessment and counseling. What should the nurse emphasize when reinforcing the client's dietary program? 1. The need to increase high-quality protein and decrease fats 2. The need to increase carbohydrates to meet energy demands and prevent ketosis 3. The need to eat a low-calorie diet that maintains the current insulin coverage and helps prevent hyperglycemia 4. The need to eat a pregnancy diet that meets increased dietary needs and to adjust the insulin dosage as necessary

Correct 4. The need to eat a pregnancy diet that meets increased dietary needs and to adjust the insulin dosage as necessary

In her 37th week of gestation, a client with type 1 diabetes has undergone an amniocentesis to determine fetal lung maturity. The lecithin/sphingomyelin ratio is 2:1, phosphatidylglycerol is present, and creatinine is 2 mg/dL (180 mcmol/L). What conclusion should the nurse draw from this information? 1. A cesarean birth will be scheduled. 2. A birth must take place immediately. 3. The fetus need not be monitored any longer. 4. The newborn should be free from respiratory problems.

Correct 4. The newborn should be free from respiratory problems.

A nurse is assessing a postpartum client for signs of hemorrhage by evaluating the degree of perineal pad saturation. What other parameter can the nurse use to estimate blood loss in a postpartum client? 1. Odor of the lochia 2. Color of the lochia 3. Presence of small clots on the pad 4. Time elapsed between pad changes

Correct 4. Time elapsed between pad changes

The nurse is interpreting the results of a nonstress test (NST) on a client at 41 weeks' gestation. Which result after 20 minutes is suggestive of fetal reactivity? 1. Absence of long-term variability 2. Above-average fetal baseline heart rate of 160 beats/min 3. No late decelerations associated with contractions 4. Two accelerations of 15 beats/min lasting 15 seconds

Correct 4. Two accelerations of 15 beats/min lasting 15 seconds

One minute after birth a nurse notes that a newborn is crying, has a heart rate of 140 beats/min, is acrocyanotic, resists the suction catheter, and keeps the arms extended. What Apgar score should the nurse assign to the newborn? Record your answer using a whole number. ______

Correct 8

A nurse assesses the process of involution of the uterus by measuring the location of the client's fundus during the postpartum period. Click on the location the fundus is expected to be 1 day after birth in a client whose bladder is not distended. THIS WILL HAVE AN IMAGE.

Correct One day after birth, the fundus is expected to be at the level of the umbilicus

At 1 minute after birth the nurse determines that an infant is crying, has a heart rate of 140 beats/min, has blue hands and feet, resists the suction catheter, and keeps the legs flexed and the arms extended. What Apgar score should the nurse assign? 1. 6 2. 7 3. 8 4. 9

Correct 3. 8

A client at her first visit to the prenatal clinic states that she has missed three menstrual periods and thinks that she is carrying twins because her abdomen is so large. She now has a brownish vaginal discharge. Her blood pressure is increased, indicating that she may have gestational hypertension. What condition does the nurse suspect the client may have? 1. Renal failure 2. Placenta previa 3.Hydatidiform mole 4. Abruptio placentae

Correct 3. Hydatidiform mole

A client is transferred to the postpartum care unit 1 hour after a spontaneous vaginal delivery. On assessment, the nurse finds the fundus at U-1 and firm and the pad saturated with blood. The pad is changed and reassessed 15 minutes later and again found saturated with blood. The fundus remains at U-1, midline and firm. Place the interventions in order of priority. *Weighing pads to measure blood loss *Assessing the episiotomy *Assessing the client for a hematoma *Calling the primary healthcare provider *Taking vital signs

Correct 1. Assessing the episiotomy Correct 2. Assessing the client for a hematoma Correct 3. Weighing pads to measure blood loss Correct 4. Taking vital signs Correct 5. Calling the primary healthcare provider

A client's membranes rupture, and the nurse immediately detects the presence of a prolapsed umbilical cord. The nurse alerts another nurse, who calls the primary healthcare provider. Place the following nursing interventions in the order in which they should be performed. *Moving the presenting part off the cord *Checking the fetal heart rate *Placing the client in the Trendelenburg position *Administering oxygen by facemask

Correct 1. Moving the presenting part off the cord Correct 2. Placing the client in the Trendelenburg position Correct 3. Administering oxygen by facemask Correct 4. Checking the fetal heart rate

A number of routine screens are performed on the pregnant client during the course of her gestation. Place in correct order the sequence of their testing. *Alpha-fetoprotein (AFP) testing for neural tube defects *Sickle cell screening *Group B streptococcus culture *Fetal movement test *Serum glucose for gestational diabetes

Correct 1. Sickle cell screening Correct 2. Alpha-fetoprotein (AFP) testing for neural tube defects Correct 3. Serum glucose for gestational diabetes Correct 4. Fetal movement test Correct 5. Group B streptococcus culture

Epidural anesthesia was initiated 30 minutes ago for a client in labor. The nurse determines that the fetus is experiencing late decelerations. List the following nursing actions in order of priority. *Reposition client on her side. *Increase intravenous fluids. *Reassess the fetal heart rate (FHR) pattern. *Document interventions and related maternal/fetal responses. *Notify the healthcare provider if late decelerations persist.

Correct1. Reposition client on her side. Correct2. Increase intravenous fluids. Correct3. Reassess the fetal heart rate (FHR) pattern. Correct4. Notify the healthcare provider if late decelerations persist. Correct5. Document interventions and related maternal/fetal responses.

What's the term for: Grating sensation during palpation

Crepitus

Lab Values: Hematocrit

Non-pregnant: 35.4- 44.4 Pregnant pt.: 28-41

What's the term for: Caused by compression of the fetal umbilical cord

Variable decelerations

The nurse is caring for a pregnant woman with class II cardiac disease. The client has anemia with a hemoglobin level of 8 g/dL (80 mmol/L). What is the nurse's primary concern for this client? 1. Impending heart failure 2. Development of heart block 3. Appearance of atrial fibrillation 4. Imminent ventricular fibrillation

Correct 1. Impending heart failure

The nurse is counseling a pregnant client with type 1 diabetes regarding medication changes as pregnancy progresses. Which medication will be needed in increased dosages during the second half of her pregnancy? 1. Insulin 2. Antihypertensives 3. Pancreatic enzymes 4. Estrogenic hormones

Correct 1. Insulin

During her first prenatal visit the client reports that her last menstrual period began on April 15. According to Nägele rule, what is the expected date of delivery (EDD)? 1. January 8 2. January 22 3. February 8 4. February 22

Correct 2. January 22

A client who has just begun breastfeeding reports that her nipples feel very sore. What should the nurse encourage this new mother to do? Select all that apply. 1. Apply cool packs to her breasts to reduce the discomfort. 2. Take the analgesic medication prescribed to limit the discomfort. 3. Remove the infant from the breast for a few days to rest the nipples. 4. Never expose the nipples to air; wear only a tight-fitting brassiere. 5. Assume a different position when breastfeeding to adjust the infant's sucking.

Correct 1. Apply cool packs to her breasts to reduce the discomfort. 2. Take the analgesic medication prescribed to limit the discomfort. 5.Assume a different position when breastfeeding to adjust the infant's sucking.

A laboring client experiences a spontaneous rupture of membranes. What is the nurse's priority? 1. Assessing the fetal heart rate 2. Estimating the amount of fluid 3. Assessing the characteristics of the fluid 4. Repositioning the client to a side-lying position

Correct 1. Assessing the fetal heart rate

During labor a client begins to experience dizziness and tingling of her hands. What should the nurse instruct the client to do? 1. Breathe into her cupped hands 2. Pant during the next three contractions 3. Hold her breath with the next contraction 4. Use a fast, deep, or shallow breathing pattern

Correct 1. Breathe into her cupped hands

The nurse is caring for a postpartum client with preeclampsia being managed with a magnesium sulfate infusion. What is the priority nursing assessment? 1. Counting respiratory rate 2. Obtaining blood pressure 3. Eliciting deep tendon reflexes 4. Monitoring urine output

Correct 1. Counting respiratory rate

The nurse is differentiating between cephalhematoma and caput succedaneum. What finding is unique to caput succedaneum? 1. Edema that crosses the suture line 2. Scalp tenderness over the affected area 3. Edema that increases during the first day 4. Scalp over the area becomes ecchymosed

Correct 1. Edema that crosses the suture line

A woman who is admitted to the labor suite has herpes simplex virus type 2 (HSV-2) with active lesions in the perineal area. What should the nurse's plan of care include? 1. Withholding oral fluid intake 2. Discussing the need for formula feeding 3. Obtaining permission for a paracervical block 4. Applying moist compresses to the perineal area

Correct 1. Withholding oral fluid intake

What must the nurse assess first when planning to promote mother-infant attachment? 1. Mother-infant interaction 2. Mother-father interaction 3. The infant's physical status 4. The mother's ability to care for her infant

Correct 1. Mother-infant interaction

Before discharge, what suggestion should the nurse give to a nonnursing mother to help limit breast engorgement? 1. Place raw cabbage leaves over the breast. 2. Stop drinking milk for 1 week. 3. Take an analgesic every 4 hours. 4. Apply warm compresses to the breasts.

Correct 1. Place raw cabbage leaves over the breast

A pregnant client experiences an episode of painless vaginal bleeding during the last trimester. What does the nurse suspect is the cause of this bleeding? 1. Placenta previa 2. Abruptio placentae 3. Frequent sexual intercourse 4. Excessive alcohol ingestion

Correct 1. Placenta previa

The nurse is reevaluating a newborn who had an axillary temperature of 97° F (36.1° C) and was placed skin to skin with the mother. The newborn's axillary temperature is still 97° F (36.1° C) after 1 hour of skin-to-skin contact. Which intervention should the nurse implement next? 1. Placing the newborn under a radiant warmer in the nursery 2. Checking the newborn for a wet diaper and then continue the skin-to-skin contact 3. Leaving the newborn in skin-to-skin contact and rechecking the temperature in 1 hour 4. Double-wrapping the newborn in warm blankets and returning the newborn to a crib by the mother's bedside

Correct 1. Placing the newborn under a radiant warmer in the nursery

The nurse is testing newborns' heel blood for the level of glucose. Which newborn does the nurse anticipate will experience hypoglycemia? Select all that apply. 1.Preterm infant 2. Infant with Down syndrome 3. Small-for-gestational-age infant 4. Large-for-gestational-age infant 5. Appropriate-for-gestational-age infant

Correct 1. Preterm infant 3. Small-for-gestational-age infant 4. Large-for-gestational-age infant

A client at 34 weeks' gestation is receiving terbutaline subcutaneously. Her contraction frequency increases to every 5 minutes, and her cervix dilates to 4 cm. The tocolytic is discontinued. What is the priority nursing care focus during this time? 1. Promoting maternal-fetal well-being during labor 2. Reducing the anxiety associated with preterm labor 3. Supporting communication between the client and her partner 4. Assisting the client and her partner with the breathing techniques needed as labor progresses

Correct 1. Promoting maternal-fetal well-being during labor

An infant born at 36 weeks' gestation weighs 4 lbs 3 oz (1,899 g) and has Apgar scores of 7 and 9. Which nursing actions will be performed upon the infant's admission to the nursery? Select all that apply. 1. Recording the neonate's vital signs 2. Administration nasal cannula oxygen 3. Offering a bottle of dextrose in water 4. Evaluation of the neonate's health status 5. Keeping the neonate's body warm

Correct 1. Recording the neonate's vital signs 4. Evaluation of the neonate's health status 5. Keeping the neonate's body warm

Late decelerations are present on the monitor strip of a client who received epidural anesthesia 20 minutes ago. What should the nurse do immediately? 1. Reposition the client from supine to left lateral. 2. Increase the intravenous flow rate from 125 to 150 mL/hr. 3. Administer oxygen at a rate of 8 to 10 L/min by way of face mask. 4. Assess the maternal blood pressure for a systolic pressure below 100 mm Hg.

Correct 1. Reposition the client from supine to left lateral.

The nurse is caring for a newborn with a caput succedaneum. What is the priority nursing action? 1. Supporting the parents 2. Recording neurologic signs 3. Applying a hard protective cap on the head 4. Applying ice packs to the hematoma

Correct 1. Supporting the parents

While performing patterned, paced breathing during the transition phase of labor, a client experiences tingling and numbness of the fingertips. What should the nurse do? 1. Tell the client to breathe into a paper bag. 2. Place an oxygen mask over the client's face. 3. Call the primary healthcare provider to report the client's response. 4. Instruct the client to begin taking slow deep breaths.

Correct 1. Tell the client to breathe into a paper bag.

A client at 26-weeks' gestation arrives at the clinic for her scheduled examination. Her blood pressure is 150/86 mm Hg. She tells the nurse that she has gained 5 lb (2.3 kg) in the last 2 weeks. What is the priority nursing action? 1. Testing the client's urine for albumin 2. Taking the client's body temperature 3. Preparing the client for a vaginal examination 4. Scheduling the client for an appointment in a week

Correct 1. Testing the client's urine for albumin

Which of the following variables are scored on a biophysical profile? Select all that apply. 1.Fetal tone 2. Fetal position 3. Fetal movement 4. Amniotic fluid index 5. Fetal breathing movements 6. Contraction stress test results

Correct 1.Fetal tone 3. Fetal movement 4. Amniotic fluid index 5. Fetal breathing movements

A female client asks a nurse about using an intrauterine device (IUD) for contraception. When explaining this method, what common problem should the nurse include in the discussion? 1. The device can be expelled. 2. The uterus may be perforated. 3. Discomfort during intercourse may occur. 4. Vaginal infections are frequent consequences.

Correct 1. The device can be expelled.

Laboratory studies reveal that a pregnant client's blood type is O, and she is Rh positive. The client asks whether her newborn will have a problem with blood incompatibility. Before responding, the nurse must remember that fetal problems may develop in what circumstance? 1. The fetus has type A or B blood. 2. The fetus is born preterm. 3. The fetus has type O, Rh positive blood. 4. The mother has diabetes.

Correct 1. The fetus has type A or B blood.

A new mother's laboratory results indicate the presence of cocaine and alcohol. Which craniofacial characteristics indicate to the nurse that the newborn has fetal alcohol syndrome (FAS)? Select all that apply. 1. Thin upper lip 2. Wide-open eyes 3. Small upturned nose 4. Larger-than-average head 5. Smooth vertical ridge in the upper lip

Correct 1. Thin upper lip 3. Small upturned nose 5. Smooth vertical ridge in the upper lip

A pregnant client is asking the nurse when she will gain the greatest amount of weight during the pregnancy. At which time during prenatal development should the nurse tell the client to expect the greatest fetal and maternal weight gain? 1. Third trimester 2. Second trimester 3. First 8 weeks 4. Implantation period

Correct 1. Third trimester

Typical signs of neonatal abstinence syndrome related to opioid withdrawal usually begin within 24 hours after birth. What characteristics should the nurse anticipate in the infant of a suspected or known opioid abuser? Select all that apply. 1. Tremors 2. Dehydration 3. Hyperactivity 4. Muscle hypotonicity 5. Prolonged sleep periods

Correct 1. Tremors 3. Hyperactivity

The nurse is caring for a pregnant client who is undergoing an ultrasound examination during the first trimester. The nurse explains that an ultrasound during the first trimester is utilized in order to do what? 1. Estimate fetal age 2. Detect hydrocephalus 3. Rule out congenital defects 4. Approximate fetal linear growth

Correct 1. Estimate fetal age

The nurse notes that a client is voiding frequently in small amounts 8 hours after giving birth. What should the nurse conclude about this minimal output of urine during the early postpartum period? 1. It may indicate retention of urine with overflow. 2. It may be indicative of beginning pyelonephritis. 3. This is common because less fluid is excreted after birth. 4. This is common because fluid intake diminishes after birth.

Correct 1. It may indicate retention of urine with overflow

A woman at 40 weeks' gestation is admitted in active labor. When the client reaches 5 centimeters dilation, the woman asks for and receives epidural analgesia. Once the epidural catheter has been inserted, which assessments and interventions should be performed? Select all that apply. 1. Maintaining intravenous fluid administration 2. Having oxygen available in case of hypotension 3. Checking the bladder for distention every 2 hours 4. Positioning the client supine for ease of monitoring 5. Monitoring fetal heart rate and labor progress per hospital protocol 6. Administering an oxytocin infusion to maintain the labor pattern

Correct 1. Maintaining intravenous fluid administration 2. Having oxygen available in case of hypotension 3. Checking the bladder for distention every 2 hours 5. Monitoring fetal heart rate and labor progress per hospital protocol

During the postpartum period a client with heart disease and type 2 diabetes asks a nurse, "Which contraceptives will I be able to use to prevent pregnancy in the near future?" How should the nurse respond? 1. "You may use oral contraceptives—they're almost completely effective in preventing pregnancy." 2. "You should use foam with a condom to prevent pregnancy—this is the safest method for women with your illnesses." 3. "You'll find that the intrauterine device is best for you, because it prevents a fertilized ovum from implanting in the uterus." 4. "You have little to worry about regarding becoming pregnant in the near future, because women with your illnesses usually become infertile."

Correct 2. "You should use foam with a condom to prevent pregnancy—this is the safest method for women with your illnesses."

A client with poorly controlled type 1 diabetes is now in her thirty-fourth week of pregnancy. The primary healthcare provider tells her that she should have an amniocentesis at 37 weeks to assess fetal lung maturity and that induction of labor will be initiated if the fetus's lungs are mature. The client asks the nurse why an early birth may be necessary. How should the nurse reply? 1. "You'll be protected from developing hypertension." 2. "Your glucose level will be hard to control as you reach term." 3. "The baby will be small enough for you to have a vaginal birth." 4. "The chance that your baby will have hypoglycemia will be reduced."

Correct 2. "Your glucose level will be hard to control as you reach term."

Which client is at the greatest risk for a postpartum infection? 1. A primipara who gives birth to an infant weighing more than 8.5 lb 2. A woman who required catheterization after voiding less than 75 mL 3. A multipara with a hemoglobin level of 11 g at the time of admission 4. A woman who loses at least 350 mL of blood during the birthing process

Correct 2. A woman who required catheterization after voiding less than 75 mL

A client is bleeding excessively after the birth of her newborn. The healthcare provider prescribes fundal massage and an IV infusion containing 10 units of oxytocin at a rate of 100 mL/hr. The nurse's evaluation of the client's responses to these interventions reveals a blood pressure of 135/90 mm Hg, a boggy uterus 3 cm above the umbilicus and displaced to the right, and a perineal pad saturated with bright-red lochia. What is the nurse's next action? 1. Increasing the infusion rate 2. Checking for a distended bladder 3. Continuing to perform fundal massage 4. Continuing to assess the blood pressure

Correct 2. Checking for a distended bladder

A nurse practitioner prescribes doxycycline for a sexually active woman with a history of mucopurulent discharge and bleeding associated with cervical dysplasia, dysuria, and dyspareunia. With which sexually transmitted infection are these clinical findings and medication therapy commonly associated? 1. Herpes simplex 2 2. Chlamydial infection 3. Treponema pallidum 4. Neisseria gonorrhoeae

Correct 2. Chlamydial infection

A nurse is caring for a client during an ultrasonogram. Which parameters does the nurse expect to be used in the determination of pregnancy dates? 1. Occipital frontal diameter at term 2. Crown-to-rump measurement until 11 weeks 3. Bi-parietal diameter of 12 cm or more at term 4. Diagonal conjugate between 26 and 37 weeks

Correct 2. Crown-to-rump measurement until 11 weeks

A nurse evaluates a client's understanding regarding oral contraceptives and concludes teaching is successful when the client states, "While I'm taking birth control pills I should increase my intake of foods containing" what nutrient? 1. Calcium 2. Folic acid 3. Vitamin A 4. Vitamin D

Correct 2. Folic acid

The nurse reviews the blood test results of a client at 24 weeks' gestation. Which finding should be reported to the healthcare provider? 1. Platelets: 230,000 mm3 (230 × 109/L) 2. Hemoglobin: 10.8 g/dL (108 mmol/L) 3. Fasting blood glucose: 90 mg/dL (4.2 mmol/L) 4. White blood cell count: 10,000 mm3 (10 × 109/L)

Correct 2. Hemoglobin: 10.8 g/dL (108 mmol/L) RATIONALE: 12-16 IS NORMAL

A small-for-gestational-age (SGA) newborn has just been admitted to the nursery. Nursing assessment reveals a high-pitched cry, jitteriness, and irregular respirations. With which condition are these signs associated? 1. Hypervolemia 2. Hypoglycemia 3. Hypercalcemia 4. Hypothyroidism

Correct 2. Hypoglycemia

Respiratory distress syndrome (RDS) develops 6 hours after birth in a neonate born at 33 weeks' gestation. What would the nurse's assessment of the newborn at this time reveal? 1. High-pitched cry 2. Intercostal retractions 3. Heart rate of 140 beats/min 4. Respirations of 30 breaths/min

Correct 2. Intercostal retractions

After a cesarean birth a nurse performs fundal checks every 15 minutes. The nurse determines that the fundus is soft and boggy. What is the priority nursing action at this time? 1. Elevating the client's legs 2. Massaging the client's fundus 3. Increasing the client's oxytocin drip rate 4. Examining the client's perineum for bleeding

Correct 2. Massaging the client's fundus

A mother is concerned that her newborn will be exposed to communicable diseases after she is discharged. While teaching the mother ways to decrease the risk of infection, what type of immunity should the nurse explain was transferred to her baby through the placenta? 1. Active natural 2. Passive natural 3. Active artificial 4. Passive artificial

Correct 2. Passive natural

A nurse is discussing immunizations needed to confer active immunity with a pregnant client during her first visit to the prenatal clinic. What information should the nurse consider including to ensure that the client understands the process of active immunity? 1. Protein antigens are formed in the blood to fight invading antibodies. 2. Protein substances are formed by the body to destroy or neutralize antigens. 3. Blood antigens are aided by phagocytes in defending the body against pathogens. 4. Sensitized lymphocytes from an immune donor act as antibodies against invading pathogens.

Correct 2. Protein substances are formed by the body to destroy or neutralize antigens.

A 16-year-old primigravida at 32 weeks' gestation is admitted to the high-risk unit. Her blood pressure is 170/110 mm Hg and she has 4+ proteinuria. She has gained 50 lb (22.7 kg) during the pregnancy, and her face and extremities are edematous. Which complication is this client experiencing? 1. Eclampsia 2. Severe preeclampsia 3. Chronic hypertension 4. Gestational hypertension

Correct 2. Severe preeclampsia

A nurse is assessing a newborn with caput succedaneum. How does the nurse explain the cause of this fetal condition to the new mother? 1. Overlap of fetal bones as they pass through the maternal birth canal 2. Swelling of the soft tissue of the scalp as a result of pressure during labor 3. Hemorrhage of ruptured blood vessels that does not cross the suture lines 4. Accumulation of fluid resulting from partial blockage of cerebrospinal fluid drainage

Correct 2. Swelling of the soft tissue of the scalp as a result of pressure during labor

A client arrives at the clinic with swollen, tender breasts and flulike symptoms. A diagnosis of mastitis is made. What does the nurse plan to do? 1. Help her wean the infant gradually. 2. Teach her to empty her breasts frequently. 3. Review breastfeeding techniques with her. 4. Send a sample of her milk to the laboratory for testing.

Correct 2. Teach her to empty her breasts frequently.

The nurse teaches a postpartum client how to care for her episiotomy in order to prevent infection. Which behavior indicates that the teaching has been effective? 1. The perineal pad is changed twice daily. 2. The client washes her hands before and after she changes a perineal pad. 3. The client rinses her perineum with water after using an analgesic spray. 4. The client cleanses the perineum from the anus toward the symphysis pubis.

Correct 2. The client washes her hands before and after she changes a perineal pad.

A client in active labor has requested epidural anesthesia for pain management. The anesthetist has completed an evaluation, and the nurse has initiated an intravenous fluid bolus. The client's partner asks why this is necessary. What is the best explanation? 1. It is the policy of the institution to provide 2 bags of lactated Ringer solution. 2. There is a risk of hypotension, and the large amount of IV fluid reduces this risk. 3. Giving the large amount of IV fluid is a means of hydrating the client when she is unable to drink. 4. The client must be given 500 mL of fluid to ascertain that the line is patent.

Correct 2. There is a risk of hypotension, and the large amount of IV fluid reduces this risk.

A client gives birth vaginally to an infant who weighs 8 lb, 13 oz (3997 g). An ice pack is applied to the perineum to ease the swelling and pain. The client complains, "This pain in my vagina and rectum is excruciating, and my vagina feels so full and heavy." What does the nurse suspect as the cause of the pain? 1. Full bladder 2. Vaginal hematoma 3. Infected episiotomy 4. Enlarged hemorrhoids

Correct 2. Vaginal hematoma

Which nursing intervention holds the highest priority for a client with class I heart disease during the postpartum period? 1. Promoting early ambulation 2. Watching for signs of cardiac decompensation 3. Assessing the mother's emotional reaction to the birth 4. Instructing the mother about activity levels during the postpartum period

Correct 2. Watching for signs of cardiac decompensation

A new mother who is learning about infant feedings asks the nurse how anyone who is breast-feeding gets anything done with a baby feeding on demand. What is the best response by the nurse? 1. "Most mothers find that feeding whenever the baby cries works out fine." 2. "Perhaps a schedule would be better because the baby is already accustomed to the hospital routine." 3. "Babies on demand feedings eventually set a schedule, so there should be time for you to do other things." 4. "Most breast-feeding mothers find that their babies do better on demand because the amount of milk ingested varies from feeding to feeding."

Correct 3. "Babies on demand feedings eventually set a schedule, so there should be time for you to do other things."

Before discharge, a breastfeeding postpartum client and the nurse discuss methods of birth control. The client asks the nurse, "When will I begin to ovulate again?" How should the nurse respond? 1. "You should discuss this at your first clinic visit." 2. "Ovulation will occur after you stop breastfeeding." 3. "Ovulation may occur before you begin to menstruate." 4. "I really can't tell you, because everyone is so different."

Correct 3. "Ovulation may occur before you begin to menstruate."

A client asks the nurse at the family planning clinic whether contraception is needed while she is breastfeeding. How should the nurse reply? 1. "As long as you aren't having periods, you won't need a contraceptive." 2. "It would be best to delay sexual relations until you have your first period." 3. "You should use contraceptives, because ovulation may occur at any time without a period." 4. "Breastfeeding suppresses ovulation, so you don't need to worry about pregnancy."

Correct 3. "You should use contraceptives, because ovulation may occur at any time without a period."

When does a nurse caring for a client with eclampsia determine that the risk for another seizure has decreased? 1. After birth occurs 2. After labor begins 3. 48 hours postpartum 4. 24 hours postpartum

Correct 3. 48 hours postpartum

A 36-year-old woman, G1 P0, is admitted to the labor and delivery unit for oxytocin induction. She is at 40 weeks' gestation. Which condition is a contraindication to the use of oxytocin induction? 1. Chorioamnionitis 2. Postterm pregnancy 3. Active genital herpes infection 4. Hypertension associated with pregnancy

Correct 3. Active genital herpes infection

Which information in a postpartum client's health history should alert the nurse to monitor the client for signs of infection? 1. Three spontaneous abortions 2. B-negative maternal blood type 3. Blood loss of 850 mL after a vaginal birth 4. Temperature of 99.9° F (37.7° C) during the first postpartum day

Correct 3. Blood loss of 850 mL after a vaginal birth

A client who is in labor is admitted 30 hours after her membranes ruptured. Which condition is this client at increased risk for? 1. Cord prolapse 2. Placenta previa 3. Chorioamnionitis 4. Abruptio placentae

Correct 3. Chorioamnionitis

The four essential components of labor are passenger, powers, passageway, and position. Passageway refers to the bony pelvis. Which type of pelvis is considered the most favorable for a vaginal delivery? 1. Android 2. Anthropoid 3. Gynecoid 4. Platypelloid

Correct 3. Gynecoid

The nurse suspects that a newborn is experiencing opioid withdrawal. Which assessment finding supports this suspicion? 1. Lethargy and constipation 2. Grunting and low-pitched cry 3. Irritability and nasal congestion 4. Watery eyes and rapid respirations

Correct 3. Irritability and nasal congestion

A client is admitted to the emergency department at 34 weeks' gestation with trauma and significant bleeding from the leg. What is the priority intervention after determining fetal well-being? 1. Obtaining the client's vital signs 2. Offering the client emotional support 3. Placing the client in a left lateral position 4. Drawing the client's blood for laboratory screening

Correct 3. Placing the client in a left lateral position

The most appropriate method for a nurse to evaluate the effects of the maternal blood glucose level in the infant of a diabetic mother is by performing a heel stick blood test on the newborn. What specifically does this test determine? 1. Blood acidity 2. Glucose tolerance 3. Serum glucose level 4. Glycosylated hemoglobin level

Correct 3. Serum glucose level

Which microorganism causes maternal mastitis? 1. Escherichia coli 2. Group B streptococcus 3. Staphylococcus aureus 4. Chlamydia trachomatis

Correct 3. Staphylococcus aureus

A client is admitted with a marginal placenta previa. Which item should the nurse have readily available? 1. One unit of freeze-dried plasma 2. Vitamin K for intramuscular injection 3. Two units of typed and screened blood 4. Heparin sodium for intravenous injection

Correct 3. Two units of typed and screened blood

The nurse places fetal and uterine monitors on the abdomen of a client in labor. While observing the relationship between the fetal heart rate and uterine contractions, the nurse identifies four late decelerations. Which condition is most commonly associated with late decelerations? 1. Head compression 2. Maternal hypothyroidism 3. Uteroplacental insufficiency 4. Umbilical cord compression

Correct 3. Uteroplacental insufficiency

A client who is pregnant for the first time attends the prenatal clinic. She tells the nurse, "I'm worried about gaining too much weight, because I've heard that it's unhealthy." How should the nurse respond? 1. "Yes, too much weight gain results in complications during pregnancy." 2. "You'll have to follow a low-calorie diet if you gain more than 15 lb." 3. "We're more concerned that you won't gain enough weight to ensure adequate growth of your baby." 4. "A 25-lb (11.3-kg) weight gain is recommended; however, the pattern of weight gain is more important than the total amount."

Correct 4. "A 25-lb (11.3-kg) weight gain is recommended; however, the pattern of weight gain is more important than the total amount."

A new mother who has begun breastfeeding asks for assistance removing the baby from her breast. Which instruction is most appropriate for the nurse to provide? 1. "Pinch the baby's nostrils gently to help release the nipple." 2. "Let the baby nurse as long as desired without interruption." 3. "Pull your nipple out of the baby's mouth when the baby falls asleep." 4. "Insert your finger in the corner of the baby's mouth to break the suction."

Correct 4. "Insert your finger in the corner of the baby's mouth to break the suction."

A new mother asks the nurse administering erythromycin ophthalmic ointment to her newborn why her baby must be subjected to this procedure. What is the best response by the nurse? 1. "It will keep your baby from going blind." 2. "This ointment will protect your baby from bright lights." 3. "There is a law that newborns must be given this medicine." 4. "This antibiotic helps keep babies from contracting eye infections."

Correct 4. "This antibiotic helps keep babies from contracting eye infections."

A new mother exclaims to the nurse, "My baby looks like a conehead!" How should the nurse respond? 1. "Are you disappointed in how your baby looks?" 2. "Don't worry—your baby's head will be round in a few days." 3. "Is there anyone in your family whose head shape is similar to your baby's?" 4. "This often happens as the baby's head moves down the birth canal—the bones move for easier passage."

Correct 4. "This often happens as the baby's head moves down the birth canal—the bones move for easier passage."

A woman at 39 weeks' gestation whose membranes have ruptured at home arrives at the clinic to be evaluated. Assessment reveals mild irregular contractions 10 to 15 minutes apart, and a fetal heart rate (FHR) of 186 beats/min is auscultated between contractions. In light of this assessment, what does the nurse conclude? 1. The fetus is not at risk. 2. A precipitous birth is imminent. 3. This is a response to an infection. 4. A further assessment is necessary.

Correct 4. A further assessment is necessary.

When checking the cervical dilation of a client in labor, the nurse notes that the umbilical cord has prolapsed. What is the priority nursing action? 1. Taking the fetal heart rate 2. Turning the client on her side 3. Covering the cord with a sterile saline-soaked cloth 4. Assisting the client into the Trendelenburg position

Correct 4. Assisting the client into the Trendelenburg position

A client at 30-weeks' gestation is admitted to the hospital with a diagnosis of low-lying placenta previa with slight vaginal bleeding. The client is stabilized and bleeding ceases. What is the nurse's primary focus when providing discharge teaching about care at home for this client? 1.Stay on strict bed rest and use a bedpan. 2. Maintain a calm and quiet environment. 3. Check fetal status with a stethoscope daily. 4. Avoid anything that may stimulate the cervix or uterus.

Correct 4. Avoid anything that may stimulate the cervix or uterus.

The nurse determines that a newborn has a cephalhematoma. What did the nurse note? 1. Ridges where the cranial bones overlap 2. Edema involving the scalp over the occipital area 3. Pulsation of the cerebral arteries in the anterior and posterior fontanels 4. Bleeding between the parietal bone and periosteum confined within the suture line

Correct 4. Bleeding between the parietal bone and periosteum confined within the suture line

The nurse is assessing a postpartum client for signs of an impending hemorrhage resulting from laceration of the cervix. Besides monitoring the client for a firm uterus, what other assessment finding is important? 1. Slowed pulse rate 2. Increased blood pressure 3. Persistent muscular twitching 4. Continuous trickling of blood

Correct 4. Continuous trickling of blood

A nurse teaches a new mother about neonatal weight loss in the first 3 days of life. How does the nurse explain the cause of this weight loss? 1. An allergy to formula 2. A hypoglycemic response 3. Ineffective feeding techniques 4. Excretion of accumulated excess fluids

Correct 4. Excretion of accumulated excess fluids

While a multiparous client is in active labor, her membranes rupture spontaneously. The nurse notes a loop of umbilical cord protruding from her vagina. What is the priority nursing action at this time? 1. Monitoring the fetal heart rate 2. Covering the cord with a saline dressing 3. Pushing the cord back into the vaginal vault 4. Holding the presenting part away from the cord

Correct 4. Holding the presenting part away from the cord

A woman who had a home birth brings the infant to the well-baby clinic on the third day after the birth, and the infant weighs 5% less than at birth. What does the nurse suspect as the cause of this weight loss? 1. Viral or bacterial infection 2. Obstructive gastrointestinal anomaly 3. Generalized muscle response to stimulation 4. Imbalance between nutrient intake and fluid loss

Correct 4. Imbalance between nutrient intake and fluid loss

A pregnant client is scheduled for ultrasonography at the end of her first trimester. What should the nurse instruct her to do in preparation for the sonogram? 1. Empty her bladder. 2. Avoid eating for 8 hours. 3. Take a laxative the night before the test. 4. Increase fluid intake for 1 hour before the procedure.

Correct 4. Increase fluid intake for 1 hour before the procedure.

Upon arriving in the birthing room the nurse finds the client lying on her back with her head on a pillow and the bed in a flat position. The nurse explains that it is important to avoid lying in the supine position because of what reason? 1. It may precipitate a severe headache. 2. It can impede the progression of labor. 3. It may cause nausea as labor progresses. 4. It will prevent adequate blood flow to the fetus.

Correct 4. It will prevent adequate blood flow to the fetus.

What is the focus of nursing care for a newborn with respiratory distress syndrome (RDS)? 1. Tapping the toes to stimulate respirations 2. Turning the infant frequently to prevent apnea 3. Maintaining oxygen concentration at 40% to support respiration 4. Keeping the infant warm to maintain body temperature at 98° F (37° C)

Correct 4. Keeping the infant warm to maintain body temperature at 98° F (37° C)

The nurse is assigned to care for an adolescent who gave birth 12 hours ago. The client continually talks on the phone to her friends and does not respond when her new baby cries. What is the priority intervention at this time? 1. Calling social service for a consult 2. Calling the psychiatric team for an intervention 3. Calling her mother and having her speak with the client 4. Modeling appropriate behaviors that encourage infant bonding

Correct 4. Modeling appropriate behaviors that encourage infant bonding

A pregnant client with severe abdominal pain and heavy bleeding is being prepared for a cesarean birth. What is the priority medical intervention? 1. Teaching coughing and deep-breathing techniques 2. Sterilizing the surgical site and administering an enema 3. Providing a sterile gown and inserting an indwelling catheter 4. Obtaining informed consent and assessing the client for drug allergies

Correct 4. Obtaining informed consent and assessing the client for drug allergies

A postpartum client is being prepared for discharge. The laboratory report indicates that she has a white blood cell (WBC) count of 16,000/mm3. (16 X 109/L) What is the next nursing action? 1. Checking with the nurse manager to see whether the client may go home 2. Reassessing the client for signs of infection by taking her vital signs 3. Delaying the client's discharge until the practitioner has conducted a complete examination 4. Placing the report in the client's record because this is an expected postpartum finding

Correct 4. Placing the report in the client's record because this is an expected postpartum finding

A nurse is monitoring a client with severe preeclampsia for the onset of eclampsia. What objective clinical finding indicates an impending seizure? 1. Persistent headache with blurred vision 2. Epigastric pain with nausea and vomiting 3. Spots and flashes of light before the eyes 4. Rolling of the eyes to one side with a fixed stare

Correct 4. Rolling of the eyes to one side with a fixed stare

The nurse is caring for a postpartum client with a history of rheumatic heart disease. The nurse plans care for this client with what knowledge regarding this client? 1. She should increase her oral fluid intake. 2. She should maintain bed rest for a minimum of 4 days. 3. She is out of immediate danger, because the stress associated with pregnancy is over. 4. She requires monitoring during the first 48 hours because of the stress on the cardiopulmonary system.

Correct 4. She requires monitoring during the first 48 hours because of the stress on the cardiopulmonary system.

The nurse evaluates a new mother who is breastfeeding. The client asks how to care for her nipples. What should the nurse recommend? 1. Putting lanolin cream on the nipples after breastfeeding 2. Applying vitamin E gel to the nipples before breastfeeding 3. Using soap and water to clean the breasts and nipples at least once a day 4. Spreading breast milk on the nipples after the feeding and allowing them to air dry

Correct 4. Spreading breast milk on the nipples after the feeding and allowing them to air

A client's membranes rupture spontaneously during the latent phase of the first stage of labor, and the fluid is greenish brown. What does the nurse conclude? 1. Infection is present 2. Cesarean birth is necessary 3. Precipitate birth is imminent 4. The fetus may be compromised in utero

Correct 4. The fetus may be compromised in utero

During a newborn assessment the nurse counts the infant's cord vessels. What does the nurse expect to observe in a healthy newborn? 1. Two vessels: one vein and one artery 2. Three vessels: two veins and one artery 3. Four vessels: two veins and two arteries 4. Three vessels: one vein and two arteries

Correct 4. Three vessels: one vein and two arteries

The nurse is caring for a client whose contraction stress test result (CST) is positive. The nurse remains with the client and continues to assess the fetal and maternal monitor strips. Which complication does the nurse anticipate? 1. Preeclampsia 2. Placenta previa 3. Fetal prematurity 4. Uteroplacental insufficiency

Correct 4. Uteroplacental insufficiency

Which statements regarding the involution process are correct? Select all that apply. 1. Involution begins immediately after expulsion of the placenta. 2. Involution is the self-destruction of excess hypertrophied tissue. 3. Involution progresses rapidly during the next few days after birth. 4. Involution is the return of the uterus to a nonpregnant state after birth. 5. Involution may be caused by retained placental fragments and infections.

Correct. 1. Involution begins immediately after expulsion of the placenta. 3. Involution progresses rapidly during the next few days after birth. 4. Involution is the return of the uterus to a nonpregnant state after birth.


Ensembles d'études connexes

Chapter 7: PART 2 records retrieval and transfer

View Set

Страны и столицы Евразии

View Set

Chapter 9: Accounting for Receivables

View Set